You are on page 1of 244

INDIAN ASSOCIATION OF PHYSICS TEACHERS

NATIONAL STANDARD EXAMINATION IN PHYSICS 2007-2008


Total time : 120 minutes (A-1, A-2 & B)
PART - A (Total Marks : 180)
SUB-PART A-1
(A) (B)
Q.1 The distance traveled by an object is given by
x = at + bt2/ (c + a) where t is time and a, b, c,
are constants. The dimensions of b and c
respectively are :
(A) [L2T 3], [L T 1] (B) [L T 2], [L T 1]
(C) [LT 1], [L2 T 1] (D) [L T 1], [L T 2]
Sol. [A] m m F = 2mg
Use the dimensional analysis. Note that the 2m
dimensions of a and c are the same as those of (A) 1 : 1 (B) 1 : 2 (C) 1 : 3 (D) 2 : 1
[length/time] and those of b are [length ×
(length/time) (time)2]. Sol. [C]
For arrangement (A), the acceleration of mass
Q.2 A person throws vertically up n balls per m is obviously [(2m – m)g/(2m + m)] = g/3.
second with the same velocity. He throws a ball For arrangement (B), the net force acting on
whenever the previous one is at its highest mass m is mg upwards so that the acceleration
point. The height to which the balls rise is : is g only and hence the result.
(A) g/n2 (B) 2gn (C) g/2n2 (D) 2gn2
Sol. [C] Q.5 A particle of mass m is made to move with
The time required for a ball to reach highest uniform speed v along the perimeter of a
point is (1/n) second giving initial velocity to regular hexagon. Magnitude of impulse
be (g/n). Use this in v2 = u2 – 2 gh to find h. applied at each corner is :
(A) mv (B) mv 3
Q.3 A particle moves at a constant speed v from
(C) mv/2 (D) mv / 3
point A to point B along a circle of radius r. If
points A and B have an angular separation of T, Sol. [A]
the magnitude of change in velocity in moving The impulse is nothing but change in
from A to B is : momentum. For taking the difference of two
momenta, note that the angle between vectors
(A) zero (B) 2v sin (T/2)
at the corner is 120º.
(C) 2v sin T (D) 2v cos (T/2)
Sol. [B]
Q.6 The maximum tension in the string of a
The change in velocity is [vB – vA] = [vB = + (–vA)].
pendulum is three times the minimum tension.
Find the magnitude nothing that the angle
If T0 be the angular amplitude, cos T0 is
between vB and vA is (180º – T).
(A) 1/2 (B) 3/4
(C) 3/5 (D) 2/3
Q.4 Two pulley arrangements (A) and (B) are as
Sol. [C]
shown in the figure. Neglect the masses of the
Tension in the string is maximum [mg(3 – 2
ropes and pulleys and the friction at the axle of
cosT)] when it is in the vertical position
the pulleys. The ratio of the acceleration of
whereas tension is minimum [mg cosT] when
mass m in arrangement (A) to that in
arrangement (B) is the string is in the extreme position.

www.examrace.com
Q.7 A body of mass 4m at rest explodes into three Q.10 A ball floats on mercury in a container with
fragments. Two of the fragments, each of mass volume V1 inside mercury. If the container is
m move with speed v in mutually perpendicular now covered and the air inside is pumped out,
directions. Total kinetic energy released in the volume V2 is found to be under mercury.
process is : Then,
(A) V1 = V2 (B) V1 > V2
(A) mv2 (B) 3mv2/2
2 (C) V2 > V1 (D) V2 = 0
(C) 2mv (D) 3mv2 Sol. [C]
Sol. [B] Since air is pumped out, upthrust due to air
Consider the conservation of linear momentum becomes zero and the ball sinks slightly more
along two perpendicular directions X and Y than before.
axes, to get the velocity of the largest particle
[(v/2)(– i – j)]. Then calculate the kinetic Q.11 Pressure of one litre of nitrogen (J = 1.4) is
energy. 500 cm of mercury. It is compressed
adiabatically to 990 cc. The final pressure of
Q.8 Two masses m and M are attached to strings as the gas (in cm of mercury) is :
(A) 507 (B) 505
shown in the figure. In equilibrium tan T is
(C) 495 (D) 502
Sol. [A]
Use the expression for adiabatic change to get
T
(dp/p) + y (dV/V). After substintution dp = 7.
Note that there is an increase of pressure.
M
45° 45° Q.12 Two identical rings A and B are acted upon by
m torques WA and WB respectively. A is rotating
(A) 1 + (2M /m) (B) 1 + (2m /M) about an axis passing through the centre of
(C) 1 + (M/2m) (D) 1 + (m/2M) mass and perpendicular to the plane of the
Sol. [A] ring. B is rotating about a chord at a distance
Use the concept of balancing components of ( 1 2 ) times the radius of the ring. If the
forces and tensions along horizontal and angular acceleration of the rings is the same,
vertical directions at the two points where the then
masses m and M are attached. (A) WA = WB
(B) WA > WB
Q.9 Two bodies of masses 2 kg and 3 kg are (C) WA < WB
connected by a metal wire of cross section 0.04 (D) Nothing can be said about WA and WB as
mm2. Breaking stress of metal wire is 2.5 GPa. data are insufficient
The maximum force F that can be applied to 3 Sol. [A]
kg block so that wire does not break is : Using perpendicular and parallel axes
3 kg theorems it is found that the moments of
2 kg
F inertia in both the cases are the same.

Q.13 Two satellites S1 and S2 revolve around a


(A) 100 N (B) 150 N (C) 200 N (D) 250 N planet in coplanar circular orbits in the same
Sol. [D] sense. Their periods of revolution are 1 hour
If T is the tension in the string connecting the and 8 hour respectively. The radius of the orbit
two bodies, (T/A) d the breaking stress, where of S1 is 104km. When S1 is closest to S2, the
A is the area of cross section. Deduce the angular speed of S2 as observed by an
relation T = (2/5) F and then the result. astronaut in S1 is
(A) S rad/hr (B) S/3 rad/hr
(C) 2S rad/hr (D) S/2 rad/hr

www.examrace.com
Sol. [B] Q.18 A long thin rod AB is charged uniformly. The
Use Kepler's law to get the radius of orbit of S2 electric field at a point C is directed
to be 4 × 104 km. The linear speeds of S1 and S2 C
happen to be 2S × 104 km/hr and S × 104 km/hr.
Then, the angular speed of S2 as seen from S1 is
A + + + + + + + + + + + + + + B
('v/'r).
(A) parallel to the rod
(B) perpendicular to the rod
Q.14 If a section of a soap bubble of radius r by a (C) along the bisector of the angle ACB
plane through its centre is considered, the force (D) along a line dividing the angle ACB in the
on the half due to surface tension is : ratio BC : AC
(A) 2S rT (B) 4S rT Sol. [C]
2
(C) S r T (D) 2r T Check it by considering two elements at the
Sol. [B] two ends then two symmetrically situated
Note that force = 2 (2Sr) T. elements of the rod.
Q.19 An electric field is given by E = yî  xˆj
Q.15 Volume of a gas (Cp/CV = J) expands from a volt/m. The work done in moving a charge of
volume V to 2V at constant pressure p. The
10μC from a point r1 = 3î  4 ĵ to another
change in internal energy is :
(A) R / (J – 1) (B) pV r2 = 2î  6 ĵ is :
(C) pV / (J – 1) (D) J pV (J – 1)
(A) 10 5 J (B) – 10 5 J
Sol. [C]
Use the expression for the change in internal (C) 10 2 × 10–6 J (D) zero
energy = nCV dT. Also use the substitution Sol. [D]
CV = R / (J – 1) and pV/T = constant. Use the relation : work done = charge ×
potential difference. Then, the potential
Q.16 A satellite is revolving round the earth with difference (-³E· dr) comes out to be negative
orbital speed v0. If it imagined to stop integral of (y dx + x dy), that is, of [d (xy)] to
suddenly, the speed with which it will strike the
be evaluated between(3,4) and (2, 6).
surface of the earth would be (ve – escape
velocity of a body from earth's surface)
(A) ve2 /v0 (B) v0 Q.20 When the switch S is closed in the circuit
(C) (ve2 – v02)1/2 (D) (ve2 – 2 v02)1/2 shown below, the current that flows through it
Sol. [D] is
Use the concept of conservation of energy and 10 : 15 :
the expressions for escape velocity and orbital + 15 V + 15 V
velocity.
20 :
Q.17 A car moves with a speed of 60 km /hr from
point A to point B and then with the speed of S
40 km / hr from point B to point C. Further it
moves to a point D with a speed equal to its
average speed between A and C. Points A, B, C (A) zero (B) 15/26 A
and D are collinear and equidistant. The (C) 15/13 A (D) 5/26 A
average speed of the car between A and D is : Sol. [B]
(A) 30 km/ hr (B) 50 km / hr Take the potential at the junction of resistors to
(C) 48 km / hr (D) 60 km / hr be V and then use Kirchhoff' s current law at
Sol. [C]
this junction. Obtain V and then the current
Noting that AB = BC, average speed between
A and C is 48 km/hr and that between A and D through 20 ohm resistor, that is, the switch.
is also 48 km/hr.

www.examrace.com
Q.21 In a standing wave formed as a result of Q.25 In case of an adiabatic process the correct
reflection from a surface, the ratio of the relation in terms of pressure p and density U of
amplitude at an antinode to that at node is x.
a gas is
The fraction of energy that is reflected is :
(A) [(x – 1) / x]2 (B) [x / (x + 1)]2 (A) p UJ = constant (B) pJ UJ 1 constant
(C) p UJ 1 = constant (D) p U J= constant
2
(C) [(x – 1)/(x + 1)] (D) [1/x]2
Sol. [C] Sol. [D]
If ai and ar denote the amplitudes of the incident
Note that pVJ = constant and that U is inversely
and the reflected waves then the net amplitude
at the antinode is (ai + ar) and that at the node proportional to V.
is (ai – ar). From this we get (ar / ai) Q.26 Three rods of the same cross section and made
= [(x – 1)/(x + 1)]. Note that the energy is of the same material form the sides of a
proportional to square of the amplitude. triangle ABC as shown. The points A and B
are maintained at temperatures T and 2T
Q.22 The fundamental frequency of a sonometer respectively in the steady state. Assuming that
wire of length l is n0. A bridge is now only heat conduction takes place, the temperature
introduced at a distance of 'l (<< l) from the at point C is :
centre of the wire. The lengths of wire on the A
two sides of the bridge are now vibrated in
60°
their fundamental modes. Then, the beat
frequency nearly is :
(A) n0 'l / l (B) 8 n0 'l / l
(C) 2 n0 'l / l (D) n0 'l / 2l B C
Sol. [B] (A) [(2 2  3 ) /(2  3 )]T
Note that the beat frequency is (n1 = n2) and the
(B) [3 2 /(2  3 )]T
corresponding vibrating lengths are (l/2 – ')
and (l/2 + 'l). (C) [2 / 3 ]T
(D) [ 5 / 2]T
Q.23 Two open organ pipes of fundamental
frequencies n1 and n2 are joined in series. The Sol. [A]
fundamental frequency of the new pipe so In s steady state, the heat conducted from B to
obtained will be : C is the same as from C to A.
(A) n1 + n2 (B) n1n2/ (n1 + n2)
(C) n1n2/(n1 – n2) (D) n12  n 2 2 Q.27 A soap bubble filled with helium floats in air.
Sol. [C] Let mw and mHe be the mass of the wall of the
Lengths of the organ pipes are (v/2n1) and bubble and that of the helium gas inside,
(v/2n2) where v is the speed of sound in air. respectively. If density of helium is 0.1384
The fundamental frequency of the new organ
times that of air, then :
pipe (after the two are joined) will be [v/2(n1 +
n2)]. (A) mw > mHe
(B) mw < mHe
Q.24 The specific heat of a solid at low temperature (C) mw = mHe
varies according to the relation c = k T3 where (D) nothing can be said about mw and mHe as
k is a constant. The heat required to raise the data are insufficient
temperature of a mass m of such a solid from
Sol. [A]
T = 0 K to T = 20 K is Density of helium is less than half the density
(A) 2 × 104 mk (B) 4 × 104 mk of air and hence the mass of helium is less than
(C) 8 × 104 mk (D) 16 × 104 mk half the mass of air. Note that (mw + mHe) d
Sol. [B] mair.
Integrate (mCdT) between 0 K and 20 K.

www.examrace.com
Q.28 A ball A moving with certain velocity in Q.31 A charge + 6μC is situated inside a closed
positive X axis direction collides with a surface. The flux, in volt-m, through a portion
of the surface subtending a solid angle of (S/2)
stationary ball B. After collision their
at the point where the charge is situated is
directions of motion make angles D and E with (A) + (1.5/H0) × 10 6 (B) + (6/H0) × 10 6
the X axis. The possible values of D and E are (C) – (1.5/H0) × 10 6 (D) + (0.75/H0) × 10 6
Sol. [A]
(A) D = 30° and E = – 45°
Note that the flux will be [(S/2)/4S] (q/H0)
(B) D = 90° and E = – 120°
(C) D = 0° and E = – 30° Q.32 A metal sheet 14 cm × 2cm of uniform
thickness is cut into two pieces of width two
(D) D = 45° and E = 0° cm. The two pieces are joined and laid along
Sol. [A] XY plane as shown. The centre of mass has
This is the only possibility where the condition the coordinates
for conservation of momentum can be satisfied. Y

Q.29 Two identical thin planoconvex lenses of


refractive index n are silvered, one on the plane
side and the other on the convex side. The ratio
of their focal lengths is :
X
(A) n / (n – 1) (B) (n – 1) / n O
(C) (n + 1) / n (D) n (A) (1, 1) (B) (19/7, 19/7)
Sol. [A] (C) (8/7, 8/7) (D) (12/7, 12/7)
When the plane face of a planoconvex lens is Sol. [B]
coated, the focal length is given by R/[2(n – 1)] Note that the centres of mass of the vertical
and it is R/2n when the convex face is coated. and the horizontal pieces are (1, 4) and (5, 1)
respectively and their masses are in the ratio
Q.30 Two plane mirrors subtend angle T between 4 : 3.
them. A ray of light incident parallel to one of
Q.33 In a double slit experiment, the wavelength of
them retraces its path after n reflections. The monochromatic light used is O and the distance
graph of T (Y axis) versus n (X axis) is : between the slits is d. The screen is at a
(A) a straight line through origin distance D from the slits. If a bright fringe is
(B) a parabola formed opposite to a slit on the screen, the
order of the fringe is :
(C) a rectangular hyperbola
(A) d/2D (B) d2/OD
(D) a straight line with a intercept on Y axis 2
(C) d /2O D (D) O D /d2
Sol. [C] Sol. [C]
Refer to the ray diagram and note that for four The distance from the centre of the screen is
reflections from the two mirrors, we have d/2 = nX where X is the fringe width and n is
90°– 3T = T, so that T = 90° /4 = 90°/(number the order of the fringe.
of reflections n). Therefore, the product of T
and n is a constant so that the graph is a Q.34 A lens formed by two watch glasses, as shown,
rectangular hyperbola. behaves like a

90–3T
3T
3T
2T 2T
T TT (A) convex lens (B) concave lens
(C) glass plate (D) mirror

www.examrace.com
Sol. [C] Sol. [C]
Note that for each of the watch glasses, the two Note that the emf induced in the inductor is [L
radii of curvature happen to be the same so that (di/dt)] and energy stored per unit time is the
their focal lengths happen to be infinite. power, that is, [emf × current]
Q.39 According to Bohr theory, for the stability of
Q.35 Six resistors each of 10 ohm are connected as
an atom, angular momentum of an electron in
shown. The equivalent resistance between
points X and Y is : an orbit is quantized. This is a necessary
X condition according to :
(A) Pauli's exclusion principle
(B) the concept of wave associated with an
electron
(C) correspondence principle
Y (D) none of the above
Sol. [B]
(A) 20 ohm (B) 5 ohm
(C) 25/3 ohm (D) 10 ohm Note that angular momentum mvr = n(h/2 S)
giving n [h/(mv)] = 2 S r. Note that [h/(mv)] is
Sol. [B] the de Broglie wavelength.
Redrawing the circuit reveals that five of the
resistors form a balanced Wheatstone's network Q.40 A radioactive substance with decay constant of
between points X and Y with the remaining 0.5/s is being produced at a constant rate of 50
resistor in parallel appearing across X and Y. nuclei per second. If there are no nuclei
present initially, the time (in second) after
Q.36 Two long parallel straight conductors carry which 25 nuclei will be present is :
currents i1 and i2 (i1 > i2).When the currents are (A) 1 (B) 2 ln (4/3)
in the same direction, the magnetic field at a (C) ln 2 (D) ln (4/3)
point midway between the wires is 20 μT. If Sol. [B]
the direction of i2 is reversed, the field becomes Use the relation di/dt = 50 – ON and integrate
50 μT. The ratio of the currents i1/i2 is : to get N = 50/O [1 – e Ot]. Then use O = 0.5/s
(A) 5/2 (B) 7/3 (C) 4/3 (D) 5/3 and N = 25.
Sol. [B]
Note that when the currents are in the sense the
Sub-Part A-2
magnetic fields due the two wires subtract and
when the currents are in opposite sense they
add.
Q.41 Two particles having the same specific charge
Q.37 Magnetic field at the centre of a circular loop enter a uniform magnetic field with the same
of area A is B. The magnetic moment of the speed but at angles of 30° and 60° with the
loop is : field. Let a, b, and c be the ratios of their
(A) BA2/(μ0S) (B) BA A / μ 0 periods, radii and pitches of their helical paths
respectively, then
(C) BA A /(μ 0 S) (D) 2BA A /(μ 0 S )
Sol. [D] (A) abc > 1 (B) a + b = 2 C
2
Use the expression for the magnetic field at the (C) a = C (D) a b = C
centre of a circular coil [B = (μ0 i) / 2r]and that Sol. [A,B,C,D]
for the magnetic moment [M = iA]. Note that Use the expression for periodic time T = (2S m)
area A = Sr2. /(Bq sin T), for radius of the circular path
Q.38 A current of 1A through a coil of inductance of r = (mv)/(Bq sin T) and for the pitch of the
200 mH is increasing at a rate of 0.5 A/s. The helical path p = T (v cos T) where the symbols
energy stored in the inductor per second is : their usual meanings. With this we get, a = b = 3
(A) 0.5 J/s (B) 5.0 J/s
(C) 0.1 J/s (D) 2.0 J/s and c = 3.

www.examrace.com
Q.42 Let v and a bet he instantaneous velocity and Q.46 A ball A moving with a velocity 5 m/s collides
the acceleration respectively of a particle elastically with another identical ball at rest
moving in a plane. The rate of change of speed such that the velocity of A makes an angle of
(dv/dt) of the particle is : 30° with the line joining the centres of the
(A) | a | balls. Then,
(B) (v ˜ a)/ | v |
(A) speed of A after collision is (5/2) m/s
(C) the component of a in the direction of v
(D) the component of a perpendicular to v (B) speed of B after collision is (5 3 / 2) m/s
(C) balls A and B move at right angles after
Sol. [B,C] collision
Use v2 = vX2 + vy2, differentiate to get (dv/dt) (D) Kinetic energy is not conserved as the
= (vx ax + vy ay)/ v = (v ˜ a)/ v. Also note that collision is not head-on
(v/v) is the unit vector along v.
Sol. [A,B,C]
Consider the conservation of kinetic energy
Q.43 A piece of metal weighs 210 g in air, 180 g in
water and 120 g in a liquid. Then, specific and the fact that the momentum must be
gravity of : conserved along two directions, namely (i) line
(A) metal is 3 (B) metal is 7 joining their centres and, (ii) perpendicular to
(C) liquid is 3 (D) liquid is 1/3 that line.
Sol. [B,C]
Use relation : relative density of metal = Wair/ Q.47 For any monoatomic gas the quantity/quantities
(Wair – Wwater) and loss of weight in liquid = independent of the nature of the gas at the
upthrust in liquid. same temperature is /are
(A) the number of molecules is one mole
Q.44 Two sphere A and B have the same radii but
(B) the number of molecules in equal volume
the heat capacity of A is greater than that of B.
The surfaces of both are painted black. They (C) the translational kinetic energy of one
are heated to the same temperature and allowed mole
to cool in vacuum. Then, (D) the kinetic energy of unit mass
(A) A cools faster than B Sol. [A,C]
(B) both A and B cool at the same rate Consider the properties of one mole of any gas
(C) at any temperature the ratio of their rates of at a given temperature.
cooling is a constant
(D) B cools faster than A Q.48 A square coil OLMN of side 5 cm is placed in
Sol. [C,D]
a magnetic field B = 3kt2 (as shown in the
At any temperature T, both the spheres lose
figure) where k is a constant, B is in tesla and t
heat at the same rate which is C (dT/dt) in
is in second. At time t = 5 second
general.
X
O N
Q.45 Two different coils have self inductance
L1 = 10 mH and L2 = 12 mH. the currents in
both the coils are increased at the same rate and &
the power in the two coils is also the same at X X X B
some instant of time. At that instant of time let
i1, V1 and W1 be the current, the induced emf
and the energy stored respectively in the first L M
X
coil. Let i2, V2 and W2 be the corresponding (A) induced current flows from O to L
quantities for the second coil. Then, (B) induced current flows from L to O
(A) i1/i2 = 6/5 (B) i1/i2 = 5/6
(C) induced emf is 75k mV
(C) W2/W1 = 5/6 (D) V2/V1 = 6/5
(D) induced emf is 1.875 k V
Sol. [A,C,D]
Use the relation power P = Vi = L(di/dt) i. Sol. [A,C]
Since P and (di/dt) are the same, L1i1 = L2i2. Use the expression : emf induced = dI/dt = l2
Note that the energy stored in an inductor is (dB/dt) and consider the sense in which the
(1/2 Li2). induced emf sends the current.

www.examrace.com
Q.49 Two card A and B are moving in the same m
direction with speeds 36 km/hr and 54 km/hr
respectively. Car B is ahead of A. If A sounds "1 "0
horn of frequency 1000 Hz and the speed of Sol. The length of the stretched spring
sound in air is 340 m/s, the frequency of sound
l = (l02 + x2)1/2 | l0 + x2 (2l0) so that the
received by the driver of car B is
extension of the spring is (l – l0) = x2 / (2l0).
(A) 928.57 Hz (B) 984.84 Hz
(C) 946.37 Hz (D) 938.47 Hz Now the tension in the spring will be T = k
Sol. [B] [x2/(2l0)]. The restoring force will be 2 T sin
Noting that speed of the observer is 15 m/s and T = kx3 / l02where sin T | T = x/l. This gives
that of the source is 10 m/s apply Doppler's the acceleration proportional to x3. The motion
relation for the apparent frequency. is obviously not simple harmonic.
m
Q.50 A glass prism is immersed in a hypothetical
liquid. The curves showing the refractive index x
n as a function of wavelength O for glass and
l0 l0
liquid are as shown in the figure. When a ray of
white light is incident on the prism parallel to
the base –
n Q.52 A body of mass m is projected inside a liquid
liquid at an angle T0 with horizontal at an initial
velocity v0. If the liquid develops a velocity
dependent force F = – kv where k is a positive
constant, determine the x and the y
components of the velocity at any instant. Also
glass determine its range.
Sol. To determine the component of velocity in the
O Oyellow O horizontal direction, consider the equation
(A) yellow ray travels without deviation Fx = – k vx and integrate to get ln v = – (kt/m)
(B) blue ray is deviated towards the vertex + ln C where the constant of integration C can
(C) red ray is deviated towards the base be determined by using the initial condition
(D) there is no dispersion that at t = 0, vx = v0 cos T0. This gives after
Sol. [A,B,C]
substitution vx = v0 cos T0 (e kt/m). Similarly
Note that the refractive indices of both the glass
and the liquid are the same for yellow and the vertical component of velocity can be
hence no deviation. However, red ray enters determined by considering the equation
from a rarer to a relatively denser medium Fy = – kvy – mg. While integrating we use the
while blue ray enters from a denser medium to condition that at t = 0, vy = v0 sin T0. This
a relatively rarer medium and hence the gives vy = (m/k) {[k/m) v0 sin T0 + g] e kt/m –
corresponding deviations. g}. The range can then be determined by
writing Fx = – k vx as m (dvxdt) = – k vx and
PART B Marks : 60 further as mvx(dvxdx) = – k vx. this given the
* All questions are compulsory. range xmax = (m v0 cos T0)/k.
* All questions carry equal marks
Q.53 Identical thin rods of length 2l carry equal
Q.51 A small bob joins two light unstretched
charges + q, uniformly distributed along their
identical springs fixed at their far ends and
arranged along a straight line, as shown in the lengths. The rods lie along X axis with their
figure. The bob is displaced in a direction centres separated by a distance of a > 2l. Show
perpendicular to the line of the springs by x that the magnitude of the force exerted by one
(<<I0) and then released. Obtain an expression rod on the other is given by
for the acceleration of the bob in terms of the F = (1/4SH0) (q2/4l2) ln [a2/(a2 – 4 l2)]
displacement x. Is the motion simple harmonic ?

www.examrace.com
Sol. Referring to the figure, write the electric field Q.55 A container of volume 0.02 m3 contains a
at a point distance d from one end of the rod. mixture of neon and argon gases at a
This comes out to be [(1/4SH0)q/ {d(d + l)}]. temperature of 27°C and pressure of 1 × 105
Using this write the electric field dE at a point N/m2. The total mass of the mixture is 28g. If
the molecular weights of neon and argon are
distance x from the origin where the centre of
20 and 40 respectively, determine the masses
one of the rods is situated. This is given by dE of the individual gases in the mixture,
= (1/4SH0) q/[(x – l) (x + l)]. From this, the assuming them to be ideal. (R = 8.314 J/mole
force dF on a small element of charge Odx of K)
the other rod can be written as (dE × dq). Sol. Let m1 and m2 be the masses of neon and
integrating this relation between (a – l) and argon respectively. Then, (m1 + m2) = 28. The
(a + l) we get the result. number of moles of the two gases are (m1/M1)
ª 1 q º and (m2/M2) where M1 and M2 are the
E= « ˜ »
q ¬ 4SH 0 d(d  2) ¼ corresponding molecular weights. Using
pV = nRT where n represents the total number
2L d
of moles, we get m1 = 4 g and m2 = 24g

Y Q.56 A metal rod of length 1 m is clamped at two


dx points as shown in the figure. Find the
x minimum frequency of natural longitudinal
q q X oscillation of the rod. (Young's modulus
–L O (a + l) Y = 1.6 × 1011 N/m2, density of metal
+L a–L
a U = 2500 kg/m3 )
P Q
R S
Q.54 A liquid is kept in a cylindrical vessel which is
rotating about its axis, as a result of which the
liquid rises at the sides. Show that the section 5 cm 15 cm
of the surface of the liquid by a vertical plane Sol. Using the relation v = (Y/U)1/2 the speed of
containing the axis is a parabola. Determine the longitudinal wave is 8000 m/s. Nodes will be
difference in height of the liquid at the centre formed at the clamped positions and there
of the vessel and its sides.
should be integral number of loops between
Sol. Referring to the figure, consider a particle at a
the nodes. If a denotes the number of loops
point P on the surface. We have N cos T = m g
and N sin T = m x Z2, giving tan T = (x Z2) / g. between P and Q, then a(O/2) = 80 or aO =
The Slope of the curve (dy/dx) is itself tan T. 160. Now, since R and S are free ends, the
With this we get the differential equation number of loops between P-R and Q-S must be
(dy/dx) = (x Z2) / g. Solve this differential odd multiples of (O/4) say, b and c
equation using the conditions that at x = 0, respectively. This gives (2b – 1) (O/4) = 5 and
y = 0 and at x = r, y = h. This gives h = (Z2 r2) / (2c – 1) (O/4) = 15. For frequency to be
(2g). minimum, a, b and c must be smallest integers.
Y With this a = 8, b = 1 and c = 3, and then,
frequency comes to be 40 kHz.
r
section Q.57 The mercury thread in a glass thermometer
by a appears to be half as thick as the cylindrical
N T
vertical h stem. Calculate the actual diameter o the
plane P (x, y) mercury thread if the actual diameter of the
mg stem is 3mm. Refractive index of glass is 1.5.
X How does the answer depend upon the
O
external diameter ? Draw a next ray diagram.

www.examrace.com
Sol. Refer to the ray diagram and write sin i1 = r/R
and sin i2 = r'/R. This gives the refractive index
n = sin i2 / sin i1 = r' / r = (apparent radius) / S1
(actual radius) giving actual radius as 1 mm. * O
S S2
the answer is obviously independent of the Screen
external diameter R.
r o actual radius Sol. Let the central maximum be formed at P at a
i2 rco apparent radius distance x from O. Then, the path difference
i1 O o Point from where {S2P + [(μg/μm) – 1] t} – S1P = 0, where t is
R a ray starts the thickness of the glass plate and μg, μm are
rc O the refractive indices of the glass of the plate
i2 r and the medium respectively. This gives x =
4.33 mm. The path difference for waves
reaching at point O is [(μg / μm) – 1] t which
corresponds to a phase difference of (13S/3). If
I0 denotes the intensity at point O and Imax the
maximum intensity, then the ratio I0/Imax = ¾.
Q.58 A glass prism in the shape of a quarter cylinder Note that the net intensity I at any point due to
lies on a horizontal table. A horizontal beam of two waves with intensities I1 and I2 is [I1 + I2 +
light falls on its vertical plane surface, as 2 (I1I2)1/2 cos G], where G is the phase
shown. If the radius R of they cylinder is 3 cm difference.
and the refractive index n of the glass is 1.5,
where on the table beyond the cylinder will a Q.60 It is proposed to use the nuclear reaction
210
patch of light be formed ? 84Po o 82Pb206 + 2He4 to produce 2kW of
electric power in a generator. The half life of
polonium (84Po210) is 138.6 days. Assuming
efficiency of the generator to be 10%,
calculate how many grams of polonium are
Sol. Referring to the ray diagram, we have, sin required per day.
Tc = 1/n and cos Tc = R/(R + x) giving x = 1.03 [Masses of nuclei : Po210 – 209.98264 amu,
cm. Now consider the lower part of the Pb206 – 205.97440 amu, He4 – 4.00260 amu
cylinder as a planoconvex lens to get f = 2R = and 1 amu = 931 MeV]
6. cm. Thus the path of light will be found Sol. The mass difference ('m) between the two
between 1.03 cm and 3 cm beyond the cylinder. isotopes is 0.00564 amu which is equivalent to
5.25 MeV. The decay constant O turns out to
be (0.693/138.6) = 0.005 /day. If M grams of
TC Po210 are required per day, the number of
nuclei in M grams is about (6 × 1023/210) M =
R N, say. Then, ON should be the number of
TC nuclei present. The energy produced per day
R x will then be ON (5.25 MeV × 1.6 × 10 19) J.
This energy is expected to be (2kW × 24 × 60
Q.59 The Young's double slit experiment is done in × 60) J. Equating the two we get M = 14.4
water of refractive index 4/3. A light source of gram. Since the efficiency is 10%, the amount
wavelength 6000Å is used and the slits are 0.45 required is 144 gram.
mm apart. One of the slits is covered by a glass
plate of thickness 10.4 μm and refractive index
3/2. The interference pattern is observed on a
screen placed 1.5 m from the slits. Determine
(1) the location of the central maximum, and
(2) the intensity of light at point O relative to
the maximum intensity.

www.examrace.com
INDIAN ASSOCIATION OF PHYSICS TEACHERS
NATIONAL STANDARD EXAMINATION IN PHYSICS 2008-2009
Total time : 120 minutes (A-1, A-2 & B)
PART - A (Total Marks : 180)
SUB-PART A-1

Q.1 The quantity [(nh)/2SqB]1/2 where n is a Q.4 A small bob attached to a string of length l is
positive integer, h is Plank's constant, q is suspended from a rigid support and rotates
charge and B is magnetic field, has the with uniform speed along a circle in a
dimensions of - horizontal plane. Let T be the angle made by
(A) area (B) speed the string with the vertical. Then the length of
(C) length (D) acceleration a simple pendulum having the same period is -
Sol. [C] (A) l/cos T (B) l sin T
Consider the expression for the energy of a
(C) l/sin T (D) l cos T
quantum E = hv to get the dimensions of h as
those of (energy X time). Again the expression for Sol. [D]
the force F acting on a charge q moving with The period of revolution of conical pendulum is
velocity v in a magnetic field B is qvB sin T. This l cos T
gives the dimensions of qB as those of T = 2S . This can be compared with the
(force/velocity). Use these to determine the g
dimensions of the quantity under consideration, expression of period of a simple pendulum.
nothing that n is a dimensionless quantity.
Q.5 Acceleration – time graph of a particle moving
Q.2 In vernier calipers, m divisions of main scale along X axis is as shown. The particle will
coincide with (m + 1) divisions of vernier have the velocity same as its initial velocity at –
scale, If each division of main scale is d units, a
the least count of instrument is - m/s2
(A) d/ (m – 1) (B) d / (m +1)
(C) d /m (D) md / (m + 1)
Sol. [B] t
Since m divisions of main scale are equivalent to O 5 10 S
(m + 1) divisions of vernier scale, one division of
vernier scale is equivalent to [m/(m+1)] divisions (A) 10 s (B) (10 + 3 ) s
of main scale. Now, use the definition : least (C) (10 + 5 3 ) s (D) (10 + 2 3 ) s
count of vernier = one division of main scale –
Sol. [C]
one division of vernier scale.
Let A be the acceleration at t = 0. Change in
velocity in first 10 second will be 5A + (1/2) 5A
Q.3 Vectors a and b include an angle T between = 15A/2 which is equal to the area under the
them. If (a + b) and (a – b) respectively subtend curve. To have velocity same as the initial
angles D and E with a, then (tan D + tan E) is velocity, the net change in velocity should be
(A) (ab sin T) / (a2 + b2 cos2T) zero. For this the area under the curve has to be
(B) (2ab sin T)/ (a2 – b2cos2T) § 15A ·
(C) (a2 sin2 T) / (a2 + b2cos2 T) ¨– ¸ . Note that the area below the X axis is
© 2 ¹
(D) (b2 sin2 T) / (a2 – b2cos2 T) negative. Slope of straight line after t = 5s is
Sol. [B] (– A/5). If point (10, 0) is used as origin,
b sin T equation of the line will be given by
Use the relation : tan D = where D is
a  b cos T y = (– A/5)x. At x = t1, y = (– A/5)t1. Now, area
the angle made by the vector (a + b) with a. between x = 10 and x = t1 will be [(1/2)t1 X
b sin T (– A/5)t1] = (– t12A)/10. This can be equated to
Similarly, tan E = where E is the (– 15A/2) so as to get t12 = 75 or t1 = 5"3s. this is
a – b cos T
the time from 10 s and hence the total time is
angle made by the vector (a – b) with a. Note that
(10 + 5"3) second.
angle between a and (– b) is (180° – T).

www.examrace.com
Q.6 A block of mass m is placed on a inclined plane cylinder
with angle of inclination T. Let N, fL and F
respectively represents the normal reaction,
limiting force of friction and the net force down
the inclined plane. Let μ be the coefficient of
friction. The dependence of N, fL and F on T is
indicated by plotting graphs as shown below.
Then, curves (1), (2) and (3) respectively
represent (A) 2 ms 2 (B) 4 ms 2
2
(C) 8 ms (D) 6 ms 2
Sol. [B]
(2) Consider the cart moving to the right with
(1) acceleration a. Then, a pseudo force ma acts on
(3) the cylinder (of mass m) to the left. In the
limiting case, taking the moments about a point
of contact of the cylinder with the cart on the
O S/2 T
left, clockwise moment (ma X h/2) =
anticlockwise moment (ma X d/2) where h and d
(A) N, F and fL (B) F, fL and N are the height and diameter of the cylinder
(C) F, N and fL (D) fL, N and F respectively.
Sol. [C]
The normal reaction N (= mg cos T) and the
Q.9 Water flows out of the hole on the side of a
frictional force fL(= μ mg cos T) vary as cosine of
bucket and follows a parabolic path. If the
the angle of inclination T. The net downward
force is given by [mg sin T – μ mg cos T]. Note bucket falls freely under gravity, ignoring air
that the downward force is zero up to the angle of resistance, the water flow -
repose, that is, till the frictional force has not (A) follows a straight line path relative the
attained its limiting value. falling bucket
(B) follows a parabolic path relative the falling
Q.7 When a solid sphere rolls without slipping bucket
down an inclined plane making an angle T with (C) decreases but continues to flow
the horizontal, the acceleration of its centre of (D) stops
mass is a. If the same sphere slides without
Sol. [D]
friction its acceleration ac is
Under a free fall the water head above the hole
(A) (7/2) a (B) (5/7) a
does not exert any pressure. Then, the pressures
(C) (7/5) a (D) (5/2) a
on the two sides of the hole become equal and
Sol. [C]
hence the flow stops.
The acceleration of the sphere when it slides
without friction is (g sin T). When the sphere rolls
without slipping, the acceleration is (5/7) (g sin T) Q.10 A spring has length l and spring constant k. It
and hence the answer. is cut into two pieces of lengths l1 and l2 such
that l1 = nl2. such that l1 = nl2. The force
Q.8 A 2 kg cylinder rests on a cart as shown in the constant of spring of length l1 is
figure. The coefficient of static friction (A) k (l + n) (B) k (l + n) / n
between the cylinder and the cart is 0.5. The (C) k (D) k/(l + n)
cylinder is 4 cm in diameter and 10 cm in Sol. [B]
height. Taking g = 10 m/s2, the minimum The length l1 = [n/(n + 1)]l. Noting that the
acceleration of the cart needed to make the elongation is proportional to the length, the force
cylinder tip over is about
constant becomes [k (n + 1)/n].

www.examrace.com
Q.11 Consider two hollow glass spheres, one Q.13 A disc of radius R = 10 cm oscillates as a
containing water and the other containing physical pendulum about an axis perpendicular
mercury. Each liquid fills about one tenth of to the pane of the disc at a distance r from its
the volume of the sphere. In zero gravity center. If r = R/4, the approximate period of
environment - oscillation is -
(A) 0.84 s (B) 0.94 s
(A) water and mercury float freely inside the
(C) 1.26 s (D) 1.42 s
spheres
Sol. [B]
(B) water forms a layer on the glass while Use the expression for the periodic time of a
mercury floats
I
(C) mercury forms a layer on the glass while physical pendulum : T = 2  where I is
mgh
water floats
the moment of inertia of the disc about the axis
(D) water and mercury both form a layer on the
under consideration. In this case it is (mR2/2 +
glass mR2/16) = 9mR2/16. The distance of the centre
Sol. [B] of mass from the point of suspension is h = R/4.
Note that cohesive force among mercury
molecules is greater than adhesive force between Q.14 Sachin (55 kg) and Kapil (65 kg) are sitting at
glass and mercury molecules, Also, adhesive the two ends of a boat at rest in still water. The
force between water and glass molecules is boat weighs 100 kg and is 3.0 m long. Sachin
greater than cohesive force among water walks down to Kapil and shakes hand. the boat
molecules. gets displaced by -
(A) zero m (B) 0.75 m
Q.12 A uniformly thick plate in the shape of an (C) 3.0 m (D) 2.3 m
arrowhead has dimensions as shown. The Sol. [B]
Kapil and the boat can be considered as one body
centre of mass lies at a point
of mass mb = (65 +100) = 165 kg. Note that the
centre of mass of the system remains unchanged
since no external force acts on the system. Let
O mS be the mass of Sachin and 'xs,'xb be the
6 cm
displacements of the combined body of mass mb
and Sachin respectively with reference to the
centre of mass. Then use the equation ms'xs +
3 6 mb'xb = 0, to get the answer.
cm cm
(A) 1.5 cm to the right of O
Q.15 A uniform solid disc of radius R and mass m is
(B) 3 cm to the right of O free to rotate on a frictionless pivot through a
(C) O itself point on its rim. The disc is released from rest
(D) 1 cm to the right of O in the position where the diameter through the
Sol. [D] pivot is along horizontal. The speed of its
Note that the centre of mass of a uniform thick centre of mass when the diameter through the
triangular sheet is at the centroid which divides a pivot is vertical is
median in a ratio 2 : 1. The required center of (A) (2/3)(gR)1/2 (B) (gR)1/2
mass must be on the line of symmetry passing (C) (2gR)1/2 (D) 2(gR/3)1/2
through O and the vertex to the right. Consider Sol. [D]
the shape of arrowhead to be obtained by cutting Moment of inertia about the axis through the
the triangular part to the left (with base 6 cm and pivot on the rim of the disc is (mR2/2 + mR2) =
height 3 cm) from the uncut triangular sheet with 3mR2/2. using the principle that change in
potential energy (mgR) is equal to the gain of
base 6 cm and height 9 cm. The ratio of masses of
kinetic energy (IZ2/2) gives the answer. Also use
these two is 27 : 9, the thickness being uniform.
v = RZ.

www.examrace.com
Q.16 A 40.0 kg boy is standing on a plank of mass Q.19 A metal wire of length L1 and area of cross
160 kg. The plank originally at rest, is free to section A is attached to a rigid support.
slide on a smooth frozen lake. The boy walks Another metal wire of length L2 and of the
along the plank at a constant speed of 1.5 m/s same cross sectional area is attached to the free
relative to the plank. The speed of the boy end of the first wire. A body of mass M is then
relative to the ice surface is - suspended from the free end of the second
(A) + 1.8 m/s (B) – 1.2 m/s wire. If Y1 and Y2 are the Yong's moduli of the
(C) + 1.2 m/s (D) + 1.5 m/s wires respectively, the effective force constant
Sol. [C]
of the system of two wires is -
The system is not subjected to any external force
(A) [(Y1 Y2)A] / [2(Y1L2 + Y2L1)]
and hence conservation of momentum can be
used. Let mb and mp represent the masses of the (B) [(Y1 Y2)A] / (L1 L2)]1/2
boy and the plank. Let vbi, vpi and vbp be the (C) [(Y1 Y2)A] / (Y1L2 + Y2L1)
velocity of the boy with respect to ice, that of the (D) (Y1 Y2)1/2 A / (L1 L2)1/2
plank with respect to ice and that of the boy with Sol. [C]
respect to the plank respectively. Then, mb vbi + Using the usual expression for the Young's
mp vpi = 0, also vbi = vbp + vpi. modulus, the force constant for the wire can be
written as k = F/'l = YA/L where the symbols
Q.17 When a soap bubble is given an electric charge, have their usual meanings. Now, the two wires
(A) it contracts together will have an effective force constant
(B) it expands (k1k2/(k1 + k2)]. Substituting the corresponding
(C) its size remains the same lengths and the Young's moduli we get the
(D) it expands or contracts depending upon answer.
whether the charge is positive or negative
Sol. [B] Q.20 Four moles of carbon monoxide are mixed
For an unelectrified soap bubble force due to with four moles of carbon dioxide. Assuming
excess pressure from inside is balanced by the the gases to be ideal, the ratio of specific heats
force due to surface tension. When it is given an
is
electric charge, there is an outward normal force
(A) 15/11 (B) 41/30 (C) 4/3 (D) 7/4
(V2/2H0 per unit area, where V is the surface
Sol. [A]
charge density) that expands the bubble.
Use the expression for the ratio of specific heats
Q.18 A wooden block floats in a liquid with 40% of n1C pl  n 2 C p 2
of a mixture : J = . Also note
its volume inside the liquid. When the vessel n1C v l  n 2 C v 2
containing the liquid starts rising upwards with that Cp and Cv for diatomic gas are 5R/2 and
acceleration a = g/2, the percentage of volume 7R/2 respectively, whereas those for a
inside the liquid is polyatomic gas are 3R and 4R.
(A) 20% (B) 60% (C) 30 % (D) 40%
Sol. [D]
Q.21 The equations of two sound waves propagating
When the vessel is stationary, the weight of the
wooden block is balanced by the upthrust, that is in a medium are given by y1 = 2 sin (200St)
VUwood g = VliqUliqg, where V is the volume of the and y2 = 5 sin (150St). The ratio of intensities
block and Vliq is the volume of the liquid of sound produced is -
displaced, Uwood and Uliq are the densities of wood (A) 4 : 25 (B) 9 : 100
and liquid respectively. This gives (Vliq/V) =
(C) 8 : 15 (D) 64 : 225
(Uwood/Uliq). When the vessel moves up, the net
upward force is (upthrust – weight). The upthrust Sol. [D]
is [(Vliq)' Uliqg] where (Vliq)' is the volume of the The intensity of a wave is proportional to square
liquid displaced in this case. The net upward force of amplitude as well as square of frequency. The
is [V Uwoodg/2]. This gives [(Vliq)'/V] = (Uwood/Uliq). amplitudes are in the ratio 2 : 5 whereas the
From this we see that the same volume of the frequencies are in the ratio 4 : 3.
wooden block remains inside the liquid.

www.examrace.com
Q.22 String A has a length l, radius of cross section Q.26 An ideal gas confined to an insulated chamber
r, density of material Uand is under tension T. is allowed to enter into an evacuated insulated
String B has all these quantities double those of chamber. If Q, W and 'Eint have the usual
string A. If fA and fB are the corresponding meanings, then
fundamental frequencies of the vibrating
(A) Q = 0, W z 0 (B) W = 0, Q z 0
strings, then
(C) 'Eint = 0, Q z 0 (D) Q = W = 'Eint = 0
(A) fA = 2 fB (B) fA = 4 fB
(C) fB = 4 fA (D) fA = fB Sol. [D]
This is a case of free expansion of a gas. Note
Sol. [B] that due to insulation, Q = 0. Since the gas
Note that the frequency of the vibrating string expands against no counteracting pressure (that
1 T is in vacuum), W = 0. This gives no change in
fv where the symbols have their usual the internal energy.
l Au U
meanings.
Q.27 Read the two statements – (1) When a solid
Q.23 The temperature of n moles of an ideal gas is melts an changes to liquid state, its volume
increased from T to 4T through a process for may increase or decrease. (II) As a result of
which pressure p = aT 1 where a is a constant. increase in pressure, the melting point of a
Then, the work done by the gas is solid may be raised or lowered.
(A) nRT (B) 4 nRT With reference to these statements, the only
(C) 2 nRT (D) 6 nRT correct statements out of the following is
Sol. [D] (A) (I) is true but (II) cannot be true
Use the relation pV = nRT with p = a/T (given). (B) (I) cannot be true but (II) is true
This gives V = nRT2 / a, so that dV = (2nRT / a)dT.
(C) (I) and (II) both are true and (I) is the
Now integrate pdV between T and 4T to get the
result. cause of (II)
(D) (I) and (II) both are true and they are
Q.24 For a monatomic ideal gas undergoing an independent of each other
adiabatic change, the relation between Sol. [C]
temperature and volume is TVx = constant An increase in pressure tends to compress the
where x is - substance. On melting if volume of a substance
(A) 7/5 (B) 2/5 decreases, an increase in pressure will help the
(C) 2/3 (D) 1/3 process of melting, so that melting point will be
Sol. [C] lower. On the other hand if volume of a
For an adiabatic change in case of a monatomic substance increases on melting, then an increase
gas, TVJ–1 = constant. In this case x itself is (J –1) in pressure will oppose the process of melting.
and J = 5/3 giving the value of x. hence, melting point will increase. Thus, the two
statements are true and (I) is the cause of (II).
Q.25 A system is taken from a given initial state to a
given final state along various paths
Q.28 A magnetic field directed along Z axis varies
represented on a p-V diagram. The quantity
that is independent of the path is as B = B0x/a, where a is a positive constant. A
(A) amount of heat transferred Q square loop of side l and made of copper is
(B) amount of work done W placed with its edges parallel to X and Y axes.
(C) Q but not W if the loop is made to move with a constant
(D) (Q – W) velocity v0 directed along X axis, the emf
Sol. [D] induced is
The only quantity (Q – W) which itself is the (A) (B0v0l2)/a (B) (B0v0l)
internal energy of the system is independent of
(C) (B0v0l2)/2a (D) (B0v0l3)/a2
the path.

www.examrace.com
Sol. [A] Q.32 A sounding body emitting a frequency of 200
Note that change in flux in one second is equal to Hz is released from a height. One second after
the emf induced. Use dI = B.dA. Since its release, it crosses a balloon rising with a
B || dA. dI = BdA with dA = l dx. Integrae dI constant velocity of 2m/s. Let the speed of
between 0 and l to obtain the initial flux which
sound be 300 m/s and acceleration due to
comes out to be (B0l3/2a). In one second the lop
gravity 10 m/s2. The change in frequency
covers a distance of v0, so that the final flux can
be found by integrating – dI between v0 and noted by an observer in the balloon at the
(v0 + l). This flux comes out to be [(B0v0l2/a) + moment of crossing is -
(B0l3/2a)] and hence the change in flux per second (A) 10 Hz (B) 8 Hz (C) 16 Hz (D) 4 Hz
can be determined. Sol. [C]
Use the relation : the apparent frequency
Q.29 An object is placed in front of a spherical ª v  v0 º
mirror of focal length f. If x and x c respectively n2 = n « » where v is the speed of sound,
¬ v – vs ¼
represent the distances of the object and the
image from the focus, then - v0 is the speed of the observer and vs is the speed
of the source of sound. The speed v0 is to be
(A) f = x + x c
considered positive or negative depending on
(B) f2 = x xc
whether the observer is moving towards or away
(C) f = | x – x c|
from the source. Similarly, vs is to be considered
(D) f = x ± x cdepending upon whether image
positive or negative depending on whether the
is real or virtual
source is moving towards or away from the
Sol. [B]
observer. Using this convention, determine the
Note that the object distance can be written as (f + x)
whereas the image distance can be written as apparent frequencies before and after the
(f + x2). Use the mirror formula to get the answer. crossing and then the difference between them.

Q.30 Different objects at different distance are seen Q.33 Refer to the figure. The number reflections
by the eye. The parameter that remains constant is - from mirrors M1 and M2 are –
(A) the focal length of the eye lens M1 M2
(B) the object distance from the eye lens 1m
(C) the radii of curvature of the eye lens
(D) the image distance from the eye lens
Sol. [D] 1m
The image formed by the eye lens is always one
the retina and the image distance is fixed.

Q.31 A body in the form of a right circular cone of


dielectric material with base radius R and 5°
height h is placed with its base on a horizontal (A) 5 and 5 (B) 6 and 5
table. A horizontal uniform electric field of
(C) 10 and 10 (D) 6 and 6
magnitude E penetrates the cone. The electric
Sol. [B]
flux that enters the body is
Consider the point at which the incident ray
(A) ERh/3 (B) ERh (C) ERh/6 (D) 2ERh
strikes M1 for the first time. let its distance from
Sol. [B]
the lower end of the mirror be x, so that tan 5° =
Note that the flux through an area is (E ˜dA).
Here the flux through the cone is the same as that (x/1) giving x = 0.087 (approximately). If n is the
through the triangular section of the cone by a number of spacings accommodated in the mirror
vertical plane passing through the vertex. The length of 1 meter, then nx = 1, giving n = 11.49.
area of this triangular section is [1/2(2R X h)] and Thus, n is greater than 11 but less than 12. This
is perpendicular to the direction of the field E. gives 6 reflections from M1 but 5 from M2.

www.examrace.com
Q.34 A ray of light is incident at angle D on the Q.37 A conductor is bent in the form of concentric
boundary separating two transparent media. It semicircles as shown in the figure. The
is transmitted. If the angle of incidence is magnetic field at the point O is :
increased very slightly, the ray gets reflected in
the same medium. The difference between
angles of deviation in the two cases will be i
close to -
(A) 2 D (B) 90° – D
(C) 180° – D (D) 180° – 2D upto f
Sol. [B] 0 a 4a
When the ray is incident at the critical angle D, 2a 8a
the angle of deviation is (90° – D) whereas just (A) zero (B) μ0i /6a
after this (that is at an angle slightly greater than (C) μ0i/a (D) μ0i / 4a
the critical angle) the angle of deviation is (180° –
Sol. [B]
2D) Magnetic field at the centre of a semicircular
current carrying conductor is given by the
Q.35 Two particles A and B having equal charged μ i(Sa ) μ 0i
after being accelerated through the same expression B = 0 where a is the
potential difference, enter a region of uniform 4S a 2 4a
magnetic field and describe circular paths of radius of the first semicircle. Note that the
radii R1 and R2 respectively. The ratio of mass current in all the turns is the same but its sense is
alternately opposite and the radii are in the
of A to that of B is
proportion 1 : 2 : 4 : 8…… Then, the net
(A) (R1/R2)2 (B) (R1/R2)1/2
(C) (R2/R1) 2
(D) (R2/R1) μ iª 1 1 1 1 º
magnetic field = 0 «1 –  –  .....» .
Sol. [A] 4a ¬ 2 4 8 16 ¼
The work done by the electric field on both the The terms in the bracket form a geometric
particles is the same and hence the gain in kinetic progression which adds to (2/3) and then the
energy for both is the same. This gives answer follows.
v2 m1
. Note that the centripetal force Q.38 A capacitor and a coil in series are connected
v1 m2
to a 6 volt ac source. By varying the frequency
necessary for the circular motion in the magnetic of the source, maximum current of 600 mA is
field is qvB. Using this for both the particles, we observed. If the same coil is now connected to
mv m2v2 a cell of emf 6 volt and internal resistance of
get 1 1 . From these two relations, we
R1 R2 2 ohm, the current through it will be
get the answer. (A) 0.5 A (B) 0.6 A
(C) 1.0 A (D) 2.0 A
Q.36 A Sound wave traveling through a medium of Sol. [A]
bulk modulus B is represented as y(x, t) = A sin The maximum current is obtained at resonance
(kx – Zt) where symbols have their usual where the net impedance is only resistive which
meanings. Then, the corresponding pressure is the resistance of the coil only. This gives the
amplitude is resistance of the coil as 10 ohm. Now, this coil
(A) BAk (B) B (A/K)1/2 along with the internal resistance of the cell gives
a current of 0.5 A.
(C) B (D) B(AK)1/2
Sol. [A] Q.39 Two radioactive substances X and Y emit D
wy and E particles respectively. Their disintegration
From the equation of the sound wave, we get
wx constants are in the ratio 2 : 3. To have equal
= kAcos (kx –Yt). Now use the expression 'p = – probabilities of getting emission of D and E
wy
B where B is the bulk modulus. This gives an particles, the ratio of number of atoms of X to
wx
that of Y at any time instant is :
expression for the pressure change 'p = – BAk
sin [(kx – Zt) – S/2], also indicating a phase lag
(A) 2 : 3 (B) 3 : 2
of S/2 with respect to displacement. (C) e : 1 (D) (e – 1) : 1
Sol. [B]

www.examrace.com
Note that (OX/OY) = 2/3. To have equal Sol. [A,B ,C]
probabilities the rates of decay must be equal, that Use the lens maker formula :
is, OXNX = OYNY at any instant. The gives the 1 § μ1 ·§ 1 1 ·
¨¨ – 1¸¸¨¨ – ¸¸ where μL
ratio (NX/NY) 3/2. f © μm ¹© R 1 R 2 ¹
and μm represent the refractive indices of the
Q.40 In a certain particle accelerator, electrons liquid filled and the surrounding medium
emerge in pulses at the rate of 250 pulses per respectively. Note that for a double concave lens
second. Each pulse is of duration of 200 ns and ª 1 1 º
the electrons in the pulse constitute a current of « – » is negative. Now, for the lens to act
250 mA. The number of electrons delivered by ¬ R1 R 2 ¼
as a diverging lens, that is, for f to be negative,
the accelerator per pulse is :
the first bracket on the right hand side of the
(A) 8.00 × 1010 (B) 5.00 × 108 relation must be positive. This requires μL to be
(C) 3.13 × 1011 (D) 9.60 × 1010 greater than μm.
Sol. [C]
The charge in a pulse dq = I dt = 5.00 X 10–8C. Q.43 Referring to the circuit diagram, the tap key is
Divide this by 1.6 X 10–19 to determine the pressed at time t = 0. After sufficiently long
number of electrons per pulse. time
R C
SUB PART – A-2
VR VC
Q.41 Let v(t) be the velocity of a particle at time t.
Then, K V
(A) |dv(t)/dt| and d| v(t)|/dt are always equal
(A) VC = 0 (B) VR = 0
(B) |dv(t)/dt| and d| v(t)|/dt may be equal (C) VC = V (D) VR = V
(C) d|v(t)|/dt| can be zero while |dv(t)|/dt| is not Sol. [B, C]
zero After sufficiently long time the charging current
(D) d| v(t)|/dt z 0 implies |dv(t)/dt| z 0 reduces to zero. Then all the voltage appears
Sol. [B, C, D] across the capacitor and no drop across the
Note that |dv(t)/dt| is the magnitude of resistor.
acceleration, while d|v(t)|/dt is the time rate of
change of speed. These two may not be always Q.44 As shown in the figure, a front coated mirror
equal, hence (a) is not correct. In fact, the two are M produces an image S2 of a source S1 of
equal when the motion is along a straight line. In monochromatic light. Then,
case of uniform circular motion, speed remains
S1 P
constant but not the velocity. Again if speed is not
constant the velocity cannot be constant.
M
S2 Screen
Q.42 A hollow double concave lens is made of a
very thin transparent material. It can be filled (A) Point P will be a point of maximum
with water (refractive index μw) or either of intensity if the path difference ' = (2n)O/2
two liquids L1 or L2 with refractive indices μ1 for n = 0, 1, 2, 3….
and μ2 respectively (μ2 > μw > μ1). The lens (B) Point P will be a point of maximum
will diverge a parallel beam of light incident on intensity if the path difference ' = (2n + 1)
it, if it is filed with - O/2 for n = 0, 1, 2, 3…..
(A) L2 and immerses in L1 (C) Point P will be a point of minimum
(B) L2 and immersed in water intensity if the path difference ' = (2n)O/2
(C) water and immersed in L1 for n = 0, 1, 2, 3….
(D) air and immersed in either water or L1 or L2 (D) there is no such condition for the path
difference as there is no interference

www.examrace.com
Sol. [B, C] Q.47 A concave lens is placed in the path of a
Note that there is an additional phase change of S uniform parallel beam of light falling on a
red or a path difference of (O/2) when a ray gets screen as shown. Then,
reflected from the mirror.
C
Q.45 Two point charges (Q each) are placed at (0, y) A
and (0, –y). A point charge q of the same O
polarity can move along X axis. Then, B
(A) the force on q is maximum at x = ± y / 2 D
Screen
(B) the charge q is in equilibrium at the origin
(A) intensity of light will be the same every
(C) the charge q performs an oscillatory
motion about the origin where on the screen
(D) The equilibrium is not a stable equilibrium (B) intensity in region AB will be smaller than
Sol. [A, B, D] what is would be in the absence of the lens
At a point with coordinates (x, 0) the force is (C) in the region AC and BD, the intensity will
2Qq x be smaller than what it would be in the
F= . For F to be maximum,
4SH0 ( x  y 2 ) 3 / 2
2
absence of the lens
dF (D) in the region AC and BD, the intensity will
equating to zero gives x = ± y/ "2. The
dx be greater than what is would be in the
charge is obviously in equilibrium at the origin. absence of the lens
However, the equilibrium is not stable since the Sol. [B, D]
force is repulsive and hence will not be able to Note that the intensity of light in the region AB
restore the charge at the origin. The charge
(when the lens is absent) now gets distributed
therefore cannot perform oscillatory motion.
over the region CD. In the regions AC and BD
Q.46 The network of identical resistors as shown light intensity is due to both the direct beam and
between points A and B is connected to a dc the diverged light from the lens.
source of emf V. Then,
C F Q.48 A hydrogen atom and a doubly ionized lithium
B
atom are both in the second excited state. If LH
and LLi respectively represent their electronic
A angular momenta and EH and ELi their
D E
energies, then -
(A) potential at point D is V/2
(B) current between points A and C is the same (A) LH > LLi and | EH | > | ELi |
as that between F and B (B) LH = LLi and | EH | < | ELi |
(C) current between C and D is half that (C) LH = LLi and | EH | > | ELi |
between points C and F (D) LH < LLi and | EH | < | ELi |
(D) current between points E and F is one third Sol. [B]
that between F and B For both the atoms the second excited state
Sol. [A, B, C, D] corresponds to n = 3. Therefore, the angular
Note that points D and E are coincident and the
momentum for each of them is 3(h/2S). The
circuit is symmetric about this point. Consider a
current i entering the circuit at point A and energy, however, is proportional to Z2 where Z is
leaving it at point B. If current i1 flows between A the atomic number and hence numerical value of
and D, a current (i – i1) flows between A and C. energy for hydrogen is less than that for lithium.
By symmetry current between D and B is i1 and
that between F and B is (i – i1). The current (i – Q.49 Refer to the circuit diagram and the
i1) gets branched at C and (1/3) (i – i1) flows
between C and D and continues upto F. This is corresponding graph. The current rises when
because there is no branching of current at D. key K is pressed. With R = R1 and L = L1 the
Obviously (2/3) (i – i1) flows between C and F. rise of current is shown by curve (1), while
Note that total resistance between C and F is half curve (2) shows the rise of current when R =
of that along the path ADF.
R2 and L = L2. Then,

www.examrace.com
i
L R
Sol. Let Z be the angular velocity of the cube
i0 (just after the bullet strikes) about an axis passing
(1) through D. Conservation of angular momentum
K V
(2) about this axis gives mv(2l/3) = IZ where I is
O time t the moment of inertia about the axis through D.
(A) R1 = R2 (B) R1 > R2 This is [(M l2/6) + (M l2 /2)] = (2M l2/3) From
(C) L1 < L2 (D) L1> L2 the above equation Z = (mv/M l). The cube will
Sol. [A, C] topple if the centre of mass is just able to rise
Consider the expression for the current rising from (l/2) to (l/"2). In such a case, the rotational
exponentially in the LR circuit. The time constant energy must be equated to the change of
is (L/R). In this case the curve (1) is rising faster § l
1 2 l·
than curve (2) indicating that (L1/R1) < (L2/R2). potential energy. Thus, IY Mg¨¨ – ¸¸ .
However, in both the cases the maximum current 2 © 2 2¹
is the same and equal to (V/R1) or (V/R2). Using the values of I and Z, we get the
Q.50 Let the energy, magnitude of linear momentum expression for v that will just topple the cube :
and angular frequency of an electron in 1/ 2
M ª §¨ 2 – 1 ·¸º
hydrogen atom be E, p and Z respectively. If n v «3gl » .
be the corresponding quantum number, then - m «¬ ¨© 2 ¸¹»¼
(A) (E/Z) varies as n
(B) (E p) / Z is independent of n Q.52 Consider flow of heat from an inner sphere of
(C) (p Z) varies as n1/2 radius a at temperature T2 to an outer
(D) (E p Z) is independent of n
Sol. [A, B]
concentric sphere of radius b at temperature T1
1 through an insulating material filled in
Note the proportionalities : energy E v 2 , between the two spheres. Obtain expressions
n
angular momentum L v n, radius of orbit for the total heat current H and the temperature
r v n2, periodic time T v n3, angular T at a distance r from the centre, when a
1 steady state is reached.
frequency Y v 3 and speed of electron in
n Sol. Consider a concentric spherical shell of radius r
1 and thickness dr where a < r < b. Let k be the
an orbit v v and get the result.
n § dT ·
coefficient of thermal conductivity and ¨ ¸ be
© dr ¹
PART B Marks : 60 the temperature gradient at this shell. Then, the
* All questions are compulsory. heat current (that is the amount of heat flowing
* All questions carry equal marks per unit time) is given by
H = – 4Sr2k ()
Q.51 A solid wooden cube of side l and mass M is
where the negative sign indicated a
resting on a horizontal table, as shown in the
figure. The cube is constrained to rotate about fall of temperature with increase of radius. From
an axis through D and perpendicular to the face this we get. Integrate this between limits a and b
ABCD. A bullet of mass m moving with speed for r and between T2 and T1 for T. This gives an
v strikes the block at a height 2l/3 as shown. expression for the heat current. Now, use to find
Let the line along which the bullet moves be in the indefinite integral with a constant of
the plane passing through the centre of mass of integration c. The constant of integration c comes
the block and parallel to the face ABCD.
out to be where temperature T2 corresponds to
Determine the minimum value of v that topples
the block. radius a. Using this we write an expression in
B C terms of temperature T at a radius r in general.
m Solving this we get an expression for the
2" v M temperature T at a distance r from the centre :
3 A D

www.examrace.com
Q.53 A double convex lens of radii of curvature 10 Sol. Refer to the figure in the question paper. Let the
cm (μ = 1.5) and a double concave lens of radii front lower edge of the wedge to be along X axis
of curvature 12 cm (μ = 1.6) are separated by a so that left extreme of the edge can be taken to
liquid of refractive index 1.2, as shown. Find be at x = 0 and the other extreme to the right to
the effective focal length of the combination. be at x = L. Choose an element of the wedge
perpendicular to this edge at a distance x and of
width dx. This section will have a height, so that
the area of cross section of this element will be
WX. Now, the resistance of this element can be
written as dR. Integrate this between x = 0 and x
4.8 cm
= L. The expression for the resistance of the
Sol. The convex lens with a plano-concave liquid lens
wedge comes out to be R.
on left can be taken to form one group of lenses.
Similarly, a plano-convex liquid lens with the
concave lens can be taken to form another group Q.55 Show that the capacitance of parallel plate
of lenses. The liquid layer of width 4.8 cm in capacitor filled with a dielectric whose
between will then be equivalent to an air slab of dielectric constant increases linearly from one
thickness (4.8 × 1.2) = 5.76 cm. Note that this plate to the other, is
distance (d) is the distance of separation between C = [H0A(K2 – K1)] / [d ln (K2/K1)]
the two lens combinations described above. The
where A is the area of each of the plates, d is
focal length of the given convex lens can be
determined using lens maker's formula and it the separation between them, K1 and K2 are he
comes out to be 10.0 cm. Similarly the focal dielectric constants near plate 2 respectively.
length of the adjacent plano-concave liquid lens
turns out to be (– 50.0) cm. The focal length of Sol. Let the relation for variation of dielectric
this combination 12.5 cm. Use the relation. in this constant be K = mx + K1 where x is the distance
case the distance of separation d is zero.
from plate l. At x = 0, that is, at plate l, K = K1
Similarly, the focal lengths of the plano-convex
liquid lens and the concave lens on right come out while at x = d, K = K2. With this we get m = (K2
to be 60.0 cm and (– 10.0) cm respectively. The – K1)/d. Now, we use E =. The potential
focal of their combination turns out to be (– 12.0) difference V between the plates of the capacitor
cm. Now, the two lens combinations are of focal can be determined by writing
lengths 12.5 cm and (– 12.0) cm separated by a V = + "dV where dV = E dx. Note that E is a
distance of 5.76 cm. The relation written above
function of x. Limits of integration for x are 0
can again be used to get the effective focal length
as 28.49 cm and acts as a converging lens. and d. Then use C = Q/V where Q = VA. The
final relation for the capacitance is C.
Q.54 A material with uniform resistivity U is formed
in the shape of a wedge as shown. Determine
the resistance between face A and face B of
this wedge.

face B
face A
y2
y1
W
L

www.examrace.com
INDIAN ASSOCIATION OF PHYSICS TEACHERS
NATIONAL STANDARD EXAMINATION IN PHYSICS 2009-2010
Total time : 120 minutes (A-1, A-2 & B)
PART - A (Total Marks : 180)
SUB-PART A-1

Q.1 The Schrodinger equation for a free electron of Sol. [B]


mass m and energy E written in terms of the If l is the length of the hanging part of the rope,
d 2  8 2 mE M
wavefunction  is  = 0. The its weight is   l × g and hence the weight of
dx 2 h2 L
dimensions of the coefficient of  in the second M
term must be - the rope on the table is   (L – l)g which is
L
(A) [M1 L1] (B) [L2]
2 also equal to the normal reaction N. Then, the
(C) [L ] (D) [M1 L 1T1]
M
Sol. [C] force of friction is μN = μ   (L – l) g.
By dimensional analysis the dimensions of each L
term in an equation must be the same. In the first Equating this to the weight of the hanging part of
term the second derivative with respect to the rope, we get the answer.
distance x indicates the dimensions of the
Q.4 A thin uniform rod XY of length 2l is hinged
coefficient of  to be [L–2] and hence the answer.
at one end X to the floor and stands vertical.
When allowed to fall, the angular speed with
Q.2 In an experiment to verify poiseullie's law which the rod strikes the floor is -
 pa 4  3g 3g
  where the symbols have their (A) (B)

8lV  4l 2l
usual meanings, the maximum error that enters 3g g
(C) (D)
in calculating the coefficient of  is due to l 2l
measurement of -
(A) pressure difference p Sol. [B]
(B) length of the tube l The moment of inertia of the rod about a
(C) volume rate of flow V horizontal axis passing through point X is
(D) radius of the tube a. 1 4 2
Ix = m(2l)2 + ml2 = ml where m is the
Sol. [D] 12 3
Since a is raised to the power 4, it has the largest mass of the rod. Now, since centre of gravity
contribution to the total error. falls through a height l, the change in
gravitational potential energy (mgl) of the rod
can be equated to the rotational energy
Q.3 A uniform rope of length L and mass M partly
1 2
lies on a horizontal table and partly hangs from  I x   where  is the angular speed.
the edge of the table. If μ is the coefficient of 2 
friction between the rope and the surface of the
table (neglecting the friction at the edge), the Q.5 A 60 cm metal rod M1 is joined to another 100
maximum fraction of the length of the rope that cm metal rod M2 to form an L shaped single
overhangs from the edge of the table without piece. This piece is hung on a peg at the joint.
sliding down is : The two rods are observed to be equally
1–μ μ inclined to the vertical. If the two rods are
(A) (B) equally thick, the ratio of density of M1 to that
μ μ 1
of M2 is
1
(C) 1 – μ (D) (A) 5/3 (B) 3/5
μ 1 (C) 25/9 (D) 9/25

www.examrace.com
Sol. [C] Sol. [B]
The angle made by each rod with the vertical is The change in potential energy can be equated to
45°. Applying the law of moments, we get the the gain of total kinetic energy :
ratio of weights and hence that of masses to be Gmm  Gmm  1 2
– – –  2 mv 
5 r  2R  2 
. Then, using mass = volume ×density, we get
3
the ratio of densities as 25 : 9. Q.8 A nonviscous liquid of density  is filled in a
tube with A as the area of cross section, as
Q.6 The string of a simple pendulum is replaced by shown in the figure. If the liquid is slightly
a thin uniform rod of mass M and length l. The depressed in one of the arms, the liquid
mass of the bob is m. If it is allowed to oscillate column oscillates with a frequency
with small amplitude, the period of oscillation
is
2(M  3m)L 3(M  2m)L 1
(A) 2 (B) 2 2
3(M  2m)g 2(M  3m)g

( M  2m ) L 2(M  m)L    2 
(C) 2 (D) 2 gA sin  1 
(M  3m)g 3(M  2m)g 1  2 
(A)
Sol. [A] 2 m
The restoring torque can be written as 1 gA (sin 1 – sin  2 )
(B)
L  2 m
– Mg  sin   mgL sin  which reduces to

2  1 gA (sin 1 – sin  2 )
(C)
L 2 m
– (M + 2m)g    if  is small. This is equated
2   – 2 
gA sin  1 
to I where the moment of inertia 1  2 
(D)
1 L2 2 m
I = ML2 + mL2 = (M + 3m). After
3 3 Sol. [C]
The force that is responsible for restoring the
3(M  2m)g 
substitution, we get  = –  . liquid levels in the two arms of the tube is

2(M  3m)L  – pA = – (h1 + h2) gA where p is the pressure
Comparing with a = – (2), expression for difference and A is the area of cross section of
angular frequency and hence period can be the tube, h1 and h2 being the rise and fall of
liquid levels in the two arms in vertical direction
obtained.
respectively. Note that the change in length of
the liquid thread along the tube will be the same,
Q.7 Two stars each of mass m and radius R say x. Using this the force can further be written
approach each other to collide head-on. as {– (x sin 1 + x sin 2)gA}.
Initially the stars are at a distance r (>> R). Writing force as mass times acceleration, we get
expression for period and then for frequency.
Assuming their speeds to be negligible at this
distance of separation, the speed with which the Q.9 A wax candle floats vertically in a liquid of
stars collide is density twice that of wax. The candle burns at
the rate of 4 cm/hr. Then, with respect to the
 1 1  1 1
(A) Gm –  (B) Gm –  surface of the liquid the upper end of the
R r  2R r  candle will
(A) fall at the rate of 4 cm/hr
 1 1  1 1 (B) fall at the rate of 2 cm/hr
(C) Gm   (D) Gm  
R r  2R r  (C) rise at the rate of 2 cm/hr
(D) remain at the same height

www.examrace.com
Sol. [B] Q.12 A metal ball B1 (density 3.2 g / cc) is dropped
Let the decrease in the height of the candle in water, while another metal ball B2 (density
outside the liquid be x. Now, if d denotes the 6.0 g / cc) is dropped in a liquid of density 1.6
diameter and L the length of the candle g / cc. If both the balls have the same diameter
respectively, after one hour applying thelaw of and attain the same terminal velocity, the ratio
floatation gives of viscosity of water to that of the liquid is :
2 2
d  d  L  (A) 2.0
  ( L – 4) wax    – x liq . Note (B) 0.5
 
2   
2 2 
(C) 4.0
1
that wax = liq makes only half of the candle of (D) indeterminate due to insufficient data
2 Sol. [B]
stand outside the liquid. Solving the equation one Use the expression for terminal velocity of a body
gets the answer. 2 r 2 ( – )g
falling in a viscous liquid v =
Q.10 A mass hangs at the end of a massless spring 9 
and oscillates up and down at its natural where symbols have their usual meanings.
frequency f. If the spring is cut at the midpoint
and the mass reattached at the end, the Q.13 A thin copper rod rotates about an axis passing
frequency of oscillation is : through its end and perpendicular to its length
with an angular speed 0. The temperature of
(A) 2f (B) 2f (C) f/2 (D) f/ 2
the copper rod is increased by 100°C. If the
Sol. [A]
If k is the spring constant for the original spring
coefficient of linear expansion of copper is
2 × 10 5 / °C, the percentage change in the
and k  that for the half-cut spring, we have
angular speed of the rod is
k  = 2k. Use the expression for frequency
(A) – 2% (B) – 4%
1 k (C) – 0.2 % (D) – 0.4 %
f= to get the answer.
2 m Sol. [D]
ml 2
The moment of inertial of the rod is . Let
Q.11 The density of a solid at normal pressure is . 3
When the solid is subjected to an excess I0, 0 denote the initial moment of inertia and
pressure . The density changed to . If the initial angular velocity respectively and I,  be
 the corresponding quantities after heating the rod.
bulk modulus of the solid is k, then the ratio
 Applying conservation of angular momentum, we
is mI 02 
0 
p k I 
(A) 1 + (B) 1 + get  0 0
3 
k p I ml0 (1  ) 2 
2

p k 
(C) (D)
3 
pk pk
= 0 (1 – 2) = 0.996 0.
Sol. [A]
Thus the change is 0.4% and is negative.
m
We use expression for density  = so as to get Q.14 The internal energy of one gram of helium at
V
100 K and one atmospheric pressure is
d dV
– . Substituting this in the expression for (A) 100 J (B) 1200 J (C) 300 J (D) 500 J
 V Sol. [C]
p The helium molecule is monatomic and hence
bulk modulus, we get k = – . Since 3
 dV  internal energy per molecule is kT . The
  2
 V 
3
increase of pressure increases the density, energy per mole is therefore is RT . One gram
 –  p 2
d =  – . Using this we get and of helium is one fourth mole and hence its
 k 1 3
hence the answer energy is  R × 100 = 300 J, taking the
4 2
value of R to be approximately 8 J/mole-K.

www.examrace.com
Q.15 Volume of a monatomic gas varies with its Sol. [B]
temperature as shown. The ratio of work done The speed with which a sphere rolling down an
inclined plane reaches the bottom is
by the gas to the heat absorbed by it when it 1/ 2

undergoes a process from A to B is 2gh  1/ 2
 10 
V v   gh 
1  I  7 

mr 2 
B where symbols have their usual meanings. The
5 
A acceleration is  g sin   . With this, the time
7 
O taken by the glass bead to roll down turns out to
T
14h 1
be . Similarly the time to go up the
(A) 2/3 (B) 2/5 (C) 2/7 (D) 1/3 5g sin 1
Sol. [B]
14h 1
Since the curve passes through the origin, other inclined plane is . Twice the
5g sin  2
V  T, the pressure being constant. Then, heat
sum of these two times is the required time of
absorbed at constant pressure is dQ = n Cp dT oscillation.
whereas the change in internal energy is
dU = n Cv dT where the symbols have their usual Q.17 Water at 20°C (coefficient of viscosity = 0.01
poise) flowing in a tube of diameter 1 cm with
meanings. The work done dW = dQ – dU = n (Cp –
an average velocity of 10 cm/s has the
Cv)dT. This gives the ratio of work done to the Reynold number
 1 (A) 500
heat absorbed as 1 –  and hence the result. (B) 1000
  (C) 2000
(D) indeterminate due to insufficient data
Q.16 A small glass bead of mass m initially at rest Sol. [B]
vD
starts from a point at height h above the Reynold number is given by N = where 

horizontal and rolls down the inclined plane
is the density, v is the velocity and D is the
AB as shown. Then it rises along the inclined diameter of the tube. Use this to get the answer.
plane BC. Assuming no loss of energy, the time
period of oscillation of the glass bead is Q.18 A billiard ball is hit by a cue at a point
distance h above the centre. It acquires a linear
C
A velocity v0. Let m be the mass and r be the
radius of the ball. The angular velocity
h acquired by the ball is
1 2
5v 0 h 2v 0 h 2v 0 h 2v 0 h
B (A) 2
(B) (C) (D)
2r 3r 2
5r 2
r2
8h Sol. [A]
(A) (sin 1 + sin 2) Equating the angular momentum (mv0h) about
g
the center of mass to l one gets the answer.
14h  1 1 
(B) 2   
5g  sin 1 sin  2  Q.19 Two pipes are each 50 cm in length. One of
them is closed at one end while the other is
8h  1 1  open at both ends. The speed of sound in air is
(C)   
g  sin 1 sin  2 
340 m /s. The frequency at which both the
pipes can resonate is
(A) 680 Hz (B) 510 Hz
8h  1 1 
(D)    (C) 85 Hz (D) None of these
5g  sin 1 sin  2 

www.examrace.com
Sol. [D] Q.22 A convex lens forms a real image with
A pipe open only at one end and another one of magnification m1 on a screen. Now, the screen
equal length but open at both the ends have their is moved by a distance x and the object is also
moved so as to obtain a real image with
fundamental frequencies n and 2n respectively.
magnification m2 on the screen. The, the focal
Again only odd harmonics are possible for a pipe length of the lens is
open at one end whereas all harmonics are m  m 
possible for a pipe open at both ends. Thus, it is (A)  1  x (B)  2  x
 m2   m1 
impossible to have a common frequency at which
x
they can resonate. (C) x (m1 – m2) (D)
(m 2 – m1 )
Sol. [D]
Q.20 The work involved in breaking a bigger size
Use the usual lens formula. In the first case if v is
spherical oil drop into n smaller size identical the image distance, (v + x) is the image distance
droplets is proportional to after the movement. The magnifications m1 and m2
3
n2 –1 3 v 
(A) (B) n in the two cases turn out to be  – 1 and
f 
(C) 3 n – 1 (D) 3 n – 1
vx 
Sol. [D]  – 1 respectively. This can be simplified
 f 
Note that the work involved in the process of
to get the expression for f.
breaking a bigger drop into smaller drops is the
change in surface area times the surface tension.
Q.23 Two metal wires of identical dimensions are
If r is the radius of smaller drop and R that of the
connected in series. If 1 and 2 are the
4 3 4 
bigger one, then R n  r 3  where n is conductivities of the metals respectively, the
3 3  effective conductivity of the combination is
the number of smaller drops. This gives   2
(A) 1 + 2 (B) 1
R 2
r= . If T is the surface tension, then the work
3
n 21 2
(C) 1 2 (D)
done will be W = (n4r2 – 4R2) T. Substituting 1   2
for r gives the expected proportionality. Sol. [D]
Use the expression for resistance in terms of
Q.21 A train moving towards a hill at a speed of 72 l
conductivity R = and note that the total
km/hr sounds a whistle of frequency 500 Hz. A A
wind is blowing from the hill at a speed of 36 resistance is (R1 + R2) and total length of the
wires is 2l.
km/hr. If the speed of sound in air is 340 m/s,
the frequency heard by a man on the hill is
Q.24 An alternating supply of 220 volt is applied
(A) 532.5 Hz (B) 565.0 Hz
across a circuit with resistance 22 ohm and
(C) 516.5 Hz (D) None of the above
impedance of 44 ohm. The power dissipated in
Sol. [A]
the circuit is
If n  represents the apparent frequency and n the (A) 1100 watt. (B) 550 watt.
actual one, then use the relation (C) 2200 watt. (D) (2200/3) watt
(v ! w )  Sol. [B]
n  –n  where v is the velocity of

(v ! w ) – vs  The current in the circuit is
V
= 5 A and hence
sound, w is the velocity of wind and vs that of the Z
source. Note that in this case the observer at rest. the power consumed is (I2R) equal to 550 watt.

www.examrace.com
Q.25 If T denotes the temperature of the gas, the Q.28 Two identical thin rings, each of radius a are
volume thermal coefficient of expansion of an placed coaxially at a distance a apart. Let
ideal gas at constant pressure is
charges Q1 and Q2 be placed uniformly on the
1 1
(A) T (B) (C) T2 (D) 2 two rings. The work done in moving a charge
T T
Sol. [B] q from the centre of one ring to that of the
The volume coefficient of expansion is given by other is
dV (A) zero
. For an ideal gas at constant pressure, we
vdT
q 2
write pdV = nRdT and substitute for p from the (B) (Q1 – Q2)
usual relation pV = nRT to get the required 4#0 a
expression for the coefficient.
q ( 2 – 1)
(C) (Q1 – Q2)
Q.26 A coil having N turns is wound in the form of a 4# 0 a 2
spiral with inner radius a and outer radius b
respectively. When a current I passes through q ( 2 – 1)
(D) (Q1 – Q2)
the coil, the magnetic field at the centre is : 4# 0 a
2μ 0 NI μ NI
(A) (B) 0 Sol. [C]
(a  b ) ab The electrostatic potential at the centre of the
μ 0 NI  b  μ 0 NI b first ring with charge Q1 is due to charge Q1 itself
(C) ln  (D) ln 
(b – a )  a  2( b – a )  a  as well as due to charge Q2 on the other ring. This
Sol. [D] 1 Q1 1 Q2
Consider dN to be the number of turns in between turns out to be  . Similarly
4#0 a 4# 0 2a
N
radii r and (r + dr) so that we get dN = dr. the electrostatic potential at the centre of the
(b – a )
The magnetic induction dB due to these many centre of the other ring is
μ I(dN) 1 Q1 1 Q2
turns at the centre is 0 . After substituting  . The difference between
2r 4# 0 2a 4#0 a
for dN and integrating between a and b, we get these potentials time the charge q is the required
the result.
word done.
Q.27 A circuit is arranged as shown. Then, the
current from A to B is : Q.29 An equilateral triangular loop of wire of side l
A carries a current i. The magnetic field
10 " 15 " produced at the circumcentre of the loop is
+ – μ 0 3 3i μ 0 9i
10 V – 10 " 5V
(A) (B)
+
4 l 4 l
μ 0 18i μ 0 6i
B (C) (D)
4 l 4 l
(A) + 500 mA (B) + 250 mA
(C) – 250 mA (D) – 500 mA Sol. [C]
Sol. [B] If l is the side of the triangle, the distance of the
Use superposition theorem. We get the potential circumcentre from each of the side of the triangle
difference between A and B to be + 3.75 volt μ0 i
when source of 5 volt is shorted, whereas – 1.25 carrying a current i is (sin 60° + sin 60°) =
4 r
volt when source of 10 volt is shorted. Therefore,
when both the sources are working the net μ 0 6i
. Since the direction of magnetic field in
potential difference is + 2.5 volt so that current is 4 l
250 mA from A to B. each case in the same, three times this would be
the total magnetic induction.

www.examrace.com
Q.30 Consider a double slit interference experiment. Q.33 Four identical mirrors are made to stand
Let E0 be the amplitude of the electric field of vertically to form a square arrangements as
shown in a top view. A ray starts from the
the waves starting from the slits. If  is the midpoint M of mirror AD and after two
phase difference between the two waves reflections reaches corner D. Then angle 
reaching the screen, the amplitude of resultant must be
B C
electric field at a point on the screen is
(A) E0 cos  (B) E0 cos (/2)
(C) 2 E0 cos (/2) (D) 2 E0 cos 
Sol. [C]
Consider the magnitudes of the electric fields  M
reaching the screen to be E0 sin t and E0 sin (t A D
1
+ ). Then, the resultant electric field at the (A) tan (0.75) (B) cot 1 (0.75)
1
screen would be sum of the two, that is, (C) sin (0.75) (D) cos 1 (0.75)
Sol. [B]
    
2E 0 cos  sin  t   . Note that the
Note that the ray starting from point M at

 2   2 an angle  reaches the corner D at the right
amplitude of the resultant electric field is the along a parallel path. Refer to the figure. Let
coefficient of the sine function. a be the length of the side, so that tan
x a–x a
= . Solving these equations
(a / 2) y a–y
Q.31 In a double slit experiment, the coherent
2a 3
sources are spaced 2d apart and the screen is one gets x = and hence cot  = .
3 4
placed a distance D from the slits. If n th bright
y a–y C
fringe is formed on the screen exactly opposite B  
to a slit, the value of n must be a–x

d2 2d 2 d2 d2
(A) (B) (C) (D) 
2$ D $D $D 4$ D
Sol. [B] x
$D
The fringe width in this case is and the nth
2d  
bright fringe is formed at a distance d away from A D
M
$D
the centre. Therefore, d = n giving the value Q.34 The reflecting surfaces of two mirrors M1 and
2d M2 are at an angle  (angle  between 0° and
of n. 90°) as shown in the figure. A ray of light is
incident on M1. The emerging ray intersects
Q.32 When two sound sources of the same amplitude the incident ray at an angle . Then,
but of slightly different frequencies n1 and n2
are sounded simultaneously, the sound one M2
hears has a frequency equal to
n  n2  
(A) | n1 – n2 | (B) 1 

2  
(C) n1n 2 (D) [n1 + n2] (A)  =  (B)  = 180° – 
Sol. [B] (C)  = 90° –  (D)  = 180° – 2
The resulting sound wave has a frequency equal Sol. [D]
If x is the angle of incidence when the ray strikes
to half the sum of the individual frequencies. Note
the mirror M1 and y be that for mirror M2, then ,
that the resulting intensity varies at the beat
using simple properties of triangle one gets  =
frequency equal to difference of the individual 180° – 2(x + y) and (Students are expected to
frequencies. draw the ray diagram and check.)

www.examrace.com
Q.35 An unpolarized light beam is incident on a Sol. [D]
surface at an angle of incidence equal to When the switch is closed, 5 ohm resistance gets
Brewster's angle. Then, shorted. Thus a current of 1 A flows from b to a.
(A) the reflected and the refracted beams are
both partially polarized Q.38 Two radioactive materials A and B have decay
(B) the reflected beam is partially polarized and
the refracted beam is completely polarized constants 5$ and $ respectively. Initially both
and are at right angles to each other A and B have the same number of nuclei. The
(C) the reflected beam is completely polarized ratio of the number of nuclei of A to that of B
and the refracted beam is partially 1
polarized and are at right angles to each will be after a time
e
other
1 1
(D) both the reflected and the refracted beams (A) (B)
are completely polarized and are at right 5$ 4$
angles to each other 5 4
(C) (D)
Sol. [C] 4$ 5$
Refer to the article on polarization by reflection Sol. [B]
when the ray is incident at Brewster's angle from Using the law of radioactive decay, one can write
any standard book. N A ( t ) N 0 exp(–5$t ) 1
. Solving this one
N B ( t ) N 0 exp(–$t ) e
Q.36 Switch S is closed at t =0. After sufficiently
gets the result.
long time an iron rod is inserted into the
inductor L. Then, the light bulb
L Q.39 The radius of the hydrogen atom in its ground
B
state is a0. The radius of a 'muonic hydrogen'
atom in which the electron is replaced by an
identically charged muon with mass 207 times
R that of an electron. is aμ equal to
a
(A) 207 a0 (B) 0
S 207
(A) glows more brightly a0
(C) (D) a 0 207
(B) gets dimmer 207
(C) glows with the same brightness Sol. [B]
(D) gets momentarily dimmer and then glows Use the expression for the first Bohr radius for
more brightly hydrogen atom This expression indicates that the
Sol. [B] radius is inversely proportional to the mass and
As the rod is inserted, inductance increases and hence the result.
hence the voltage across inductor increases. This
caused a drop in the voltage across the bulb and Q.40 The instantaneous magnitudes of the electric
hence it gets dimmer.
field (E) and the magnetic field (B) vectors in
an electromagnetic wave propagating in
Q.37 In the circuit shown below, the current that
flows from a to b when the switch S is closed is : vacuum are related as
a 10 " (A) E
B
(B) E = c B
C
B
– (C) E = 2 (D) E = c2 B
5" 10 V 20 " c
+
Sol. [B]
At every instant the ratio of the magnitude of the
b
electric field to that of the magnetic field in an
(A) – 1.5 A (B) + 1.5 A
(C) + 1.0 A (D) – 1.0 A electromagnetic wave equals the speed of light.

www.examrace.com
Sol. [A,B,CD]
SUB-PART A-2
Use the given expression for force to get an
Q.41 A monkey holds a light rope that passes over a expression for acceleration. Integrate this to get
smooth pulley. A bunch of bananas of equal an expression for velocity. Unless otherwise
mass as that of the monkey is attached to the stated about the initial conditions, the velocity
other end of the rope. The monkey starts
climbing the rope to get to the bananas. Then,
turns out to be v =
3mT
f0
2
% &
3t 2 T – t 3 . Use this to
(A) the bananas also move up get the required results.
(B) the bananas move downwards
(C) the distance between the monkey and the
bananas decreases Q.44 A sound wave of angular frequency  travels
(D) the distance between the monkey and the with a speed v in a medium of density  and
bananas remains constant. bulk modulus B. Let k be the propagation
Sol. [A, D] constant. If p and A are the pressure amplitude
Note that the masses of the monkey and the bunch and displacement amplitude respectively, then
of bananas are equal and the pulley is smooth. the intensity of sound wave is
Q.42 Consider the curve representing the Maxwell- 1 vp 2
Boltzmann speed distribution of gas molecules (A) BkA 2 (B)
2 2B
at some temperature. Let vrms, vavg and vmp be
the rms, the average and the most probable p2 p2
(C) (D)
speeds respectively. Then, 2v 2 B
(A) the curve has a maximum at vmp Sol. [A, B, C, D]
(B) the area under the curve gives the total Intensity, by definition, is the energy flowing per
number of molecules of the gaseous unit area per unit time. The displacement
system.
P  
(C) vrms > vavg > vmp amplitude is given by A = where k  
(D) vvg < vmp < vrms Bk  v
Sol. [A, B, C] B
kT kT is the propagation constant. The speed v .
Note that vrms = 1.73 , vavg = 1.60 , 
m m
Use these relations to get the required
kT expressions.
vmp = 1.41 where the symbols have their
m
usual meanings. The most probable speed vmp is Q.45 Two conducting plates A and B are placed
the speed at which the curve reaches the peak. parallel to each other at a small distance
The area under the curve is obviously the total between them. Plate A is given a charge q1 and
number of molecules.
plate B is given a charge q2. Then,
Q.43 A particle of mass m moves along a straight
(A) the outer surfaces of A and B (not facing
line under the action of a force f varying with
each other) get no charge
 t – T 2 
time as f = f 0 1 –    where f0 and T are (B) the inner surfaces of A and B (facing each

 T   other) get all the charge
positive constants. Then (C) the inner surfaces of A and B (facing each
(A) the speed of the particle after a time 2T is other) get equal and opposite charge of
4f 0 T q – q2
magnitude 1
3m 2
(B) after time interval of 3T, the particle starts (D) the outer surfaces of A and B (not facing
moving backwards
each other) get charge of the same polarity
(C) between time instants 0 and 2T, the
acceleration first increases and then decreases. q  q2
and of magnitude 1
(D) the particle stops at t = 3T 2

www.examrace.com
Sol. [C, D] Q.48 In cyclotron (particle accelerator) an ion is
Let a charge q  be present on the inner surface of made to travel successively along semicircles
plate A so that on its outer surface the charge is of increasing radii under the action of a
(q1 – q ). Obviously a charge – q  will get induced magnetic field. The angular velocity of the ion
on the inner surface of the plate B and a charge
is independent of
(q2 + q) will move to its outer surface. With these
(A) speed of the ion (B) radius of the circle
charges, write the net electric field at a point
inside the plate and equate it to zero. This relation (C) mass of the ion (D) charge of the ion
can be simplified to get the value of q  and hence Sol. [A, B]
the conclusions. Use the expression qvB = mr2 where v = r.
The symbols carry their usual meanings. This
Q.46 A man with normal vision uses a magnifying indicated that angular velocity is independent of
lens focal length 10 cm. Then, the radius of the circular path and the speed of
(A) magnification of any value is possible the ion.
(B) maximum magnification possible is 3.5
Q.49 Physical quantities A and B have the same
(C) minimum magnification possible is 2.5
(D) magnification depends upon the distance of dimensions. Then.
the lens from the eye (A) A ± B must be a meaningful physical
Sol. [B, C, D] quantity.
In case of a microscope the magnification is (B) A ± B may not be a meaningful physical
 D quantity
1   when the image is formed at the
 f  (C) A/B must be a dimensionless quantity
distance of distinct vision D. However if the (D) both must be either scalar or vector
image is formed at infinity, the magnification is quantities.
D Sol. [B, C]
simply .
f The ratio of two quantities having the same
dimensions must necessarily be a dimensionless
Q.47 An electromagnetic wave is traveling through a quantity. However, two quantities having the
medium of refractive index n1 and is incident at same dimensions may not add to necessarily give
the boundary of a medium of refractive index a meaningful quantity; for example, work and
n2. If the wave reflects at the boundary, torque have the same dimensions but their
(A) the wave undergoes a phase change of addition is meaningless.
180°, if n1 < n2
Q.50 A step voltage V0 is applied to a series
(B) the wave undergoes a phase change of
combination of R and C as shown. Then,
180°, if n1 > n2
V0 VR
(C) the wave undergoes no phase change, if
n1 < n2 10 V R 1 K"
C VC
(D) the wave undergoes no phase change, if 1μF
n1 > n2
t
Sol. [A, D] t=0
Note that an electromagnetic wave undergoes a (A) after sufficiently long time VR = 0
phase change of 180° upon reflection from a (B) as time passes VR decrease as (1/t)
medium that has a higher index of refraction than (C) after 1 ms, VC = 6.3 volt (approximately)
the one in which it is traveling. However, there is (D) initially current through R is 10 mA
no phase reversal if the case is opposite.

www.examrace.com
Sol. [A, C, D] 2L
After sufficiently long time since the charging
current drops to zero, the drop across the
resistance is zero. The time constant is 1 k" × 1 μ
F = 1ms. hence according to the definition of
time constant, the voltage across the capacitor
would be about 63% of the maximum, that is, 6.3 Sol. For the semicircular plate of radius l, the center
volt after 1 ms. Initial current is obviously (10 4l
of mass lies at a distance of from the centre.
volt/1K") = 10 mA. 3
Taking  to be the mass per unit area, the
PART B Marks : 60 position of centre of mass of the remaining piece
* All questions are compulsory. of the square would be at a distance of
* All questions carry equal marks l (3 – 4)
from the centre of the original square
Q.51 A cube of side 10 cm is rigidly joined to a thin 3 (8 – )
rod of length 40 cm. The rod is pivoted at the plate. Now, taking the centre of the original
other end so that the rod along with the cube is square to the origin, the centre of mass of the
able to rotate freely about the pivot in a vertical new structure can be determined. This turns out
plane. A bullet of mass 50 g, moving l
horizontally hits a point of the cube 5 cm from to be at a distance of to the right of the origin.
3
the lower end and gets embedded into it.
Determine the speed of the bullet so that the Q.53 Consider two long parallel and oppositely
system just rises to a horizontal position. (mass charged thick wires of radius d with their
of the rod = 100 g, mass of the cube = 750 g) central axes separated by a distance D apart.
Sol. Note that the collision between the bullet and the
Obtain an expression for the capacitance per
cube is inelastic. Hence the kinetic energy is not
unit length of this pair of wires.
conserved but the momentum is conserved.
Sol. Note that when the two wires form a capacitor,
Equating the angular momentum of the bullet to
the charges reside only on the inner side; positive
that of the rod together with the cube about the
on one and negative on the other. Let us consider
pivot gives a relation angular frequency  = a point P distance r from the axis of one wire.
v Using Gauss flux theorem, the electric field E1 at
, where v is the speed of the bullet. Note
7.49 P due to the positive charge (of surface charge
that, in general, the moment of inertia of the rod density ) on this wire of unit length is E1(2r) =
1 (d ) d
of length l and mass m about the pivot is ml2 ' E1 = . The point P is at a
3 #0 # 0 ( 2r )
and that of the cube of mass m about an axis distance (D – r) from the axis of the other wire
1 carrying negative charge. Again using Gauss
through its centre of mass is ma2 where a sit
6 theorem, the electric field E2 can be written as E2
he side of the cube. The total moment of inertia of d
= . Obviously the two fields are in
the bullet, the rod and the cube about the pivot # 0 2(D – r )
comes out to be 0.168 kg-m2 . After the collision the same direction so that net electric field is E =
the system rises through a height (0.40 + 0.05) m.
d 1 1 
The gain in gravitational potential energy can  . Integrate this between the
then be equated to the loss of rotational kinetic 2# 0
r D – r 

energy. This gives  = 6.646 s–1. From this the limits d and (D – d) to get the potential
speed of the bullet can be calculated to be 49.78  D–d
m/s. difference d ln . Then, the
#0  d 
Q.52 Consider a uniform square plate of side 2l capacitance per unit length turns out to be
made of wood. A semicircular portion is cut # 0
.
and attached to the right as shown. Determine D–d
ln 
the centre of mass of the redesigned plate.  d 

www.examrace.com
Q.54 Fermat's principle states that 'when light ray Q.55 Two sample X and Y of a gas have equal
travels between two points, the path is the one volumes and pressure. The gas in X is allowed
that required the least time'. Use this principle to expand isothermally to 1.5 times its initial
to derive law of reflection regarding angle of volume, while that in Y is allowed to expand
incidence and angle of reflection. You may adiabatically to an equal volume. if the work
refer to the following figure. done in the first expansion is 1.5 times that in
the second, show that the ratio of specific
heats satisfies a relation
3 3 2
1
( – 1) ln  1 –   
 2  2
 3  
Sol. Let n1 and n2 be the number of moles of samples
h
X and Y of the gas, the initial temperatures of T1
1
2 and T2 respectively and p0 and V0 be their initial
pressure and volume. Then, p0V0 = n1RT1 =
n2RT2. Now, the work done in isothermal
d
3
Sol. Consider the total distance d to be made up of x to expansion is W1 = n1RT1 ln   whereas that in
the left of the point of incidence and (d – x) to its 2
right. If n is the refractive index of the medium n 2 RT2 – n 2 RT2 
adiabatic expansion is W2 .
and c the speed of light in vacuum, then the speed –1
c Note that during adiabatic expansion the
in the medium under consideration is   . The
n temperature falls to T . Also in case of adiabatic
total time of travel can be written as process, TV –1 = constant. Applying this, we get
1
x2  h2 (d – x )  h
2 2 3 3
t  . According to T2 = T2   . Using the fact that W1 = W2 ,
c c  
2 2
  we get the required expression.
n n
Fermat principle, for the least time, calculate
 dt 
  and equate it to zero. Using simple
 dx 
x
geometry, we write = sin1 and
x  h2
2

(d – x )
= sin 2, we get the law of
(d – x ) 2  h 2
reflection that the angle of incidence is equal to
the angle of reflection.

www.examrace.com
INDIAN ASSOCIATION OF PHYSICS TEACHERS
NATIONAL STANDARD EXAMINATION IN PHYSICS 2010-2011
Total time : 120 minutes (A-1, A-2 & B)
PART - A (Total Marks : 180)
SUB-PART A-1

Q.1 Displacement of an oscillating particle is given spring balance and distance of hook from the
by y = A sin (Bx + Ct + D). The dimensional hinge point of the inclined plane. The graph
formula for [ABCD] is - that correctly represents this variation is :
(A) [M0 L 1 T0] (B) [M0L0T 1]
0 1 1
(C) [M L T ] (D) [M0L0T0] Spring
Sol. [B]
balance
Apply the rules of dimensional analysis. The
quantity A must have the dimensions of
4 5
3
displacement. The bracketed quantity must be 1 2
dimensionless and hence, B must have the
dimensions of reciprocal of displacement, C must
have the dimensions of reciprocal of time and D
must be dimensionless. f f
Q.2 Two small spheres of equal masses start
moving in opposite directions from a point A in (A) (B)
a horizontal circular orbit with tangential
velocities v and 2v respectively. Between o d o d
collisions, the spheres move with constant
speeds. The number of elastic collisions the f f
spheres will make before they reach point A
again is - (C) (D)
(A) 4 (B) 3 (C) 2 (D) 1
Sol. [C] o d o d
Refer to the figure. The first collision will take
Sol. [A]
place at point B. Due to elastic collision, the
spheres will exchange their velocities and collide The reading on the spring balance is the force
at point C, again there will be an exchange of required to lift the plane. Since the angular
velocities and the bodies will come to point A to displacement is the same every time, the work
collide for the next time. done is fixed and hence the torque. In other
v A 2v words the product of force and distance of point
of application of force from the hinge must be
constant.

120° 120° Q.4 Identical point masses are placed at (n – 1)


120° vertices of a regular polygon of n sides. The
B C &
vacant vertex has a position vector a with
respect to the centre of the polygon. Therefore,
Q.3 On one arm of an inclined plane 5 hooks are the position vector of the centre of mass of the
fixed (at the same separation) to lift the upper system is -
&
arm relative to the other arm kept horizontal as & –a
shown. The hook fixed on the inclined plane is (A) (n – 1) a (B)
(n – 1)
lifted through the same angle with the help of a &
spring balance, using hook 1, 2, 3, 4, 5 in order. & –a
(C) na (D)
A graph is plotted between the reading of n

www.examrace.com
Sol. [B] Sol. [D]
Note that the centre of mass will get shifted in the L L
opposite direction with reference to the position Extension 'l v Ÿ 'l v 2 . The extension
A d
vector of the vacant vertex
is obviously maximum in case of option (d).
Q.8 Consider an expression F = Ax sin 1(Bt) where
Q.5 Three identical balls moving together along a
F represents force, x represents distance and t
horizontal line with velocity v collide with two
represents time. Dimensionally the quantity
similar balls at rest along the same line. The
collision is elastic. After the collision - AB represents -
(A) two balls move with velocity v. (A) energy (B) surface tension
3v (C) intensity of light (D) pressure
(B) two balls move with velocity Sol. [C]
2
Dimensional analysis suggests that the quantity
(C) three balls move with velocity v
Ax on RHS must have the dimensions of force
2v
(D) three balls move with velocity whereas B must have the dimensions of
3 reciprocal of time. Then, the product AB will
Sol. [C] have the dimensions of energy per unit area per
As per the law of conservation of linear
unit time, the same as those of intensity of light.
momentum, the two balls originally at rest along
with one form those already in motion will move
with velocity v. Two of the three balls originally
Q.9 Velocity – displacement curve of a particle
moving will naturally come to rest. moving in a straight line is as shown. Line PB
is normal to the curve and line PA is normal to
Q.6 A block is placed on a surface with vertical the X axis. The instantaneous acceleration of
x2 the particle at P is –
cross section given by the equation y = . If v
20
m/s
the coefficient of friction is 0.5, the maximum P
(0, 4)
height above the ground at which a block can
be placed without slipping is -
(A) 1.00 m (B) 1.25 m
(C) 1.50 m (D) 1.90 m O A(1, 0) B(2, 0) S m
Sol. [B] 2
(A) 2 m/s (B) 1.5 m/s2
By drawing the usual free body diagram, we can
(C) 1 m/s2 (D) zero
write, in equilibrium mg sin T = μsN and mg cos
Sol. [C]
T = N giving tan T = μs. But tan
dv
dy x x Acceleration can be written as v = v tan T
T= = . This gives 0.5 = Ÿ x = 5. ds
dx 10 10
where (tan T) is the slope of the given curve and
x2
Use this value in the equation y = to get other symbols have their usual meanings. Since
20 the slope of PB is –4. The slope of the curve tan
maximum height y = 1.25 m
T = 4. Putting these values, we get the
acceleration as 1 m/s2.
Q.7 Let L be the length and d be the diameter of
cross section of a wire. Different lengths of
Q.10 Suppose that the gravitational force varies
wire of the same material are subjected to the
inversely as the nth power of the distance.
same tension. In which of the following cases
will the extension be maximum ? Then, the period of a planet in circular orbit of
(A) L = 300 cm, d = 1.0 mm radius R around the sun will be proportional to -
n 1 n 1
(B) L = 200 cm, d = 0.5 mm 2
(A) R (B) R 2
(C) L = 100 cm, d = 0.2 mm
(C) Rn (D) R n/2
(D) L = 50 cm, d = 0.05 mm

www.examrace.com
Sol. [A] Q.13 In the circuit shown, the potential differences
In this case we can write, across C1 and C2 are respectively
4S 2 R n 1
2
GMm § 2S · 400 : C2 = 5 μF
= mRZ 2
= mr ¨ ¸ Ÿ T2
= .
Rn © T ¹ GM
This gives the desired result. G 100 :

Q.11 A planet of mass m moves around the sun of C1 500 :


= 4 μF
mass M in an elliptical orbit. The maximum
and minimum distances of the planet from the
sun are r1 and r2 respectively. Therefore, the 2.0 V
(A) 1 volt, 1 volt (B) 1 volt, 1.2 volt
time period of the planet is proportional to -
(C) 2 volt, 1 volt (D) 1 volt, 2 volt
(A) (r1 + r2)3 (B) (r1 + r2)3/2 Sol. [B]
(C) (r1 + r2)2/3 (D) (r1 + r2)4 The total current in the circuit flows through the
Sol. [B] two resistors and the galvanometer only and it is
equal to 2 mA. This produces a drop of 1 volt
The semi-major axis of the elliptical orbit of
across C1 and a drop of 1.2 volt across C2.
§r r ·
planet around the sun is ¨ 1 2 ¸ . With the sun
© 2 ¹ Q.14 A ball is dropped from a height h above a
at the focus, Kepler's law then gives the horizontal concrete surface. The coefficient of
restitution for the collision involved is e. The
proportionality.
time after which the ball stops bouncing is -
Q.12 One mole of an ideal is taken from an initial
2h § 1 · 2h § 1 ·
state A (P0, V0) to a final state B (2P0, 2V0) by (A) ¨ ¸ (B) ¨ ¸
g ©e¹ g ©1 – e ¹
two different processes. – (1) Gas expands
isothermally to double its volume and then 2h § 1  e · 2h § 1 ·
(C) ¨ ¸ (D) ¨ ¸
pressure is doubled at constant volume to the g ©1 – e ¹ g © 1 – e2 ¹
final state. (2) Gas is compressed isothermally Sol. [C]
until its pressure is doubled and then its volume The time required for the free fall of the ball is
is doubled at constant pressure to the final 2h
. Then the time taken for rise and next fall
state. The p-V diagram that correctly represent g
the two processes is : 2h
will be 2 (e) . The time taken for one more
P P g
2p0 2 2p0 2 B
B 2h 2
2 2 1 rise and fall will be 2 (e ) etc. Therefore,
(A) p0 (B) p0 g
A 1 1 A 1 the total time for which the ball will be in
O v0 2v0 V O v0 2v0 V motion, will be
P P 2h 2h
2 B 2 e(1 + e + e2 +….) =
2p0 2 2p0 g g
B 2 1
2
(C) p0 1 (D) p0 2h 2h 1 2h § 2e ·
A 1  2e u ¨1  ¸
1 g g 1– e g © 1– e¹
v
O 0 2v0 V O v0 2v0 V This on simplification gives the result.
Sol. [C]
Since the first step in both the processes is Q.15 A metal block is resting on a rough wooden
isothermal, we have pV = constant giving the pV surface. A horizontal force applied to the block
is increased uniformly. Which of the following
diagram the shape of a rectangular hyperbola. This
curves correctly represents velocity of the
is observed in option (C) only and hence the answer. block ?

www.examrace.com
Sol. [B]
v v
Vector relation between linear velocity and
& & &
angular velocity is v Z u r , so that option (b)
(A) (B) is incorrect.
o t o t Q.18 Three energy levels A, B and C in an atomic
system are such that EA < EB < EC. If the
v v wavelengths corresponding to the transitions
C o B, B o A and C o A are O1, O2 and O3
(C) (D) respectively, then
(A) O1 + O2 + O3 = 0 (B) O32 = O12 + O22
o o
t t O1O 2
(C) O3 = O1 + O2 (D) O3 =
O1  O 2
Sol. [C]
Sol. [D]
Let f = μsN at t = t1. Therefore, v = 0 for t < t1.
For t > t1 net force on the block = kt – b where In terms of energy differences, we can write ECA
dv = ECB + EBA. This can further be written as
b = μkN. Therefore, m = kt – b
dt hc hc hc
 and hence the result.
k 2 b O3 O1 O 2
Ÿv= t – t  A . Now, A = 0 since v = 0
m m
at t = 0. Thus, graph of v against t is a parabolic
Q.19 The refracting angle of a prism is A and
curve as in (c).
refractive index is cot (A/2). The angle of
Q.16 The earth has mass M1 and radius R1. Similarly minimum deviation is -
the sun has mass M2 and radius R2. Distance (A) (180° – A) (B) (180° – 2A)
between their centres is r. It is known that the (C) (90° – A) (D) (90° – 2A)
centre of mass of the earth-sun system lies well
Sol. [B]
within the sun. Therefore.
Use the prism formula
M1 R M1 R
(A) ! 2 (B)  2
M1  M 2 r M1  M 2 r § A  Gm · § § A  Gm ··
sin ¨ ¸ ¨ sin ¨ ¸¸
(C) M1R2 < M2R1 (D) M1R2 > M2R1 μ © 2 ¹ Ÿ cot A ¨ © 2 ¹¸
Sol. [B] §A· 2 ¨ §A· ¸
sin ¨ ¸ ¨ sin ¨ ¸ ¸
Distance of centre of mass from centre of the sun ©2¹ © ©2¹ ¹
M1 r
will be . Since the centre of mass lies §A· § A  Gm ·
M1  M 2 cos¨ ¸ sin ¨ ¸
Ÿ ©2¹ © 2 ¹
M1 r
within the sun < R2 and hence the §A· §A·
M1  M 2 sin ¨ ¸ sin ¨ ¸
©2¹ ©2¹
result.
§A· § A  Gm ·
Ÿ cos ¨ ¸ = sin ¨ ¸ . This suggest that
Q.17 Consider a particle of a rigid body. Its motion ©2¹ © 2 ¹
& & & & &
can be described by vectors r , Z, a r , a t , and D the angles on the two sides are complementary,
(symbols have their usual meanings). Then, § A · § A  Gm ·
which of the following equations is incorrect ? that is, ¨ ¸¨ ¸ = 90°. This can be
& ©2¹ © 2 ¹
& dT & & &
(A) Z (B) v r u Z simplified to get the result.
dt
& & & & & & &
(C) a r Z u (Z u r ) (D) a t D u r

www.examrace.com
Q.20 Refer to the arrangement of logic gates. For Q.23 A cylindrical vessel contains a liquid of
A = 0, B = 0 and A = 1, B = 0, the values of density p filled upto a height h. The upper
surface of the liquid is in contact with a piston
output Y are, respectively - of mass m and area of cross section A. A small
A hole is drilled at the bottom of the vessel.
B (Neglect the viscous effects). The speed with
Y which the liquid comes out of the hole is -
§ m ·
(A) 2gh (B) 2g¨¨ h  ¸¸
(A) 0 and 1 (B) 1 and 0 © pA ¹
(C) 1 and 1 (D) 0 and 0 § m · § 2m ·
(C) g¨¨ h  ¸ (D) g¨¨ h  ¸
Sol. [B] © pA ¸¹ © pA ¸¹
Refer to the truth tables of AND, OR and NOR
gates from any standard book. Sol. [B]
Use Bernoulli's theorem at the upper surface and
Q.21 A plastic ring of radius R has a charge + 3Q at the small hole. We get an equation
distributed uniformly along one quarter of its mg 1 2
circumference and a charge – Q uniformly pgh + Uv where v is the speed of
A 2
distributed along the rest of the circumference. efflux. Solving this we get the expression for
The potential on its axis at a distance of 3R 2mg § m ·
speed v = 2gh  2g¨¨ h  ¸.
is - UA © UA ¸¹
§ 1 · 2Q § 1 · 4Q
(A) ¨¨ ¸
¸ (B) ¨¨ ¸
¸
© 4SH0 ¹ R © 4SH 0 ¹ R Q.24 A charged capacitor discharges through a
resistance R. Let U be the energy stored by the
§ 1 ·Q § 1 · Q
(C) ¨¨ ¸
¸ (D) ¨¨ ¸
¸
capacitor and let P be the rate at which energy
© 4SH0 ¹ R © 4SH 0 ¹ 2R gets dissipated. Then, the time constant is :
Sol. [C] 4U U 2U U
(A) (B) (C) (D)
Using standard relation, the potential P P P 2P
1 ª 3Q Q º Sol. [C]
= « – » Take the ratio of the energy stored in the
4SH 0 «¬ R 2  3R 2 R 2  3R 2 »¼ capacitor to the power dissipated. that is,
which on simplification gives the answer. §1 2·
¨ CV ¸
U ©2 ¹ RC Ÿ RC 2U .
Q.22 The figure shows four orientations, at angle T P § V2 · 2 P
& ¨ ¸
with a magnetic field B , of a magnetic dipole ¨ R ¸
& © ¹
with moment M . The magnitude of torque (W)
and potential energy (U) is best represented by Q.25 The following figure shows different
1 2 arrangements of two identical pieces of plano-
convex lenses. The refractive index of the
T T & liquid used is equal to that of the glass. Then,
B the effective focal lengths in the three cases
T T
are related as –
4 3
(A) W1 = W2, W3 = W4 and U1 = U2 = U3 = U4
(B) W1 = W2 = W3 = W4 and U1 = U4, U2 = U3
(C) W1 = W4, W2 = W3 and U1 = U2 = U3 = U4
(D) W1 = W2 = W3 = W4 and U1 = U2 = U3 = U4 f1 f2 f3
Sol. [B]
Note that the magnitude of the torque acting on liquid
the dipole is W = MB sin T and potential energy is (A) f1 = f2, f3 = 0 (B) f1 z f2 z f3
U = – MB cos T. (C) f1 = f2 > f3 (D) None of the above

www.examrace.com
Sol. [D] Q.29 A photon of wavelength O(less than threshold
The focal lengths f1 and f2 are equal. The wavelength O0) is incident on a metal surface
arrangement in the third case is effectively a plate
of work function W0. The de Broglie
and hence has an infinite focal length.
wavelength of the ejected electron of mass m
Q.26 A long wire carrying a current 1 A is placed is :
along the axis of a long hollow tube of radius 5 ª § hc ·º h
cm also carrying a current of 1 A in the same (A) h «2m¨ – W0 ¸» (B)
¬ ©O ¹¼ § hc
2m¨ – W0 ¸
·
direction. The magnetic field at a distance of
© O ¹
2.5 cm from the axis is :
(A) 8 × 10 6T (B) 16 × 10 6 T (C)
h
(D)
1
6
(C) 4 × 10 T (D) Zero § hc · § hc ·
2m¨ – W0 ¸ h 2m¨ – W0 ¸
©O ¹ ©O ¹
Sol. [A] Sol. []
Note that the current through the wire only will hc
contribute to the magnetic field at a point inside With usual notation, Ek = – W0 . Also if p is
O
the hollow tube.
p2
the momentum of the photoelectron, Ek =
Q.27 A Uniform solid sphere of mass m has a radius 2m
R. The gravitational potential at a distance Ÿ p = 2mEk . Thus, the de Broglie
r (< R) from he centre of the sphere is : wavelength of the ejected electron
Gm Gm 2 2 h h
(A) – 3 (3R 2 – r 2 ) (B) – (R – r ) Oc = which one substitution gives
R 2R 3 p 2mE k
Gm Gm
(C) – 3
(3R 2 – r 2 ) (D) – 3 (R 2 – r 2 ) the answer.
2R R
Sol. [C] Q.30 In the following V-T diagram for a perfect gas,
The gravitational potential at a distance r < R, is the relation between p1 and p2 is :
made up of two parts – one due to the mass of
V p2
sphere of radius r, say V1 and that due to the
remaining mass, say V2. One finds that
GMr 2 3Gm 2 p1
V1 = – 3
and V2 = – 3
(R – r2), and
R 2R
then by adding one gets the result. Refer to any T
standard book. O
(A) p2 = p1 (B) p2 < p1
Q.28 The focal length of a concave mirror is f. An (C) p2 > p1 (D) uncertain
object is placed at a distance x from the focus Sol. [B]
Note that the slope of the V-T diagram is
and forms a real image. Therefore, the
inversely proportional to pressure p.
magnification (numerically) is :
2 2
f §f · x §x· Q.31 A photographic plate placed at a distance of 10
(A) (B) ¨ ¸ (C) (D) ¨ ¸
x ©x¹ f ©f ¹ cm from a point source is exposed for a 4
Sol. [A] second. If the plate is moved farther away by 10
Since the image formed is real the object must be cm, the time required to have the same exposure
beyond the focus. Therefore, taking the object (A) 4 second (B) 16 second
distance to be (f + x) and using mirror formula, (C) 8 second (D) 64 second
f (f  x )
we get the image distance as and then Sol. [B]
x According to the inverse square law, illuminance
v f is inversely proportional to square of the
the magnification .
u x distance.

www.examrace.com
Q.32 The voltage over a cycle varies as Sol. [B]
S Using the law of floatation, we equate the weight
v = V0 sin Zt for 0 d t d
Z of the body to the upthrust and get
S 2S 4 4 3
= – V0 sin Zt for d t d S(R3 – r3)Umg = SR U1g Ÿ
Z Z 3 3
The average value of the voltage for one cycle r3 U1
1– and the result follows.
is : R3 Um
V V 2V0 Q.35 A metal strip 6 cm long, 0.6 cm wide and 0.7
(A) 0 (B) 0 (C) zero (D)
2 2 S mm thick moves with constant velocity v
Sol. [D] through a uniform magnetic field of induction
The voltage represents the output of a full wave 0.9 T directed perpendicular to the strip as
rectifier whose de component or the average shown. A potential difference of 1.62 μV is
2V0 induced across points M and N of the strip.
value is .
S Therefore, the speed v is :

Q.33 Young's double slit experiment is first × × v × ×


performed in air and then by immersing the
whole setup in a liquid. The 10th bright fringe × × × ×
when in liquid is formed at the point where 8th
× × M N× ×
dark fringe is located when in air. The
refractive index of the liquid is :
× × × ×
(A) 1.25 (B) 1.33 (C) 1.40 (D) 1.20
Sol. [B]
× × × ×
OD
Fringe width w = where symbols have their (A) 0.1 m/s (B) 0.2 m/s
d
(C) 0.3 m/s (D) 0.4 m/s
usual meanings. Note that wavelength Oin air
Sol. []
O Error in wording, question deleted.
changes to in a liquid of refractive index μ.
μ
Now, we can write 10 × wliq = 7.5 × wair Q.36 The ratio of magnetic field at the centre of a
O liq D O D O current carrying circular coil to its magnetic
Ÿ 10 7.5 air Ÿ 10 air = 7.5 Oair.
d d μ moment is x. If the current and the radius both
10 are doubled, the value of this ratio would be
This then gives μ = = 1.33. (A) 2x (B) 4x (C) x/4 (D) x/8
7.5
Sol. [D]
Q.34 A spherical shell made of a metal of density Um Note that magnetic field at the centre of a current
remains just below the surface of a liquid of μ nI
carrying coil is 0 and its magnetic moment
density U". If r and R are respectively the inner 2r
and the outer radii of the shell, then, the ratio is Sr2nl. Therefore their ratio varies inversely as r3.
r
is :
R Q.37 A conducting ring of radius r is placed in a
U" U" varying magnetic field perpendicular to the
(A) 1 – (B) 3 1 – plane of the ring. If the rate at which the
Um Um
magnetic field varies is x, the electric field
Um U" intensity at any point of the ring is -
(C) –1 (D) 2 1 –
U" Um (A) r x (B) r x/2 (C) 2 r x (D) 4 r/x

www.examrace.com
Sol. [B] & Sol. []
Let E be the electric field intensity at a point on Error in wording, question deleted.
the circumference of the ring. Then, the emf
&
³
induced H = E ˜ dl where dl is a length element SUB - PART - A -2
& &
of the ring. Since | E | is constant and E || dl , the
Q.41 A hoop rolls down an inclined plane without
integral works out to be E (2Sr). Also the induced
slipping. Then,
dI dB
emf is H = = Sr2 = Sr2x. Equating the (A) the inclined plane is smooth
dt dt
(B) the inclined plane is rough and still there is
two, we get the result.
no loss of mechanical energy
Q.38 Unpolarized light intensity of 32 W/m2 passes (C) the point of contact of the hoop with the
through three polarizers. The transmission axis inclined plane is always at rest
of the last polarizer is crossed with that of the (D) the linear speeds of different points on the
first. If the intensity of light emerging out of rim of the hoop are different
the third polarizer is 3 W/m2, then the angle Sol. [C, D]
between the transmission axis of the first two refer to any standard book.
polarizers is Q.42 Which of the following phenomena is / are
(A) 10° (B) 30° related to the variation in density of
(C) 45° (D) 60° atmospheric air ?
Sol. [B] (A) mirage
Let the angle between the axes of the first two (B) in winter sound of a whistle of a railway
polarizers be T. The intensity after the first engine is heard at much longer distances
polarizer is half of that incident on it, that is 16 (C) twinkling of a star
W/m2. The intensity after the second polarizer (D) visibility of sun for some time after the
will be 16 cos2T W/m2. The intensity after the sunset
third polarizer can be written as 3 = (16 cos2 T) Sol. [A,B,C,D]
cos2 (90 – T) since the angle between the axes of All the phenomena re due to formation of layers
the second and the third polarizers is (90 – T). This of atomospheric air with different densities and
gives on simplification, hence refractive indices.
3
sin (2T) = Ÿ 2q = 60° Ÿ T = 30°. Q.43 The graph shows the displacement of a body
2
as a function of time. Which of the following
Q.39 Two soap bubbles of radii 2r and 3r are in is / are the conclusion ?
contact with each other. The radius of curvature
of the interface between the bubbles is -
(A) 2 r (B) 6 r (C) 3 r (D) r x
Sol. [B]
Excess pressure difference across the interface is
4T 4T 4T 4T
– which must be where R is
2r 3r 6r R
o t
the radius of curvature at the interface. This gives
R = 6r.
(A) The graph represents motion with constant
velocity
Q.40 A radioactive element X converts into another
(B) The graph represents accelerated motion
stable element Y. Half life of X is 3 hrs.
(C) The body comes to rest after a long time
Initially only nuclei of X are present. After
(D) The graph represents a retarded motion
time t, the ratio of number of nuclei of X to that Sol. [C, D]
of Y is found to be 1 : 8. Therefore, Slope of the curve at a point is the velocity which
(A) t = 9 hrs is decreasing in this case and hence the body is
(B) t = 6hrs decelerating. The graph is rising exponentially
(C) t = 7.5 hrs and therefore the body will take a long time to
(D) t is between 6 hrs and 9 hrs come to rest.

www.examrace.com
Q.44 A transistor is connected in common emitter Sol. [A, B, C]
mode. The collector supply is 10 volt and Since in case of a resistor the voltage and the
voltage drop across resistor of 1 k: in the current are in phase, option (d) is not possible.
collector circuit is 0.5 volt. If the current gain is Due to reactive component – capacitor other
E is 49, then options given are possible.
(A) the base current is 50 μA
Q.47 A person is sitting in a moving train and is
(B) current gain D is 0.98.
facing the engine. He tosses up a coin which
(C) the emitter current is about 510 μA
(D) the base current is 10 μA falls behind him. He concludes that the train is
Sol. [B, C] moving -
0.5 (A) forward with increasing speed
The collector current is obviously = 0.5 (B) forward with decreasing speed
1000
mA. use the standard relations for the current (C) backward with increasing speed
gains D, E and the relation between them (D) backward with decreasing speed
D Sol. [A, D]
E= . Also note that IE = IB + IC.
1– D Only in case of forward acceleration and
Q.45 The variation of gravitational field intensity backward deceleration is the given observation
with distance from the centre of a body is possible.
shown in the graph from which one can
conclude that Q.48 For an LCR circuit
A
B
Eg

D
Eg
O Zc Z
4
(A) A and B represent R and Z respectively
r
O R1 R R2 (B) A and B represent Z and R respectively
(C) A, B, C and D represent Z, X1, R and XC
respectively
(A) variation of gravitational field intensity is (D) for Z = Zc, the phase difference between
due to the spherical mass body of radius R current and voltage becomes zero
(B) Eg v r for r < R Sol. [C, D]
(C) the separation of two points R1 and R2 is
Note that R is independent of Z, XL directly
9R/4
(D) the separation of two points R1 and R2 is proportional to Z and XC inversely proportional
R/4 to Z. Again Z has a maximum or a minimum
Sol. [A, B] value at Z = Zc at which the voltage and the
Refer to any standard book. current are in phase.
Q.49 A furnace has a two layered wall as shown
Q.46 When an alternating current flows through a
circuit consisting of a resistor in series with a schematically. Each layer has the same area of
capacitor, during the cycle at some instant it is cross section. The temperature T at the
possible to have - interface of two layers can be reduced by
(A) voltage across the circuit zero but current "i "0
through it not zero inner outer
(B) current through the circuit zero but the layer layer
voltage across it not zero
(C) current through the capacitor not zero but 800°C 80°C
ki k0
the voltage across it zero
(D) current through the resistor not zero but the
voltage across it zero T°C

www.examrace.com
(A) increasing the thermal conductivity of outer PART B Marks : 60
layer * All questions are compulsory.
(B) decreasing the thermal conductivity of * All questions carry equal marks
inner layer Q.51 Assume that a constant power P is supplied to
(C) by increasing the thickness of inner layer an electric train and it is fully used in
(D) by decreasing the thickness of outer layer accelerating the train. Obtain relation giving
Sol. [A, B, C, D] the velocity of the train and distance traveled
800 – 80 by it as functions of time.
Rate of heat flow H = which
§ l1 · § l0 · P
¨¨ ¸¸  ¨¨ ¸¸ Sol. Power P = constant, therefore P = Fv Ÿ F =
© K 1A ¹ © K 0 A ¹ v
800 – T dv P
is also equal to . Using these two Ÿ m = . Integrating this we get
§ l1 · dt v
¨¨ ¸¸
© K 1A ¹ 2P dx
v= t . Writing the velocity v as and
720 m dt
relations we get, T = 800 – . Thus further integrating, we get the expression for the
§ K1 ·§ l0 ·
1  ¨¨ ¸¸¨¨ ¸¸
2 2P 2 3
© K 0 ¹© l1 ¹ distance x = t .
one can reduce the temperature at the interface by 3 m
any of the four options given.
Q.52 A block of mass 1.5 kg rests on a rough
Q.50 Simple pendulums P1 and P2 have lengths horizontal surface. A horizontal force applied
l1 = 80 cm and l2 = 100 cm respectively. The to the block increases uniformly from 0 to 15
bobs are of masses m1 and m2. Initially both are N in 5 second. Determine velocity and
at rest in equilibrium position. If each of the displacement of the block after 5 second. Use
bobs is given a displacement of 2 cm, the work μs = 0.6 and μk = 0.5 and g = 10 m/s2.
done is W1 and W2 respectively. Then Sol. With usual notation, Fs = μsN = 9N and
(A) W1 > W2 if m1 = m2 Fk = μkN = 7.5 N. Applied force rises to Fs = 9 N
(B) W1 < W2 if m1 = m2 at t = 3s. Therefore, for t d 3 s, v = 0 and s = 0.
m 5 For t t 3 s, net force on the block is (3 t – 7.5),
(C) W1 = W2 if 1 dv dv
m2 4 that is m = 3t – 7.5 Ÿ = 2t – 5 Ÿ
dt dt
m1 4
(D) W1 = W2 if v = t2 – 5t + A. Here m = 1.5 kg. Now, at t = 3 s, v
m2 5 = 0 , gives A = 6. With this we get for t t 3 s, v =
Sol. [A, D] t2 – 5t + 6. Therefore, at t = 5 s, v = 6 m/s. From
With usual notation, the height through which the ds
the equation for v, we get = t2 – 5t + 6 Ÿ s =
§ T· dt
bob falls is h = l(1 – cosT) = l ¨ 2 sin 2 ¸ =
© 2¹ t 3 5t 2
– + 6t + B. Again at t = 3 s, s = 0, giving
§ T2 · 3 2
2l ¨¨ ¸ since T is small. Therefore, we can write
¸
© 4 ¹ 9
B = – . With this we get,
2 2
lT 2 l §a· a2
h= ¨ ¸ . Thus, the work done W t 3 5t 2 9
2 2© l ¹ 2l s = –  6t – . Hence at t = 5 s,
3 2 2
mga 2 1 14
= P.E. = mgh = ŸWv s= = 4.67 m.
2l l 3

www.examrace.com
Q.53 A tower used for power transmission leaks a Sol. Refer to the figure. With usual notation, we have
current 1 into the ground. Assume that the 1 2qV mv 2
mv 2 = qV Ÿ v = . Again, =
current spreads uniformly (hemispherically) 2 m R
into the ground. Let p be the resistivity of the mv
qvB Ÿ R = . Eliminating v from the two
ground and r be the distance from the centre of qB
the tower (assumed to be rod). The lower end
1 2mV
of the rod is spherical with radius b. Determine relations, we get R . Now, sin T =
B q
(1) current density as a function of r, (2)
magnitude of electric field at a distance r. and l q
lB . Substituting the values gives
(3) potential difference between the lower end R 2mV
of the rod and a point distance r. 1
sinT = Ÿ T = 30°
2
tower O

T× ×
R
b r inside
× ×
ground R
" T
& & × ×
Sol. Use I = ³ J ˜ dA where current density vector J
v
is parallel to the area element dA of the q × ×
hemispherical surface. When integrated, the area
comes out to be 2Sr2. This gives the current
Q.55 A thin planoconvex lens of focal length f is cut
I
density directed radially outward at any along the axis into two halves. The two halves
2Sr 2 are placed at a distance d from each other as
point. Using microscopic form of Ohm's law J = V E shown. The images formed by the two halves
where s is the conductivity, we get the magnitude of lie in the same plane. The distance between the
J I object plane and the image plane is 1.8 m. The
electric field E = U where we have used magnification produced by one of the halves is
V 2Sr 2 2. Determine f, d and the magnification
J produced by the other half.
U = . Now, to determine the potential
V Sol. Obviously L1 forms an image with magnification
dV v
difference we use the relation Er = – Ÿ dV 2 = 1 so that v1 = 2u1. Again u1 + v1 = 1.8 m
dr u1
giving u1 = 0.6 m and v1 = 1.2 m. Using lens
= – ³ Edr . Substituting the value of E and
formula we get f = 0.4 m. Now, for lens L2, u2 =
integrating between the limits b and r, we get the d + 0.6 and v2 = 1.2 – d. Using these values and
also f = 0.4, we get d (d – 0.6) = 0 or d = 0.6 m.
IU ª 1 1 º Further the magnification produced by L2 is
potential difference = –
2S «¬ r b »¼ 0.6 1
= .
1.2 2
Q.54 An alpha particle is accelerated through a
potential difference of 10 kV. Then it enters object d image
into a region of transverse magnetic field of plane plane
induction 0.10 T extended upto a distance of
0.10 m. Determine the angle through which the L1 L2
alpha particle deviates. (mass of the alpha
particle = 6.4 × 10 27 kg)

www.examrace.com
INDIAN ASSOCIATION OF PHYSICS TEACHERS
NATIONAL STANDARD EXAMINATION IN PHYSICS 2011-2012
Total time : 120 minutes (A-1, A-2 & B)
PART-A (Total Marks : 180)
SUB-PART A-1 : ONLY ONE OUT OF FOUR OPTIONS IS CORRECT
N.B. Physical constants are given at the end
SUB-PART A-1
1. A piece of n-type semiconductor is subjected to an electric field Ex. The left end of the semiconductor is
exposed to a radiation so that electron-hole pairs are generated continuously. Let n be the number density of
dn
electrons. The electron current density Je, is given by Je = eneEx + eDe . The dimensions of electron drift
dx
mobility (e) and electron diffusion coefficient (De) are respectively.
(a) [M 1 T 2 I1] and [L2T 1] (b) [M1 T 2 I 1] and [M1 L2 T 1]
1 2 1 2 1
(c) [M T I ] and [L T ] (d) [M 1 T2 I2] and [L1T 2 I1]
Ans. [c]
dn
Sol. J e  en e E x  eDe
dx
ITL3 ( e ) MLT 2 1
IL 2   IT ( De ) 4
IT L
1 2
e = [M I T ]
De = [L2T 1]

2. A metal sample carrying a current along X axis with density Jx is subjected to a magnetic field Bz (along Z
axis). The electric field Ey (Hall field) developed along Y axis is directly proportional to Jx as well as Bz. The
constant of proportionality (Hall coefficient) has SI unit
(a) C/m2. (b) m2s/C (c) m2/C (d) m3/C
Ans. [d]
Sol.
y

e x

B
z
I
Jx = J x
A
I = J xA
evdBz = eEy
Ey = vdBz
neAvd = JxA
J B
Ey  x z
ne
1 m3
Constant of proportionality = =
ne C

www.examrace.com
A 3
3. A vibratory motion is represented by x = 2A cos t + A cos
t    A cos(t  )  cos
t   . The
2  2 2 
resultant amplitude of the motion is
9A 5A 5A
(a) (b) (c) (d) 2A
2 2 2
Ans. [b]
Sol.
A

A/2
A 2A
R

A/2 A

A2 5A
R  A2  
4 2

1/3
4. A force (F) acting on a body is dependent on its displacement s as F  s . Therefore, the power delivered
by the force varies with its displacement as
(a) s2/3 (b) s1/2 (c) s 5/3 (d) s0
Ans. [d]
Sol. F = k s 1/3
k
a = s 1 / 3
m
k
 vdv  s 1 / 3 ds
m
v 2 k  3s 2 / 3 
   …. (i)
2 m  2 
v  s1/3
P = Fv  s 1/3 × s1/3
P  s0

5. Bamboo strips are hinged to form three rhombi as shown. Point A0 is fixed to a rigid support. The lengths of
the side of the rhombi are in the ratio 3 : 2 : 1. Point A3 is pulled with a speed v. Let vAl and vA2 be the speeds
with which points A1 and A2 move. Then, the ratio vAl : vA2 is -

(a) 2 : 3 (b) 3 : 5 (c) 3 : 2 (d) 5 : 2

www.examrace.com
Ans. [b]
Sol.

4x cos 
3x 3x
2x 2x x x 2x cos 
 6x cos   x
  

As per constant relation


v A1 3

v A2 5

6. A particle of mass m is made to move with uniform speed v along the perimeter of a regular hexagon.
Magnitude of impulse applied at each corner of the hexagon is
mv
(a) mv (b) mv 3 (c) (d) zero
2
Ans. [a]
Sol.

v
60º
v
60º
Change = 2v sin
  v , Imp. = p = mv
2 
7. Two chambers of different volumes, one containing m1 g of a gas at pressure p1 and other containing m2 g of
the same gas at pressure p2 are joined to each other. If the temperature of the gas remains constant the
common pressure reached is
m p  m2 p 2 m p  m2 p1 m p (p  p ) (m1  m2 ) p1 p2
(a) 1 1 (b) 1 2 (c) 1 22 1 2 2 (d)
m1  m2 m1  m2 m1  m2 m1 p2  m2 p1
Ans. [d]
Sol.

m1g m2g

m1 m
n1  ; n2  2
M M
No. of mole remain constant
p1V1 p2V2 pV1 pV2
  
RT RT RT RT
p1V1  p2V2
p=
V1  V2
m m
p1 × V1 = 1 RT ; p2V2 = 2 RT
M M
m RT  m2 RT p1 p2 (m1  m2 )
p 1 
m1RT m2 RT m1 p2  m2 p1

p1 p2

www.examrace.com
8. Two liquid drops of equal radii are falling through air with the terminal velocity v. If these two drops
coalesce to form a single drop, its terminal velocity will be
(a) 2v (b) 2v (c) 3 4v (d) 3 2v
Ans. [c]
Sol. Terminal velocity, vT  r 2
Let radius of single drop = r
and radius of bigger drop = R
4 r 3 4 R 3
2 
3 3
1/3
 R = (2) r
As vT  r2
vT'  R 2  (2)1 / 3 r 2
vT 1 1
'
 1/ 3  1/ 3
vT (2) ( 4)
vT'  3 4vT
9. An elevator of mass M is accelerated upwards by applying a force F. A mass m initially situated at a height
of 1m above the floor of the elevator is falling freely. It will hit the floor of the elevator after a time equal to
2M 2M 2M 2M
(a) (b) (c) (d)
F  mg F  mg F F  Mg
Ans. [d]
F
Sol. ae  
m
as   g 
F F  Mg
as/e = as – ae = g
m M
1 2
s  ut  at
2
1 F  Mg 2
1
t
2 M 
M
t 2
F  Mg
10. The formation of solid argon is due to vander Waals bonding. In this case the potential energy as a function
of interatomic separation can be written as (Lennard Jones 6-12 potential energy) E(r) = – Ar 6 + Br 12 where
A and B are constant > Given that A = 8.0 × 10 77 Jm6 and B = 1.12 × 10 133 Jm12 , the bond length for solid
argon is
(a) 3.75 nm (b) 0.0375 nm (c) 0.750 nm (d) 0.375 nm
Ans. [d]
dE (r )
Sol.  6 Ar 7  12 Br 13  0
dr
6 A 12 B
 0
r 7 r13
12 A 2 B 2  1.12  10 133
r6   = = 0.28 × 10 56
6A A 8  10 77
r6 = 0.28 × 10 2 × 10 54 = 0.0028 × 10 54
r = 0.375 nm

www.examrace.com
11. Let A and B be the points respectively above and below the earth's surface each at a distance equal to half the
radius of the earth. If the acceleration due to gravity at these points be gA and gB respectively, then gB : gA
(a) 1 : 1 (b) 9 : 8 (c) 8 : 9 (d) zero
Ans. [b]

GM 4GM
Sol. gA  2

R 9R 2

R  
2
GMr GM R GM
gB  3  3  =
R R 2 2R 2
gB 1 9 9
  
gA 2 4 8

12. Let vrms, v* and vavg represent the root mean square, the most probable and the average velocities respectively,
in case of a gaseous system in equilibrium at certain temperature. Then, vrms : v* : vavg is
(a) 8 : 3 : 2 (b) 8 : 2 : 3 (c) 3 : 2 : 8 (d) 3 : 2 : 8
Ans. [c]
3RT
Sol. vrms 
M
2 RT
vm. p. 
M
8RT
vav. 
M
vrms : vm.p. : vav. = 3 : 2 : 8

13. In the arrangement of resistance shown below, the effective resistance between points A and B is

(a) 23.5 ohm (b) 38.0 ohm (c) 19.0 ohm (d) 25.0 ohm
Ans. [c]
Sol.
3
15 12
1 25 5
30
10 10
A 24
A 7 B B
30 24 10
10
2 25
6
15 12
4

www.examrace.com
24

10 30 25 12 10


A 3 B
10 30 12 5 10
1 7 4
2 25 6
24
15/2

12 

5 15 25/2  5
A B
6
15/2 

5 15 12 
A B
5
15/2  6

Req. = 19

14. A block of material of specific gravity 0.4 is held submerged at a depth of 1 m in a vessel filled with water.
The vessel is accelerated upwards with acceleration of a0 = g/5. If the block is released at t = 0 s, neglecting
viscous effects, it will reach the water surface at t equal to (g = 10 ms 2)
(a) 0.60 s (b) 0.33 s (c) 3.3 s (d) 1.2 s
Ans. [b]
Sol. In frame of vessel
g g
 V s
g    V "
g    V s a
5 5

"   s  g  g  a


s 5

0.6
(12)  a
0.4
a = 18 m/s2
1 2
s  ut  at
2
1
1  18t 2
2
1 1
t2   t s
9 3

www.examrace.com
15. The maximum tension in the string of a simple pendulum is 1.2 times the minimum tension. If 0 is the
angular amplitude, then 0 is
4 3 15 7
(a) cos 1
 (b) cos 1
 (c) cos 1
 (d) cos 1

5 4 16  8
Ans. [c]
Sol.

0
T2
T1

v
2
mv
T1  mg 
"
m
T1  mg  2 g"(1  cos )!
"
T1 = 3mg – 2 mg cos 
T2 = mg cos 
T1 6

T2 5
3mg  2mg cos  6

mg cos  5
15 mg – 10 mg cos  = 6 mg cos 
15 mg = 16 mg cos 
cos  = 15/16

16. A uniform line charge with density " = 50 C/m lies along X axis. The electric flux per unit length crossing
the portion of the plane z = – 3 m bounded by y = ± 3 m is
(a) 4.68 C/m (b) 9.36 C/m (c) 50 C/m (d) 18.7 C/m
Ans. []
Sol.
z

" y
32  x 2
3
x=0  dx
x


"
4  x2
dA 2 #0
d$  EdA cos 
"  dx 3
= 
2 # 0 9  x 2 y  x2

www.examrace.com
2 2
3"  dx  3" x
 d$   2
= 2 tan 1 
2
# 0 (9  x )  3 # 0 3 0

 " 2
2   tan 1 
 #0 3
IAPT has given (b) as the correct answer to this question.
But The dimensions of the answer & value of answer is not matching to any of the given options Hence the
question is wrong.

17. A plane mirror perpendicular to XY plane makes an angle of 30º with the X axis. An object placed at (–20, 0)
forms an image in the mirror. The point of incidence is (0, 0) and the plane of incidence is the XY plane. The
coordinates of the image are :
(a) (10 3 , 10) (b) (10 3 , 10) (c) (10,  10 3 ) (d) (10 3 ,  10)
Ans. [b,c]
Sol.

20 30º x
(–20, 0) 30º
30º30º y
20
I x
x
sin 30º =
20
1
x  20  10
2
y
cos 30º =
20
3
y  20  10 3
2
x  10 , y = – 10 3

18. Magnetic flux through a stationary loop with a resistance R varies during the time interval % as $ = at(% – t)
where a is a constant. The amount of heat generated in the loop during the time interval % is
a 2 %3 a 2 %3 a 2 %3 a 2 %3
(a) (b) (c) (d)
6R 4R 3R 2R
Ans. [c]
d$
Sol. e  a%  2at
dt
dH e 2 (a%  2at ) 2
 =
dt R R
% %
(a%  2at ) 2  (a%  2at ) 3  (a%  2a%) 3  (a%) 3  a 3 % 3  a 3 %3 a 2 %3
H  R
dt =   =
 R (3)(2a )  0 (6a ) R
=
(1)(6a) R
=
3R
0

www.examrace.com
19. Four functions given below describe motion of a particle. (I) y = sin t – cos t, (II) y = sin3 t,
3
(III) y  5 cos
 3t  , (IV) y = 1 + t + 2t2. Therefore, simple harmonic motion is represented by
4 
(a) only (I) (b) (I), (II) and (III) (c) (I) and (III) (d) (I) and (II)
Ans. [c]
Sol. I & III are showing S.H.M. on the basis of superposition principle.

20. A magnetic field is established with the help of a pair of north and south poles as shown. A small bar magnet
placed freely in the field will undergo

(a) pure translational motion


(b) pure rotational motion
(c) rotational motion superimposed on translational motion
(d) oscillatory motion
Ans. [c]
Sol.

B in Non uniform
& Rotational motion super imposed on translation motion

Q.21 In a hydrogen atom, the magnetic field at the centre of the atom produced by an electron in the nth orbit is
proportional to -
1 1 1 1
(a) 2 (b) 3 (c) 4 (d) 5
n n n n
Ans. [d]
 0 ev


 I 2 R 
Sol. B= 0 =
2 R 2 R
v (1 / n)
B 2  2 2
R (n )

www.examrace.com
1
B
n5
'
Q.22 A particle of mass m carries a charge +q. It enters into a region of uniform magnetic field B existing below
the line l l' as shown. The time spent by the particle in the magnetic field is -

m
(a) ( –2) (b) infinite as the particle gets trapped
qB
m m
(c) 2 (d) ( + 2)
qB qB
Ans. [d]
Sol.
+q


l l'
90

× 90  90  ×
90 
× × C × B
 )

2 (  2) m
Time spend = = .
w qB
Q.23 A 2F capacitor is charged as shown in the figure. The change in its stored energy after the switch S is
turned to position 2 is -

(a) 96% (b) 20% (c) 4% (d) 80%


Ans. [a]
1
Sol. U2F = (2)(v)2
2
2 V  8  0 V
Vcommon = =
28 5
2
1 V
& U 2' F = (2)

2 5

www.examrace.com
U '–U
& * Change = × 100 = 96%
U
Q.24 An infinite number of charges each equal to 0.2 C are arranged in a line at distances 1, 2, 4, 8 … meter from
a fixed point. The potential at the fixed point is -
(a) 1800 volt. (b) 2000 volt. (c) 3600 volt. (d) 2250 volt.
Ans. [c]
Sol.
0.2 0.2 0.2
1 1 1 1 
V = k 0.2     .......  10 6
1 2 4 8 
1 9 16
V=
 × 9 × 10 × 0.2 × 10
1 – 1/ 2 
V = 3600 volt

Q.25 A ball of mass m moving with a speed u along a direction making an angle  with the vertical strikes a
horizontal steel plate. The collision lasts for a time interval t. If e is the coefficient of restitution between the
ball and the plate, the average force exerted by the plate on the ball is -
emu emu cos  2(e  1)mu cos  (1  e)mu cos 
(a) (b) (c) (d)
t t t t
Ans. [d]
Sol.

u v
 $

v cos $
e=  v cos $ = eu cos  …… (i)
u cos 
u sin  = v sin $ ….. (ii)
p
Av. force =
t
m[v cos $  u cos ] mu cos (1  e)
 
t t
Q.26 In a Young's double slit experiment sources of equal intensities are used. Distance between slits is d and
wavelength of light used is "(" << d). Angular separation of the nearest points on either side of central
maximum where intensities become half of the maximum value is -
" " " "
(a) (b) (c) (d)
d 2d 4d 6d
Ans. [b]
Sol.


x

I+I+2 I I cos $

www.examrace.com
2I = 2I + 2I cos$
cos $ = 0

$=
2
2 x
=
" 2
"
x =
4
dx "
=
D 4
D"
x=
4d
2x 2 D" "
= = × =
D D 4d 2d

Q.27 The variation of magnetic field along the axis of a solenoid is graphically represented by (O is the centre with
l, l' as the extremities of the solenoid along the axis)

(a) (b)

(c) (d)

Ans. [d]

Q.28 A wooden cube is placed on a rough horizontal table. A force is applied to the cube. Gradually the force is
increased. Whether the cube slides before toppling or topples before sliding is independent of -
(a) the position of point of application of the force ..
(b) the length of the edge of the cube.
(c) mass of the cube.
(d) coefficient of friction between the cube and the table.

www.examrace.com
Ans. [c]
Sol. IAPT has given (b) as the correct answer to this question.
But the most appropriate answer to this question should be (c). For more details refer the solution given below
N

f
a

mg
The block will slide if F > μ mg
a mga
The block will topple if Fh > mg
 or F >
2
 2h
mga a
The sliding will occur earlier if μmg < μ<
2h 2h
a
& topping will occur earlier if μ >
2h
So independent of mass.
[This answer is correct but the answer provided by IAPT is wrong]

Q.29 There are two organ pipes of the same length and the same material but of different radii. When they are
emitting fundamental notes -
(a) broader pipe gives note of smaller frequency
(b) both the pipes give notes of the same frequency
(c) narrower pipe gives note of smaller frequency
(d) either of them gives note of smaller or larger frequency depending on the wavelength of the wave.
Ans. [a]
v
Sol. f= where x = 0.6 r
4(l  2 x)
& higher the radius lower the frequency

Q.30 The wavelength of sodium line observed in the spectrum of a star is found to be 598 nm, whereas that from
the sodium lamp in the laboratory is found to be 589 nm. Therefore, the star is moving with a speed of about -
(a) 2.7 × 106 m/s away from the earth (b) 5.4 × 106 m/s towards the earth
(c) 1.6 × 106 m/s away from the earth (d) 4.6 × 106 m/s away from the earth
Ans. [d]
Sol. " = 589
"' = appearant = 598 at is higher
than original " & some is moving away.
"'–" v
=
"' C
598 – 589 v
=
598 3 108

www.examrace.com
3  108  9
V=
598
= 4.6 ×106 m/s away from earth.
Q.31 In a series LCR circuit, impedance Z is the same at two frequencies f1 and f2. Therefore, the resonant
frequency of this circuit is -

f1  f 2 2 f1 f 2 f12  f 22
(a) (b) (c) (d) f1 f 2
2 f1  f 2 2
Ans. [d]
Sol. Z f1 = Z f 2

 R is same in both series


so, X f1 = X f 2

 ( X L – X C ) f1 = ( X C – X L ) f 2

 ( X L ) f1 + ( X C ) f 2 = ( X C ) f 2 + ( X C ) f1

1 1 1
 2 L(f1 + f2) =
 
2 C
f1 f 2 

1 f1  f 2
 2 L(f1 + f2) =


2 C f1 f 2 
1
 4 2LC =
f1 f 2
1
 2 LC =
f1 f 2

1
 Resonant frequency = = f1 f 2
2 LC
Q.32 Two particles are moving along X and Y axes towards the origin with constant speeds u and v respectively. At
time t = 0, their respective distances from the origin are x and y. The time instant at which the particles will
be closest to each other is -
x2  y 2 vx  uy ux  vy 2 x2  y2
(a) (b) (c) (d)
u 2  v2 u 2  v2 u 2  v2 uv
Ans. [c]
Sol.
x – ut ut

y – vt

vt
Z2 = (x – ut)2 + (y – vt)2
dZ
2Z = – 2u(x – ut) + 2v(y – vt) = 0
dt

www.examrace.com
ux – u2t + vy – v2t = 0
ux  vy
t= 2
u  v2

Q.33 A container of volume 0.1 m3 is filled with nitrogen at a temperature of 47°C and a gauge pressure of
4.0 × 105 Pa. After some time, due to leakage, the gauge pressure drops to 3.0 ×105 Pa and the temperature to
27°C. The mass of nitrogen that has leaked out is about -
(a) 128 g (b) 84 g (c) 154 g (d) 226 g
Ans. [b]
Sol. P1 = 5 × 105
V1 = 0.1 m3
T1 = 320 k
P1V1 5  10 4
n1 = =
RT1 8.3  320
4  10 4
n2 =
8.3  300
10 4  5 4 
n = n1 – n2 = –
8.3  320 300 


10 4 1500 – 1280 

8.3  300  320 
220  10 4

320  3  8.3  10 2
 2.76 mole
Now of N2 leaked out = 2.76 × 28
 77.28 gm
Most probable answer = 84 gm

Q.34 Ninety percent of a radioactive sample is left over after a time interval t. The percentage of initial sample
that will disintegrate in an interval 2t is -
(a) 38% (b) 19% (c) 9% (d) 62%
Ans. [b]
Sol. In time interval of 't', 10% decays so in next interval of 't' again 10% of remaining sample will decayed hence
total 81% sample in left so 19% will decay.

Q.35 A circuit is arranged as shown. At time t = 0 s, switch S is placed in position l. At t = 5 s, contact is changed
from 1 to 2. The voltage across the capacitor is measured at t = 5 s and at t = 6 s. Let these voltages be V1 and
V2 respectively. Then, V1 and V2 respectively are -

www.examrace.com
(a) 10 volt and 0 volt (b) 9.18 volt and 3.67 volt
(c) 9.18 volt and 3.37 volt (d) 10 volt and 3.67 volt
Ans. [c]
Sol. At t = 0 Charging is started
At t = 5 sec Voltage across capacitor

Vc = V0 1  e  t / RC 

5
=

10 1  e 2010 3
10010 6 




= 10 1  e  2.5 
= 9.18 volt
At t = 5 sec discharging is started
At t = 6 sec capacitor has been discharged
for 1 sec so
Vc = V0 e  t / RC
1

= 9.18 e 1010
3
10010 6

= 9.18 × e
= 9.18 × 0.37
= 3.37 volt

Q.36 There are two thermocouples A and B made of the same pair of metals. In A each wire is 50 cm long and in B
each wire is 150 cm long. Both the thermocouples are maintained between the same lower temperature 1
and higher temperature 2. Each of the two thermocouples is connected to the same microammeter
successively. Let # be the thermoemf and I be the thermoelectric current. Then which of the following
statements is true ?
(a) both # and I are equal for A and B (b) Both # and I are greater for B than those for A
(c) # is the same for both but I is greater for A (d) # is the same for both but I is greater for B.
Ans. [c]
Sol. (i) Temperature difference is same thermo emf will be same.
(ii) Length of loop B is more so resistance of loop B is more.
 Thermoelectric current in loop B is less.

Q.37 Two identical particles move at right angles to each other, possessing de Broglie wavelengths "1 and "2. The
de Broglie wavelength of each of the particles in their centre of mass frame will be -
"21  "22 "1  " 2 2"1" 2 2"1" 2
(a) (b) (c) (d)
2 2 "1  " 2 "21  "22
Ans. [D]
Sol. Let m be the mass of each particle and thus verticals are v1 and v2 respectively

www.examrace.com
v1
y

v2 x
mv2 v 2
vcm( x )  
2m 2
mv v
vcm( y )  1  1
2m 2
v2 v2
v2cm( x )  v2  
2 2
v1
v2cm( y )  
2

v 22 v12 v12  v 22
So magnitude of velocity of 2nd particle is C.O.M frame becomes  =
4 4 2
h h
"=  v1 =
mv1 m"1
h h
"2 =  v2 =
mv2 m" 2
So De broglie wavelength of 2nd partical in COM frame is
h 2h 2
"= = =
2
v  v2 2
h 2
h 2 1 1
m 1 m  2 2 2
 2
2 2 2
m "1 n2 " 2 "1 " 2

2" 1 " 2

"21  "22

Q.38 The stopping potential for photoelectrons emitted from a surface illuminated by light of wavelength 400 nm
is 500 mV. When the incident wavelength is changed to a new value, the stopping potential is found to be
800 mV. New wavelength is about -
(a) 365 nm (b) 250 nm (c) 640 nm (d) 340 nm
Ans. [a]
12400
Sol. Energy of photon (1) E1 = eV
"( Å )
I case
12400
E1 = = 3.1 eV
4000
 Vstopping = 500 m  KE of emitted e = 0.5 eV
 Ephoton = W + KEe
 Work function w = 2.6 eV
II case
Vstopping = 800 mv  KEe = 0.8 eV

www.examrace.com
& Ephoton = W + KEe
= 2.6 + 0.8
= 3.4 eV
12400
 Ephoton = eV
"( Å )
12400
3.4 =
"
" = 3650 Å = 365 nm
Q.39 for the logic circuit given below, the outputs Y for A = 0, B = 0 and A = 1, B = 1 are -

(a) 0 and 1 (b) 0 and 0 (c) 1 and 0 (d) 1 and 1


Ans. [b]
Sol.
P
A
X
y
Y
B Z
A B X Y Z P y
0 0 1 1 0 0 0
1 1 0 0 0 0 0

Q.40 In a hydrogen like atom electron makes transition from an energy level with quantum number n to another
with quantum number (n –1). If n >> 1, the frequency of radiation emitted is proportional to -
1 1 1
(a) (b) (c) n2 (d)
n2 n3 n4
Ans. [b]
13.6 Z 2
Sol. En = –
n2

13.6 Z 2
E(n 1) =–
(n – 1) 2

 1 1
E = 13.6 Z2  2
– 2
 (n – 1) n 

www.examrace.com
 n 2 – (n – 1) 2 
 13.6 Z2  2 2 
 n (n – 1) 

 2n – 1 
 13.6 Z2  2 2
 n (n – 1) 
1
 as (n >> 1).
n3

www.examrace.com
Sub-Part A-2
In question 41 to 50 any number of options (1 or 2 or 3 or all 4) may be correct. You are to identify all of them
correctly to get 6 marks. Even if one answer identified is incorrect or one correct answer is missed, you get
zero score.

41. Consider an electron orbiting the nucleus with speed v in an orbit of radius r. The ratio of the magnetic
moment to the orbital angular momentum of the electron is independent of
(a) radius r (b) speed v
(c) charge of electron e (d) mass of electron me
Ans. [a, b]
magentic moment q
Sol. =
angular moment 2m

e
is valid for all charge moving on a circular path =
2me

42. A current I0 enters into a parallel combination of resistors R1 and R2. Current I1 flows through R1 and I2
through R2. The current I0 distributes in such a way that
(a) power consumed in R1 and in R2 is the same
(b) total power consumed in R1 and R2 is minimum
(c) I1 is proportional of R2 and I2 is proportional to R1
(d) the power consumed in each of R1 and R2 is minimum
Ans. [c, b]
I1 R1

Sol. I0

I2 R2
(i) In parallel I  1/R
 I1 : I2 = R2 : R1
(ii) Total power = I12 + I22R2
P = I12R1 + (I0 – I1)2 R2
dP
= 0 for minima
dI1
2I1R1 + 2(I0 – I1) × (–1)R2 = 0
I1(R1 + R2) – I0R2 = 0
I0R 2
I1 =
R1  R 2
It mean total power is minima.

www.examrace.com
43. Weight of a body on the surface of the earth depends on
(a) distance of the body from the centre of the earth
(b) the latitude of the place on the earth surface where the body is placed
(c) the longitude of the place on the earth surface where the body is placed
(d) the angular speed of rotation of the earth about its own axis
Ans. [a,b.d]
Sol. Theory based
44. Which of the following is /are involved in the formation of rain drops in a cloud ?
(a) saturation of vapour pressure (b) temperature
(c) viscosity (d) surface tension
Ans. [a,b,d]
Sol. Theory based

45. A cyclic process on p-V diagram is as shown below. The same process can be shown on p-T or V-T diagrams.
Choose the correct alternative /s.
p 1 2

4
V
p V
1 2 4 3

(a) 3 (b) 2
4 T 1 T

p V
4 3 1 2

(c) 2 (d) 3
1
T 4 T
Ans. [a, b]

P
1 2 Isothermal

3
Sol. 4
V
Isothermal

www.examrace.com
P
1 2

3
4 T

V
4 3

2
1 T

46. When a bright light source is placed 30 cm in front of a thin lens, an erect image is formed at 7.5 cm from the
lens. A faint inverted image is also formed at 6 cm in front of the lens due to reflection from the front surface
of the lens. When the lens is turned around, this weaker inverted image is now formed at 10 cm in front of
the lens. Therefore,
(a) the lens is diverging biconcave
(b) the refractive index of the glass of the lens is 1.6
(c) radii of curvature of surfaces of the lens are 10 cm and 15 cm respectively.
(d) the lens behaves as a converging lens of focal length 30 cm when immersed in a liquid of refractive index 2
Ans. [a,b,c,d]

Sol. Incident zone – + Refraction zone

1 1 1
= –
f v u

1 1 1
= –
f – 7.5 – 30

1 1 1
= +
f – 7.5 30

1 –1
=
f 10
f = – 10
lens is diverting as f is in refraction zone

www.examrace.com
Image due to reflect
R1

+ –
O

30

1 1 1
= +
f u v

1 1 1
= +
f 30 6

1 1
=
f 5
f=–5
R1
=–5
2
R1 = 10 cm
R2

30
1 1 1
= +
f 30 10

1 4
=
f 30

30
f=
4
R2 30
=
2 4
R2 = 15 cm

– +

1 n – n1  1 1 
= 2  – 
f n1  R1 R2 

www.examrace.com
1 n –1  1 1
= 2  – 10 – 15 
– 10 1  
1  – (3  2) 
– = n2 – 1  
10  30 
1 (n – 1)
– = 2 ×5
10 30
3
= n2 –1
5
n2 = 1.6
1 1.6 – 2  1 1
=  – – 
f 2  10 15 
1 – 0.4  – 5 
=
f 2  30 

60
f= = 30 cm
2

47. Let n1 and n2 moles of two different ideal gases be mixed. If ratio of specific heats of the two gases are +1 and
+2 respectively, then the ratio of specific heats + of the mixture is given through the relation
( n  n2 ) n1 n2
(a) (n1 + n2)+ = n1+1 + n2+2 (b) 1 = +
+ 1 +1  1 +2 1
+ + +
(c) (n1 + n2) = n1 1 + n2 2 (d) (n1 + n2)(+ – 1) = n1(+1 – 1) + n2(+2 – 1)
+ 1 +1  1 +2 1
Ans. [b,c]
n1+1 n +
n1C p  n2 C p 2  2 2
+ 1 +2 1
Sol. += 1
= 1
n1CV  n2 CV 2 n1 n
1  2
+1  1 + 2  1

48. A resistance of 4 ohm is connected across a cell. Then it is replaced by another resistance of 1 ohm. It is
found that power dissipated in resistance in both the cases is 16 watt. Then,
(a) internal resistance of the cell is 2 ohm
(b) emf of the cell is 12 volt
(c) maximum power that can be dissipated in the external resistance is 18 watt
(d) short circuit current from the cell is infinite
Ans. [a,b,c]
E r
2
E
Sol.
 × 4 = 16
4 r 
4

E r
2
E

 × 1 = 16
1 r 
1

www.examrace.com
4 1
2
=
(4  r ) (1  r ) 2
4 r
2=
1 r
2 + 2r = 4 + r
r = 2 (a)
2
E

 × 4 = 16
4 2
E
=2
6
E = 12 (b)
Max power  when R = r
2
E E2
& Pmax =
 ×R=
2R  4r
12  12
= = 18 watt. (c)
4 2
E 12
Short circuit current = = = 6 Amp
r 2
& Ans (a, b, c)

49. Two solid spheres A and B of equal volumes but of different densities dA and dB respectively, are connected
by a string. They are fully immersed in a fluid of density dF. They get arranged in an equilibrium state as
shown in the figure with the tension in the string. The arrangement is possible only if

(a) dA < dF (b) dB > dF (c) dA > dF (d) (dA + dB) = 2dF
Ans. [a,b,d]
VdFg

T VdAg
Sol.
VdFg

VdBg

For equilibrium of entire system 2VdFg = VdAg + VdBg


2dF = dA + dB
To keep the string having tension, dF > dA & dB > dF

www.examrace.com
50. A particle Q is moving along + Y axis. Another particle P is moving in XY plane along a straight line
x = –d(d > 0) with a uniform speed v parallel to that of Q. At time t = 0, particles P and Q happen to be along
X axis whereas a third particle R situated at x = + d starts moving opposite to P with a constant acceleration
a. At all further instants the three particles happen to be collinear. Then Q
v
(a) has an initial speed
2
v
(b) will come to rest after a time interval
a

a
(c) has an acceleration –
2
2v
(d) will return to its initial position after a time interval
a
Ans. [a,b,c,d]
y 2 – y1
Sol. y – y1 = ( x  x1 )
x2  x1
1 2
 at  vt
y – vt = 2 (x  d )
2d
on differentiating for R (x = 0)
 at  v  at  v
v, – v = – (d )  v, – v = ....(1)
2d 2
0v v
(i) at t = 0 ; v, – v =  v, =
2 2
 at  v v
(ii) for v, = 0; 0 – v = t=
2 a
a a
(iii) diff (1) a, – 0 = –  a, = –
2 2
2v
(iv) will return to its position after time
a

www.examrace.com
PART B MARKS : 60
All question are compulsory. All question carry equal marks.

1. (a) A 40 watt, 120 volt incandescent bulb has a tungsten filaments 0.381 m long. The diameter of the
filament is 33 μm. Tungsten has a resistivity 5.51 × 10 8 ohm-m at room temperature (20ºC). Given that the
resistivity of the tungsten filament varies as T6/5, estimate the temperature of the filament when it is operated
at its rated voltage.
(b) Assume that the electrical power dissipated in the filament is radiated from the surface of the filament. If
emissivity of the filament surface is 0.35, determine the temperature of the filament and compare it with that
obtained in part (a)
(120) 2
Sol. (a) Resistance at rating = = 360 
40
" 5.51 10 8  0.381
Initial resistance = = = 24.55 
A (16.5  10  6 ) 2
L
R=
A
RA 360
= =  (3.3  10 6 ) 2
L 0.381 4
 = 8.08 × 10 7 ohm-m
  T6/5
6/5
 T
=


0 T0 
6/5
8.08  10 7 I
8
=

5.5  10 293 
T = 2764 K
(b) e-./0  01
40
T4 = 8
= 51 × 1012
0.35  5.67  10  2 (16.5  10  6 )(0.381)
T = 2672 kelvin

2. A parallel plate capacitor with a separation d = 1.0 cm is subjected to a potential difference of 20 kV with air
as a dielectric. Assume that air behaves as a dielectric (insulator) upto a maximum electric field (called
dielectric strength) of 30 kV / cm (after which is breaks down). Now, a thin plate of glass (dielectric constant
K = 6.5 and dielectric strength = 290 kV/ cm) is inserted. Determine the maximum thickness of glass plate to
avoid breakdown in the capacitor.
Sol.

t V2
V1
k=1
k=6.5

d –t t
20 kV

www.examrace.com
V1 C
= 2 V1 + V2 = 20 kV
V2 C1

C2 C1
V1 =  20kV V2 =  20kV
C1  C 2 C1  C 2
V1 V
E1 = E2 = 2
dt t
V1 V2
2 30kV 2 290kV
dt t
# A
C1 = 0
dt
6.5# 0 A
C2 =
t
C 2 6 .5
  (d  t )
C1 t
1 t  20 20t
V2 =  20 = =
C2 t  6.5 (d  t ) 6.5 d  5.5t
1
C1
1  20 1  20 6.5(d  t )  20
V1 = = =
C1 t 6.5d  5.5t
1 1
C2 6 .5 ( d  t )
V2
2 290
t
20
2 290
6.5d  5.5t

2 < 188.5d – 159.5t


159.5t < 186.5
186.5
t<
159.5
13 < 19.5d –16.5t
16.5t < 19.5 –13
6.5
t<
16.5
t < 0.394
0.394 cm Ans.

3
3. (a) Cauchy's empirical formula for refractive index of a transparent medium is n – 1 = A
1  2 
" 
Hence obtain the condition for achromatic combination of two lenses made from different glasses. Refractive
indices of flint glass and crown glass are given below.
Red light " = 640 nm Blue light " = 480 nm
Flint glass 1.644 1.664
Crown glass 1.514 1.524

(b) Determine the focal length of the two lenses (one of flint glass and the other of crown glass) such that
their combination has focal length of + 40 cm for all colours.

www.examrace.com
1 µ 1  1 1 
Sol. =   
f 1  R1 R2 
1 1 
loge1 – logef = log (μ – 1) + log   
 R1 R2 
 df dµ
=
f µ 1
 df
=   dispersive power of material
f
1 1 1
= +
f eq f1 f2
As feq should not change with wavelength for achromatic combination
df eq.
& 0
d"
df df
0 = – 21  22
f1 f2
1 
+ 2 =0
f1 f2
µv  µR 1.664  1.644 0.02
1 = = = = 0.03058
vµ  µ R 1 .664  1. 644 0 .654

 1
 1
2  2 
1.524  1.514 0.01
2 = = = 0.01927
1.524  1.514 0.519
1
2
1 1 1
= + ....(1)
40 f1 f2
0.03058 0.01927
+ =0 .....(2)
f1 f2
 f1 = –1.58692f2
1 1 1
 = +
40 1.58692 f 2 f2
1 1 1
 =
1  
40 f 2 1.58692 
 f2 = 40 × 0.36985 = 14.794 cm
f1 = –23.47689 cm

4. Obtain an expression for the magnetic moment associated with a solenoid of length L and number of turns N
carrying I. The inner and outer radii of the solenoid are r1 and r2.
x

Area
Sol.
dx

N
dM = dx  i  A
"

www.examrace.com
NiA
M=  " = NiA
"
Magnetic moment of solenoid of area A = NiA

dr
r

N  dr
No. of turns in element =
r2  r1
r2
IN
 r
2
Magnetic moment = dr
r2  r1
r1

I  N   r23  r13 
M=  
r2  r1  3 

5. One end of a string is attached to a rigid wall at point O, passes over a smooth pulley and carries a hanger S
of mass M at its other end. Another objects P of mass M is suspended from a light ring that can slide without
friction, along the string, as shown is figure. OA is horizontal. Find the additional mass to be attached to the
hanger S so as to raise the object P by 10 cm.
O cm A
40 3

Sol.
40 3

P 20 3 •

 
T
T
Q
M M
initially
2T cos  = Mg
2(Mg) cos  = Mg
cos  = 1/2
 = 60°

www.examrace.com
20 3
tan 60 =
PQ
PQ = 20 cm
P, 20 3

10
 1300
Q,
M
M+m
Now P,Q, = 20 – 10 = 10 cm
2T cos  = Mg
10
2(M + m)g × = Mg
1300
1
2(M + m) × =M
13
2M + 2m = 13 M
2m = M( 13 – 2)
M ( 13  2)
m= = 0.9 M
2

Physical constants you may need –


19
1. Charge on electron e = 1.6 × 10 C
31
2. Mass of an electron me = 9.1 × 10 kg
11
3. Universal gravitational constant G = 6.67 × 10 N m2/kg2
12
4. Permittivity of free space #0 = 8.85 × 10 C2 / N m2
5. Gas constant R = 8.31 J/ K mol
34
6. Planck constant h = 6.62 × 10 Js
7. Stefan constant - = 5.67 × 10 W/ m2 K4
8

23
8. Boltzmann constant k = 1.38 × 10 J/K

www.examrace.com
INDIAN ASSOCIATION OF PHYSICS TEACHERS
NATIONAL STANDARD EXAMINATION IN PHYSICS 2012-2013
Date of Examination : 24th November 2012
Time 09.30 to 11.30 hrs
Total time : 120 minutes (A-1, A-2 & B)
[Q.P. CODE NO. : 1-1-6]
PART-A (Total Marks : 180)
SUB-PART A-1 : ONLY ONE OUT OF FOUR OPTIONS IS CORRECT

N.B. Physical constants are given at the end


SUB-PART A-1
1. Two thermally insulated compartments 1 and 2 are filled with a perfect gas and are connected by a short tube
having a valve which is closed. The pressures, volumes and absolute temperatures of the two compartments
are respectively (p1, V1, T1) and (p2, V2, T2). After opening the valve, the temperature and the pressure of
both the compartments respectively are -
T1T2 (p1V1  p 2 V2 ) p1V1  p 2 V2 p1V1  p 2 V2
(a) , (b) T1T2 ,
(p1V1T1  p 2 V2 T2 ) V1  V2 V1  V2

T1T2 (p1V1  p 2 V2 ) p1V1T1  p 2 V2T2 T1  T2 p1V1  p 2 V2


(c) , (d) ,
(p1V1T1  p 2 V2 T2 ) V1T1  V2T2 2 V1  V2
Ans. [Bonus]
Sol. *Internal energy before connection
C V p1V1 C V p 2 V2 C V p'
 [V1 + V2]
R R R
p1V1  p 2 V2
p'
V1  V2

n1 + n2 = n1'  n '2
*Law of mole conservation
p1V1 p 2 V2 p' V1  p' V2

RT1 RT2 RT ' RT'

T1T2 (p1V1  p 2 V2 )
?T'=
(p1V1T2  p 2 V2 T1 )
Comment : No option is matching so question is bonus.

www.examrace.com
2. An inductance coil is connected to an ac source through a 60 ohm resistance in series. The source voltage,
voltage across the coil and voltage across the resistance are found to be 33 V, 27 V and 12 respectively.
Therefore, the resistance of the coil is -
(a) 30 ohm (b) 45 ohm (c) 105 ohm (d) 75 ohm
Ans. [b]
Sol.
L,R 60:

27V 12V

~
VS = 33
12 1
I=
60 5

VS2 VL2  (VR  V60 ) 2

VS2 VL2  VR2  2VR V60  V60


2

(33)2 = (27)2 + 2VR(12) + (12)2


1089 = 729 + 24VR + 144
24VR = 216
216
VR = =9
24
IR = 9
9 9
R= = 45:
I 1/ 5

3. An ideal inductance coil is connected to a parallel plate capacitor. Electrical oscillation with energy W are set
up in this circuit. The capacitor plates are slowly drawn apart till the frequency of oscillations is doubled. The
work done in this process will be -
(a) W (b) 2W (c) 3W (d) 4W
Ans. [c]
1
Sol. f=
2S LC
1
frequency is doubled when capacitance is th
4

Q2
energy Ÿ U1 = W
2C

www.examrace.com
Q2
energy after shifting U2 = = 4W
C
2
4
Work done = U2 – U1 = 4W – W = 3W

4. Two equal masses are connected by a spring satisfying Hooke's law and are placed on a frictionless table.
The spring is elongated a little and allowed to go. Let the angular frequency of oscillations be Z. Now one of
the masses is stopped. The square of the new angular frequency is -
Z2 Z2
(a) Z2 (b) (c) (d) 2Z2
2 3
Ans. [b]
Sol.

m M

m
P=
2
k 2k
Z2 = =
P m
When one block is stopped
k Z2
Z12 = =
m 2

5. When a particle oscillates in simple harmonic motion, both its potential energy and kinetic energy vary
sinusoidally with time. If Qbe the frequency of the motion of the particle, the frequency associated with the
kinetic energy is -
Q
(a) 4Q (b) 2Q (c) Q (d)
2
Ans. [b]
Sol. x = A sin Zt
frequency is Q
v = ZA cos Zt
1 1 1 1 § 1  cos 2Zt · 1
KE = mv2 = mZ2A2cos2Zt = KA2 cos2 Zt = KA2 ¨ ¸ KA 2 (1 + cos 2Zt)
2 2 2 2 © 2 ¹ 4

so, the frequency of energy is doubled o 2Q

www.examrace.com
6. A gas expands from i to f along the three paths indicated. The work done along the three paths denoted by
W1, W2 and W3 have the relationship -
P
4 i

3 2 3
1
2
1 f

V
1 2 3 4 5 6
(a) W1 < W2 < W3 (b) W2 < W1 = W3 (c) W2 < W1 < W3 (d) W1 > W2 > W3
Ans. [a]

Sol. We know W = ³ P.dV area under PV curve

(Area)3 > (Area)2 > (Area)1


? W3 > W2 > W1

7. An ideal gas at 30ºC enclosed in cylinder with perfectly non conducting side and a piston moving without
friction in it. The base of the cylinder is perfectly conducting. Cylinder is first placed on a heat source till the
gas is heated to 100ºC and the piston raised by 20 cm and the atmospheric pressure is 100 kPa. The piston is
then held in final position and cylinder is placed on the heat sink to cool the gas to 30ºC. Denoting 'Q1 as the
heat supplied during heating and 'Q2 as the heat lost during the cooling then ['Q1 ~'Q2] would be equal to
(a) 436 J (b) 336 J (c) 236 J (d) 136 J
Ans. [Bonus]
Sol. Comment : Either cross sectional area should be given or number of moles of gas should be given in this
questions ? question is bonus.
'Q1 = nCp'T – nCv'T
Ÿ n(Cp – Cv)'T
Ÿ nR'T = PdV = P(Adx)
n n
Not given Not given

8. Equal amount liquid helium and water at their respective boiling points are boiled by supplying the heat from
identical heaters in time tHe and tw. The latent heats of vaporization of He and Water are 2.09 × 104 J /kg and
540 kcal/kg, then tHe is -
(a) about 0.1 tw (b) about 0.05 tw
(c) just greater than 0.01 tw (d) just less than 0.01 tw
Ans. [d]

www.examrace.com
Sol. PHe = Pwater
m He .L He m w .L water
t He t water

2.09 u 10 4 540 u 103 u 4.2


=
t He t water

t water 54 u 4.2
Ÿ = 108.51
t He 2.09

t He 1
or
t water 108

? tHe = just less than (0.01) tw




9. A 5 litre vessel contains 2 mole of oxygen gas at a pressure of 8 atm. The average translational kinetic energy
of an oxygen molecule under this condition is -
14 21
(a) 8.4 × 10 J (b) 4.98 × 10 J
16 21
(c) 7.4 × 10 J (d) 4.2 × 10 J
Ans. [b]
f 3
Sol. E= KT = KT
2 2

PV 8 u 105 u 5 u 103
T =
PR 2 u 8.31

3 23 4 u 10 2 u 5
= × 1.38 × 10 ×
2 8.31

6 u 1.38 u 10 23 u 102 u 5 21


= | 5.06 × 10 J
8.31

10. Six identical conducting rods are joined as shown. The ends A and D are maintained at 200ºC and 20ºC
respectively. No heat is lost to surroundings. The temperature of the junction C will be -

A B C D
200ºC 20ºC

(a) 60ºC (b) 80ºC (c) 100ºC (d) 120ºC


Ans. [b]

www.examrace.com
Sol.

R R
A R B C R D
200ºC 20ºC
R R

A R D
200ºC

B R •
C R 20ºC

(R  R )20  R.200
TC =
3R
40  200
= = 80ºC
3

11. Three corners of an equilateral triangle of side 'a' are occupied by three charges of magnitudes q. If the
1
charges are transferred to infinity, their kinetic energy will be times -
4SH0

2q 3q 2 q2 3q
(a) (b) (c) (d)
a a 3a a
Ans. [b]
Sol.
q

a a

q q
a
Kqq Kqq Kqq
Potential energy =  
a a a

3Kq 2 1 § 3q 2 ·
= ¨ ¸
a 4SH0 ¨© a ¸¹

= kinetic energy
12. An LDR (light dependent resistance) is connected to an appropriate voltage source and a current measuring
meter in series (Assuming that the LDR current is proportional to the intensity of the incident light). The
LDR is illuminated with light from a distant metal filament bulb. The filament voltage V, the distance d of
LDR from the bulb and the LDR current I are noted. If both V and d are doubled, the LDR current is -
(a) 16 I (b) 4 I (c) I (d) less than I

www.examrace.com
Ans. [c]
Sol. Current is directly proportional to intensity
Power V2 / R
Intensity =
Area 4Sd 2
V2
Intensity v
d2

13. A point source is placed at a distance of 20 cm from a convex lens of focal length f on its axis and the image
is formed on a screen at a distance of 60 cm from the lens. Now the lens is split into two halves. One half is
moved perpendicular to the lens axis through a distance of 5 cm. It is found that the two halves of the lens
form two images on the screen and the images are separated by a distance d. The values of f and d
respectively, are -
(a) 20 cm, 15 cm (b) 20 cm, 10 cm (c) 30 cm, 10 cm (d) 30 cm, 5 cm
Ans. [a]
Sol.
f

O

30cm 60cm
1 1 1

f v u
1 1 1

f 60  30
f = 20 cm

(2) I2
10
5cm 5
O • I1
(1)
30cm
I2 = m × O
v 60
Ÿ × O Ÿ ×5
u 30
Ÿ 10
Separation between I1 and I2 = 10 + 5
= 15

www.examrace.com
14. The angle of refraction of a very thin prism is 1º. A light ray is incident normally on one of the refracting
surfaces. The ray that ultimately emerges from the first surface, after suffering reflection from the second
surface, makes an angle of 3.32º with the normal. The deviation of the ray emerging from the second surface
and the refractive index of the material of the prism respectively, are -
(a) 0.66º, 1.66 (b) 1.66º, 1.5 (c) 1.5º, 1.66 (d) 0.66º, 1.5
Ans. [a]
Sol. r1 + r2 = A
here r1 = 0
so r2 = A = i
r1= 0 i
r1= 0
90º
r2= i
r2= i
P 88º
3.32º r3
μ
so angle of incidence of the light at first surface when emerging from it will be
r3 = 2º
bring snells law at point P.
μ sin2º = sin 3.32º
for small angle sin T | Tso
μ × 2º = 3.32
3.32
μ= = 1.66
2
The deviation of the ray emerging from the second surface
' = A (μ –1) = 1º(1.66 – 1) = 0.66º

15. A beam of light from a distant axial point source is incident on the plane surface of a thin plano-convex lens;
a real image is formed at a distance of 40 cm. Now if the curved surface is silvered, the real image is formed
at a distance of 7.5 cm. The radius of curvature of the curved surface of the lens and the refractive index of
the material of the lens respectively, are -
(a) 40 cm, 1.5 (b) 24 cm, 1.6 (c) 20 cm, 1.6 (d) 7.5 cm, 1.5
Ans. [b]
Sol.

f = 40 cm

www.examrace.com
P 1 1
= (less maker formula)
R f
P 1 1
= ....(i)
R 40
1

Ÿ
2

Now system behave as a mirror


1 2 1
= +
f eq f1 f2

2 1
= + ×2
40 R
1 1 2
= +
f eq 20 R

I
feq

feq = 7.5
1 1 2
= +
7.5 20 R
2
0.08 = ; R = 24
R
P 1 1
=
R 40
R
P–1=
40
R
P=1+ Ÿ 1.6
40

16. A convex lens forms the image of an axial point on a screen. A second lens with focal length f cm is placed
between the screen and the first lens at a distance of 10 cm from the screen. To view the image the screen has
to be shifted away from the lens by 5 cm. A third lens having focal length of the same magnitude f cm is used
to replace the second lens at the same position. But this time to view the image the screen has to be shifted
towards the lens by d cm. The values of f and d respectively, are -
(a) 30 cm, 2.5 cm (b) 30 cm, 5 cm (c) 7.5 cm, 2.5 cm (d) 7.5 cm, 5 cm

www.examrace.com
Ans. [a]
Sol.

O I

1 2

O – + I1 I2

f
10 cm 5 cm
1 1 1
= –
f v u
1 1 1
= –
f 15 10
1 5
=
f 150
f = – 30 lens is diverging lens
lens of f Ÿ 30

O – + I1

Portion of I2 f
10 cm

1 1 1
= –
30 v 10
1 1 1
= +
v 30 10
30
v= Ÿ 7.5 cm
4
Screen is to shift by Ÿ 10 – 7.5 cm Ÿ 2.5 cm

www.examrace.com
17. Cerenkov effect : If the speed of an electron in a medium is greater than the speed of light in that medium
then the electron emits light. An electron beam in a medium is accelerated by a voltage V. The light that is
emitted just suffers total internal reflection at the boundary of the medium placed in air when the angle of
incidence is 45º. The value of the voltage is -
(a) 63.91 kV (b) 255.64 kV (c) 200.34 kV (d) 127.82 kV
Ans. [d]
Sol. For just total internal reflection
i = TC
45 = TC ....(i)
.E. of e Ÿe × V
1
eV = mv2
2

2eV
Q=
m
c
v>
P

2eV c
>
m P
2
§c· m
V > ¨¨ ¸¸ × ....(ii)
P
© ¹ 2 e

From (i); sin 45 = sin TC


1 1
=
2 P

1 1
= …(iii)
P 2
2

from (ii) and (iii)


(3.0 u 108 ) 2 9.1 u 10 31
V> ×
2 2 u 1.6 u 10 19
12.81 × 10000 V
Ÿ 127.8 kV approx

18. In an electrolytic process certain amount of charge liberates 0.8 gram of oxygen. Then the amount of silver
liberated by same amount of charge is -
(a) 10.8 gram (b) 1.08 gram (c) 0.9 gram (d) 9.0 gram
Ans. [a]

www.examrace.com
Sol. According to Faraday's second law
Q1 = Q2
M1 E1
?
M2 E2

0.8 8
M2 108
M2 = 10.8 gm

19. The energy state of doubly ionized lithium having the same energy as that of the first excited state of
hydrogen is -
(a) 4 (b) 6 (c) 3 (d) 2
Ans. [b]
Sol. Li++ have Z = 3
13.6Z 2 13.6
– 2
=– 2
n 2
13.6 u 32 13.6
2
= 2
n 2
n2 = 9 × 4
n=6

20. The logic circuit shown below is equivalent to -

Z
X
Y
X X
(a) Z (b) Z
Y Y
(c) X Z (d) X Z
Ans. [d]
Sol.

x
x
xy x  xy
x
y x (1  y) x
x .y
y
so it is equivalent to
X Z

www.examrace.com
21. In the circuit shown below, the potential of A with respect to B of the capacitor C is -
1.0V 20:

0.5V A B
C

2.5V 10:
(a) 2.00 volt (b) – 2.00 volt (c) – 1.50 volt (d) + 1.50 volt
Ans. [c]
Sol.
1.0V 20:
i
0.5V +Q –Q
A B

2.5V 10: i

Q
VA – VB =
C
Using Kirchoff
–1 + 20 × i + 10 × i + 2.5 = 0
30i = –1.5
1.5 1
i= Ÿ  Amp.
300 20
Q
–0.5 + –20i + 1 = 0
C
Q
–0.5 + +1+1=0
C
Q
= –1.5
C

22. Two pendulums differ in lengths by 22 cm. They oscillate at the same place so that one of them makes
30 oscillations and the other makes 36 oscillations during the same time. The lengths (in cm) of the
pendulums are -
(a) 72 and 50 (b) 60 and 38 (c) 50 and 28 (d) 80 and 58
Ans. [a]
T0 "1
Sol. T1 = 2S
30 g

www.examrace.com
T0 "2
T2 = 2S
36 g

T1 36 "1
T2 30 "2

6 "1
Ÿ
5 "2
36 "
= 1
25 "2
11 "1  " 2 22cm
25 "2 "2
"2 = 50 cm
"1 = 72 cm

23. The voltage drop across a forward biased diode is 0.7 volt. In the following circuit, the voltages across the
10 ohm resistance in series with the diode and 20 ohm resistance are -

10:

20:

+ – 10:
10V
(a) 0.70 V, 4.28 V (b) 3.58 V, 4.28 V (c) 5.35 V, 2.14 V (d) 3.58 V, 9.3 V
Ans. [b]
Sol.
0.7V
10:
i–x
(1)
x 20:
(2)
i 10V 10:

Kirchoff in loop (1)


0.7 + 10(i – x) – x × 20 = 0
0.7 + 10i – 30x = 0 ….(1)
Kirchoff in loop (2)
x × 20 + 10 × i – 10 = 0
10i + 20x – 10 = 0 ….(2)
–10 – 0.7 + 50x = 0
x = 0.214 Amp

www.examrace.com
Voltage across 20 : = 4.28 V
10 i + 20 × 0.214 – 10 = 0
i = 0.572
i – x = 0.358 Amp
voltage across 10 : = (i – x) × 10 Ÿ 3.58 volt

24. The magnetic flux I through a stationary loop of wire having a resistance R varies with time as I = at2 + bt
(a and b are positive constants). The average emf and the total charge flowing in the loop in the time interval
t = 0 to t = W respectively are -
aW 2  bW aW 2  bW aW  b aW 2  bW aW 2  bW
(a) a W+ b, (b) a W+ b, (c) , (d) 2(a W + b),
R 2R 2 R 2R
Ans. [a]
change in flux
Sol. Average emf =
time
I Ÿ at2 + bt
At t = 0, I = 0
At t=W
I = aW2 + bW
'I = aW2 + bW

aW 2  bW
Average emf = = aW + b
W
dI
e= Ÿ 2at + b
dt
2at  b
i=
R
dq 2at  b
=
dt R
W
(2at  b) aW 2  bW
q= ³
0
R
dt Ÿ
R

25. Three waves of the same amplitude have frequencies (n – 1), n and (n + 1) Hz. They superpose on one
another to produce beats. The number of beats produced per second is -
(a) n (b) 2 (c) 1 (d) 3n
Ans. [b]
Sol. Beats produced is max difference between two wave frequency.
(1) and (2) (n) – n – 1 = 1
(2) and (3) (n + 1) – n = 1
(1) and (3) (n + 1) – (n – 1) = 2

www.examrace.com
26. A spherical ball of mass m1 collides head on with another ball of mass m2 at rest. The collision is elastic. The
fraction of kinetic energy lost by m1 is -
4m1m 2 m1 m2 m1m 2
(a) (b) (c) (d)
(m1  m 2 ) 2 m1  m 2 m1  m 2 (m1  m 2 ) 2
Ans. [a]
Sol.

m1 u m2 Ÿ m1 v1 m2 v2
u2 = 0

by linear momentum
m1u + 0 = m1v1 + m2v2
For head on elastic collision
§ m  m2 · 2m 2 u 2
v1 = ¨¨ 1 ¸¸ u1 + …(i)
m
© 1  m 2 ¹ ( m1  m2 )

(K.E) f  (K.E) I
For fractional K.E. =
(K.E) I

1
(K.E)i of m1 = m1u2 …(ii)
2
2
1 1 § m  m2 · 2
(K.E)f = m1u12 = m1 ¨¨ 1 ¸¸ u
2 2 © m1  m 2 ¹

§ m  m2 ·
Put (u2 = 0) in equation (i), v1 = ¨¨ 1 ¸¸ u
© m1  m 2 ¹
2
1 1 § m  m2 · 2
m1u 2  m1 ¨¨ 1 ¸ .u
'K.E 2 2 © m1  m 2 ¸¹
=
K.E 1
m1u 2
2
'K.E 4m1m 2
=
K.E ( m1  m 2 ) 2
27. In the circuit shown below, the switch is in position 1 for a long time. At some moment after that the switch
is thrown in position 2. The quantity of heat generated in the resistance of 375 ohm after the switch is
changed to position 2 is

8 PF 250:
2
1
375:

250V
(a) 0.15 J (b) 0.25 J (c) 0.50 J (d) 0.10 J

www.examrace.com
Ans. [a]
1
Sol. Ustored = × 8 × (250)2
2
When switch is thrown to 2
capacitor start discharging
? whole stored energy get discharged in form of heat in two resistances which are connected in series

8 PF

i H2 250:

H1

375:
Heat Ÿ H = Ustored
H1 + H2 = H
H1 375
H 2 250
375
H1 = × Ustored Ÿ 0.15 Joule
625

28. A conducting square frame of side a and a long straight wire carrying current I and located in the same plane
as shown in the figure. The frame moves to the right with a constant velocity v. The emf induced in the frame
will be proportional to
x
I

1 1 1 1
(a) (b) (c) (d)
x 2
(2 x  a ) 2
(2 x  a ) 2 (2 x  a )(2x  a )
Ans. [d]
Sol.
x
I A E

e1 … v
B
D F
a

www.examrace.com
a u P 0iv
emf in AD Ÿ e1 Ÿ
§ a·
2S¨ x  ¸
© 2¹
a u P 0i u v
emf in EF Ÿ e2 Ÿ
§ a·
2S¨ x  ¸
© 2¹
Net emf = e1 – e2
ª º
aP 0iv « 1 1 »
Ÿ «  »
2S « x  a x  a »
¬ 2 2¼
kua
e=
(2 x  a )(2x  a )

29. In the circuit shown below, the switch S is closed at the moment t = 0. As a result the voltage across the
capacitor C will change with time as -

S
+
100V R C

R

VC VC
100 100

(a) 50 (b) 50

0 t 0 t
VC VC
100 100

(c) 50 (d) 50

0 t 0 t
Ans. [c]
Sol.
S

R C
100V
R

Find potential across C is VC

www.examrace.com
i
V1 VC

i
V1 = VC
100
V1 = u R Ÿ 50
2R
? VC = 50 volt
 ? Initially C is neutral so graph should start from origin and finally reach upto 50V.

30. The ratio of the rotational kinetic energy to the total kinetic energy of one mole of a gas of rigid diatomic
molecules is
2 2 3 5
(a) (b) (c) (d)
3 5 5 2
Ans. [b]
2
Sol. R.K.E. = nRT = nRT = RT
2
Rotational DOF = 2
? Total DOF is = 5
5
Total K.E. = RT
2
R.K.E. RT 2
Total K.E. 5 5
RT
2

31. A metal cylinder of length L is subjected to a uniform compressive force F as shown in the figure. The
material of the cylinder has Young's modulus Y and Poisson's ratio μ. The change in volume of the cylinder
is -
F

μFL (1  μ)FL (1  2μ)FL (1  2μ)FL


(a) (b) (c) (d)
Y Y Y Y
Ans. [d]

www.examrace.com
Sol. V = Sr2L
dV 2dr dL

V r L
'L  'd
μ
L d
'd 'L
P
d L
dV § 2dr 'L ·
¨  ¸
V © r L ¹
§ P'L 'L ·
= ¨ 2  ¸
© L L ¹
'L
= (–2P + 1)
L
F.AL
dV = (–2P + 1)
AY
(1  2P)FL
=
Y

32. Three persons A, B and C note the time taken by their train to cover the distance between two successive
stations by observing the digital clocks on the platforms of two stations. The clocks display time in hours and
minutes. The three persons find the intervals 3, 5 and 4 minutes respectively. Assume the maximum
discrepancy of 2 seconds in actual starting and stopping of the train and the observations by A, B and C.
Then,
(a) All A, B and C can be correct.
(b) Only A and B or B and C can be correct
(c) Only one of A, B and C can be correct
(d) C is correct since it is equal to the average of the three observations
Ans. [c]

33. When two drops of water coalesce – (I) Total surface area decreases. (II) There is some rise in temperature.
Which of the following is correct ?
(a) Both (I) and (II) are wrong statements
(b) Statement (I) is true but (II) is not true
(c) Both (I) and (II) are true and the two statements are independent of each other
(d) Both (I) and (II) are true and (I) is the cause of (II)
Ans. [d]
Sol.  when coalesce then
surface area decreases and due to which internal energy increases that's why temperature increases
So, (I) is the cause of (II) that's why correct option is (d).

www.examrace.com
34. Two capacitors 0.5 μF and 1.0 μF in series are connected to a dc source of 30 V. The voltages across the
capacitors respectively are -
(a) 10 V, 20 volt (b) 15 V, 15 volt (c) 20 V, 10 volt (d) 30 V, 30 volt
Ans. [c]
Sol.
0.5 1

V1 V2

30V

1
V1 = u 30
0.5  1
30
= = 20 volt
1.5
0.5
V2 = × 30
1. 5
= 10 volt

232 208
35. The Th 90 atom has successive alpha and beta decays to the end product Pb 82 . The numbers of alpha and
beta particles emitted in the process respectively are -
(a) 4, 6 (b) 4, 4 (c) 6, 2 (d) 6, 4
Ans. [d]
Sol. Mass number get charge by Ÿ 4 × n
because atomic number charge occur only in D decay
4n = 232 –
4n = 24
n=6
DŸ4
Mass number get charge by Ÿ 24
New atomic number Ÿ 90 – 6 × 2 + P
90 – 12 + P = 82
P = 94 – 90 = 4
E Ÿ 4

36. If the breakdown field of air is 2.0 × 106 V / m, the maximum charge that can be given to a sphere of
diameter 10 cm is
(a) 2.0 × 10 4 C (b) 5.6 × 10 7 C (c) 5.6 × 10 5 C (d) 2.0 × 102 C
Ans. [b]

www.examrace.com
Q
Sol. E=
4SH0 R 2

Q max u 4
2.0 × 106 =
4SH0 (0.1) 2

9 u 109 Q max u 4
2 × 106 =
0.01

1 106
= u = Qmax
18 1011
5
= 0.0555 × 10
7
= 5.6 × 10

37. Density of ocean water varies with depth. This is due to -


(a) elasticity (b) viscosity (c) surface tension (d) all of three
Ans. [a]
Sol. ? density vary with property of elasticity.

38. A spring of certain length and having spring constant k is cut two pieces of lengths in a ratio 1 : 2. The spring
constants of the two pieces are in a ratio -
(a) 1 : 1 (b) 1. : 4 (c) 1 : 2 (d) none of the above
Ans. [d]
Sol.
Ÿ
"0 "0 2" 0
3 3

"0 § 2" ·
k"0 = k1 = k2 ¨ 0 ¸
3 © 3 ¹
3
k1 = 3k Ÿk2 = k
2
k1 2
=
k2 1

39. When a metal surface is illuminated with light of wavelength O, the stopping potential is V0. When the same
V0
surface is illuminated with light of wavelength 2O, the stopping potential is . If the velocity of light in air
4
is c, the threshold frequency of photoelectric emission is -
c c 2c 4c
(a) (b) (c) (d)
6O 3O 3O 3O

www.examrace.com
Ans. [b]
hc
Sol. eV0 = –I …..(1)
O
eV0 hc
= –I ….(2)
4 2O
1 ª hc º hc
«  I» = –I
4¬ O ¼ 2O

hc 2hc
I  4I
O O
hc
3I =
O
hc
3 × hfth =
O
c
fth = 
3O

40. Two elastic wave move along the same direction in the same medium. The pressure amplitudes of both the
waves are equal, but the wavelength of the first wave is three times that of the second. If the average power
transmitted through unit area by the first wave is W1 and that by the second is W2, then -
(a) W1 = W2 (b) W1 = 3W2 (c) W2 = 3W1 (d) W1 = 9W2
Ans. [a]
Sol. we know

P02
I=
2UV
in a given medium
V = constant
U = constant
P0 = constant (given)
P
Ÿ ?I =
A
?W1 = W2

www.examrace.com
SUB-PART A-2
In question 41 to 50 any number of options (1 or 2 or 3 or all 4) may be correct. You are to identify all of them
correctly to get 6 marks. Even if one answer identified is incorrect or one correct answer is missed, you get
zero score.

41. A cube floats both in water and in a liquid of specific gravity 0.8. Therefore,
(a) apparent weight of the cube is the same in water and in the liquid
(b) the cube has displaced equal volume of water and the liquid while floating
(c) the cube has displaced equal weight of water and the liquid while floating
(d) if some weights are placed on the top surface of the cube to make it just sink, the load in case of water
will be 0.8 times of that to be used in case of the liquid.
Ans. [a, c ]
Sol. (a) cube is floating in water and in liquid so apparent weight of cube in both will be zero.
(c) both liquid and water will apply same thrust on the cube is equal to its weight.

42. On the basis of the kinetic theory of gases one compares 1 gram of hydrogen with 1 gram of argon both at
0ºC. Then,
(a) the same temperature implies that the average kinetic energy of the molecules is the same in both the
cases
(b) the same temperature implies that the average potential energy of the molecules is the same in both the
cases.
(c) internal energies in both the cases are equal.
(d) when both the samples are heated through 1ºC, the total energy added to both of them is not the same.
Ans. [b,d]
Sol.  P.E. of ideal gas is zero
for all type of gases ooption (b) is correct
m
n = which is different for different gases
M
option (d) is also correct.
So choice b & d are correct.

43. While explaining the action of heat engine, one can say that
(a) heat cannot be fully converted into mechanical work.
(b) the first law of thermodynamics is necessary but not sufficient.
(c) heat under no circumstances can flow from lower to higher temperature.
(d) A body can not be cooled to absolute zero
Ans. [a,b,d]

www.examrace.com
Sol. Option (a) is correct  100 % efficiency not possible.
Option (b) is correct :  first law thermodynamics is used and second law is also used.
TL
Option (d) is correct K = 1 –
TH
if TL = 0K
? K= 1 or 100 % not possible.

44. The rate of change of angular momentum of a system of particles about the centre of mass is equal to the sum
of external torques about the centre of mass when the centre of mass is
(a) fixed with respect to an inertial frame (b) in linear acceleration
(c) in rotational motion (d) is in a translational motion
Ans. [a,b,c,d]

45. Light is traveling in vacuum along the Z-axis. The sets of possible electric and magnetic fields could be
& & & & & &
(a) E îE 0 sin(Zt – kz), B ĵB0 sin(Zt – kz) (b) E îE 0 sin(Zt – kz), B ĵB0 cos(Zt – kz)
& & & & & &
(c) E ˆjE 0 sin(Zt – kz), B  îB 0 sin (Zt – kz) (d) E îE 0 sin(Zt – kz), B ĵB0 sin (Zt – kz + G)

Ans. [a,c]
& &
Sol. * Direction of EM wave is in E u B
& &
? E u B should be in k̂ direction.
* E & B oscillate in same phase.
? Correct option are a & c.

46. In case of photoelectric effect,


(a) since photons are absorbed as a single unit, there is no significant time delay in the emission of
photoelectrons.
eI
(b) Einstein's analysis gives a critical frequency Q0 = , where I is the work function and the light of this
h
frequency ejects electrons with maximum kinetic energy.
(c) only a small fraction of the incident photons succeed in ejecting photoelectrons while most of them are
absorbed by the system as a whole and generate thermal energy.
(d) the maximum kinetic energy of the electrons is dependent on the intensity of radiation.
Ans. [a, c]

www.examrace.com
47. A parallel combination of an inductor coil and a resistance of 60 ohm is connected to an ac source. The
current in the coil, current in the resistance and the source current are respectively 3A, 2.5 A and 4.5 A.
Therefore,
(a) Kirchhoff's current law is NOT applicable to ac circuits
(b) impedance of the coil is 50 ohm
(c) electric power dissipated in the coil is 150 watt
(d) impedance of the circuit is 33.3 ohm
Ans. [b, d]
Sol.
3A L

2.5A R

4.5A
~
Vrms
2.5
VR rms Ÿ 60 u = 150 volt
10
Vrms = VR rms = 150 V

Vrms 150 150 100


Z= Ÿ Ÿ u 10 = = 33.3 :?Ans (d)
i rms 4.5 4.5 3

VL rms = 150 = irms × XL

150
XL = = 50 : ?Ans. (b)
3
So correct option are (b) & (d)

48. The nuclear forces


(a) are stronger being roughly hundred times that of electromagnetic forces
(b) have a short range dominant over a distance of about a few fermi
(c) are central forces independent of the spin of the nucleons.
(d) are independent of the nuclear charge.
Ans. [a, b, d]

www.examrace.com
49. Consider a mole of a sample of hydrogen gas at NTP.
(a) The volume of the gas is exactly 2.24 × 10 2 m3.
(b) The volume of the gas is approximately 2.24 × 10 2 m3.
(c) The gas will be in thermal equilibrium with 1 mole of oxygen gas at NTP.
(d) The gas will be in thermodynamic equilibrium with 1 mole of oxygen at NTP.
Ans. [b,c,d]
Sol. At NTP for 1 mole of gas
Volume contain is 22.4 litre
or 22.4 × 10 3 m3
or 2.24 × 10 2 m3
choice (a) is correct.
o in thermal equilibrium
PV
PV = nRT Ÿ n = n,V & P = constant, then (T = constant)
RT
So choices a & c are correct and it is same valid therefore choice (b).is also correct.

50. A particle moves in one dimension in a conservative force field. The potential energy is depicted in the graph
below.
V(x)

Potential
energy

x
A B C D E
If the particle starts to move from rest from the point A, then
(a) the speed is zero at the points A and E. (b) the acceleration vanishes at the points A, B, C, D, E
(c) the acceleration vanishes at the points B, C, D (d) the speed is maximum at the point D
Ans. [a, c]
dV
Sol. F=
dx
dV
= 0 at B, C, D
dx
?F = 0 at B, C, D
so a = 0 at B, C, D
speed is maximum at B as potential energy at B is minimum.
? (Ans. a, c)

www.examrace.com
PART B MARKS : 60
All questions are compulsory. All questions carry equal marks.

1. (a) A conductor having resistance R (independent of temperature) and thermal capacity C is initially at
temperature T0 same as that of the surrounding. At time t = 0 it is connected to a source with constant
voltage V. The thermal power dissipated by the conductor to the surrounding varies as q = k(T – T0).
C
Determine the temperature T of the conductor at any time t and at the time t = .
k
(b) A particle moves rectilinearly in an electric field E = E0 – ax where a is a positive constant and x is the
q
distance from the point where the particle is initially at rest. Let the particle have a specific charge .
m
Find (I) the distance covered by the particle till the moment at which it once again comes to rest, and
(II) acceleration of the particle at this moment.
Sol. At time t temperature of conductor is T.
Rate of heat loss to surrounding = k(T – T0)
V2
Rate of heat generation =
R

dQ V2
Ÿ Rate of heat gain by conductor = – k(T – T0)
dt R
dQ dT
= ms
dt dt
dT
=C
dt

V2 dT
– k(T – T0) = C
R dt
V2 – kR(T – T0) = RCdT
t T
1 dT
³
0
RC
dt = ³V
T0
2
 kR (T  T0 )

t 1 V 2  kR (T  T0 )
=– "n
RC kR V2

kt V 2  kR (T  T0 )
= –"n
C V2
kt

2 2 C
V – kR(T – T0) = V e
kt

kR(T – T0) = V2[1 – e C ]

www.examrace.com
kt
V2 
(T – T0) = [1 – e C ]
kR
kt
V2 
T = T0 + [1 – e C ]
kR
C
At t =
k

V2
T = T0 + [1 – e 1]
kR

V2
T = T0 + × 0.63
kR
(b) F = q(E0 – ax)
q
a= (E0 – ax)
m
x =0

qE 0
at x = 0 a=
m
Particle will start moving in the x direction
dv
v =a
dx
dv q
v = (E0 – ax)
dx m
0 x0
q
³
0
vdv = ³ m (E
0
0  ax ) dx

q §¨ x 02 ·
¸
0= E x  a
m ¨© 2 ¸¹
0 0

x 02
E0x0 = a
2
2E 0
x0 =
a
2E 0
Distance covered Ÿ
a
before coming at rest
q§ 2E ·
a= ¨ E0  a u 0 ¸
m© a ¹
qE 0
Ÿ – toward negative x-axis.
m

www.examrace.com
2. One mole of an ideal gas (J = 1.4) with initial pressure of 2 atm and temperature of 57ºC is taken to twice its
volume through different processes that include isothermal, isobaric and adiabatic processes. Determine the
process where maximum work is done and the amount of work in this case. By what percentage is this work
larger than the work done in a process in which it is the least ?
Sol.

Isobaric

P n Isothermal

Adiabatic
V V o 2V
Area under isobaric process is
maximum & minimum in adiabatic process.
ª 8.31 u 330 º
o (W)isobaric = P 'V = 2 × 105 « »
¬ 2 u 105 ¼
= 2742.3 J
nR'T nR[T1  T2 ] R[T1  T2 ] 8.31[330  250]
Ÿ(W)adiabatic = = = 1659 J
( J  1) J 1 (1.4  1) 0.4
(1.4 1) 0.4
ªV º ª1º
[ T1V1 (J 1)
= T2 V2 (J 1)
Ÿ T2 = T1 . « 1 » = 330 × « » = 250.14ºC]
¬ V2 ¼ ¬2¼
( W ) isobaric  ( W ) adibatic
Ÿ So the % is 100 × 65.29% 
( W ) adiabatic

3. A railway carriage of mass MC filled with sand of mass Ms moves along the rails. The carriage is given an
impulse and it starts with a velocity v0. At the same time it is observed that the sand starts leaking through a
hole at the bottom of the carriage at a constant mass rate O. Find the distance at which the carriage becomes
empty and the velocity attained by the carriage at that time. (Neglect the friction along the rails.)
Sol. From the law of conservation of linear momentum, linear speed of carriage will not change with time
because sand's momentum does not change after leaking through the hole so final velocity of the carriage
MS
will be 'v0' and constant with time. Time taken by the carriage to become empty =
O
v0MS
So distance covered by the carriage in this time = speed × time =
O

www.examrace.com
4. Show that, for any angle of incidence on a prism
1 1
sin (A  G) cos (r1  r2 )
2 =μ 2
1 1
sin (A ) cos (i  e)
2 2
(symbols have usual meanings)
and that the right-hand side reduces to at minimum deviation.
Sol.

i
μ1 = 1 e
r1 r2
μ3 = 1
μ2 = μ

r1 + r2 = A .....(1)
1 sin i = μ sin r1 .....(2)
μ sin r2 = 1 sin e .....(3)
G = i + e – A
dG de
=1+ –0=0
di di
de
= 0 Ÿ |e| = |i|
di
* sin i = sin e = μ sin r1 = μ sin r2
r1 = r2
* r1 = r2 = A/2
G min  A
* Gmin = 2i – A = 2e – A Ÿ i =
2

§G A·
sin ¨ min ¸
* μ=
sin i
= © 2 ¹
sin r1 sin(A / 2)
* In a given expression
§ A  G min · § 1 ·
sin ¨ ¸ ¨ cos (r1  r2 ) ¸
© 2 ¹ =μ ¨ 2 ¸
sin(A / 2) ¨ cos 1 (i  e) ¸
¨ ¸
© 2 ¹
R.H.S Side : - Put r1 = r2 and i = e then it becomes μ.
§ A  G min ·
sin ¨ ¸
Hence : - © 2 ¹ = μ
sin(A / 2)

www.examrace.com
5. (a) A small amount of solution containing Na24 nuclides with activity 20500 disintegrations per second was
injected in the blood stream of a person. The activity of 1 ml of blood sample taken after 5 hours later,
was found to be 20 disintegration per minute. The half life of the radioactive nuclides is 15 hours. Find
the total volume of the blood of this person.
(b) The wire loop shown in the figure lies in uniform magnetic induction B = B0cos Zt perpendicular to its
plane. (Given r1 = 10 cm and r2 = 20 cm, B0 = 20mT and Z = 100 S). Find the amplitude of the current
induced in the loop if its resistance is 0.1 :/m.

r1 r2


Sol. (a) Volume of blood of person = V
Na24 is uniformly mixed in V volume of blood.
Activity of Na24 v total no. of nuclei
20500 20500
A Ÿ Activity of Na24 of 1 ml sample = × 1 ml =
V ml V

A
After 5 hr activity =
2n
1
n = as half live is 15 hrs.
3
A
=
21/ 3
20 A
= 1/ 3
60 2
21/ 3
A=
3

20500 21/ 3
=
V 3
V = 48812.58 ml
(b)

r1 r2

B = B0 cos Zt

www.examrace.com
dB
= – B0Z sin Zt
dt

§ dB ·
¨ ¸ = B0Z
© dt ¹ max

dI dB § dB ·
e1 = – = –A1 = Sr12 ¨ ¸ = Sr12 B0Z
dt dt © dt ¹ max

dI dB § dB ·
e2 = – = –A2 = – Sr22 ¨ ¸ = Sr22 B0Z
dt dt © dt ¹ max
induced emf is opposite to each other

enet = e2 – e1 = SB0Z (r22  r12 )

Res = 2S(r1 + r2)O

e net SB0 Z(r22  r12 ) B Z(r  r )


I= = = 0 2 1
R es 2S(r1  r2 )O 2O

20 u 10 3 u 100S(0.2  0.1)
I= = S ampere
2 u 0.1
= 3.14 ampere
*****
Physical constant you may need ....
1. Charge on electron e = 1.6 × 10 19C
31
2. Mass of electron me = 9.1 × 10 kg
11
3. Universal gravitational constant G = 6.67 × 10 Nm2/kg2
12
4. Permittivity of free space H0 = 8.85 × 10 C2/Nm2
5. Gas constant R = 8.31 J/K mol
34
6. Planck constant h = 6.62 × 10 Js
7. Stefan constant V = 5.67 × 10 8 W/m2 K4
23
8. Boltzman constant k = 1.38 × 10 J/K

www.examrace.com
INDIAN ASSOCIATION OF PHYSICS TEACHERS
NATIONAL STANDARD EXAMINATION IN PHYSICS 2013-2014
Date of Examination : 24th November 2013
Time 9.30 to 11.30 Hrs

1 3 7
Q. P. Code No.
ONE THREE SEVEN
INSTRUCTION TO CANDIDATES
1. In addition to this question paper, you are given answer sheet for part A and an answer paper
for part B.
2. On the answer sheet for part A, fill up all the entries carefully in the space provided, ONLY In
BLOCK CAPITALS.
Incomplete / incorrect / carelessly filled information may disqualify your candidature.
3. On part A answer sheet, use only BLUE of BLACK BALL PEN for making entries and marking
answer.
4. Part A has two parts. In part A1 (Q.No. 1 to 40) each question has Four alternatives, out of
which only one is correct. Choose the correct alternative and mark a cross (X) in the
corresponding box on the answer sheet.

For example, Q. a b c d

22

Part A2 (Q.Nos. 41 to 50) has four alternatives in each question, but any number of these (4, 3,
2, or 1) may be correct. You have to mark ALL correct alternatives and mark a cross (X) for
each, like

5. For Part A1, each correct answer gets 3 marks; wrong answer gets a penalty of 1 mark. For
Part A2 full marks are 6 for each question, you get them when ALL correct answer only are
marked.
6. Any rough work should be done only on the sheets provided with part B answer paper.
7. Use of nonprogrammable calculator is allowed.
8. No candidate should leave the examination hall before the completion of the examination.
9. After submitting your answer papers, read the instructions regarding evaluation given on next
page.
10. Attempt the examination honestly. Any dishonestly will disqualify you.
PLEASE DO NOT MAKE ANY MARKS OTHER THAN (X) IN THE SPACE PROIDED ON
THE ANSWER SHEET OF PART A.
Answer sheets are evaluated with the help of a machine. Hence, CHANGE OF ENTRY IS NOT
ALLOWED.
www.examrace.com
Scratching or overwriting may result in wrong score.
DO NOT WRITE ANYTHING ON THE BACK SIDE OF PART A ANSWER SHEET
INSTRUCTIONS TO CANDIDATES:
1. The answers / solutions to this question paper will be available on our website-
www.iapt.org.in by 3 rd Decembet 2013.

EVALUATION PROCEDURE (NSEP) :


2. Part 'A' Answers of ALL the candidates are examined.

3. Part B is evaluated of only those students who get marks above a certain' "cut oft" marks
in Part A. (e.g. NSEP Total marks for Part A are 180. Students getting (say) 100 or more
than 100 marks In Part A are identified and their Part B is evaluated. Thus "cut off" marks
are 100 in this example.)

4. PART ‘B’ IS NOT EVALUATED OF ALL THE CANDIDATES.

CERTIFICATES & AWARDS


Following certificates are awarded by the I.A.P.T. to students successful in NSEP.
i) Certificate for “Centre Top 10%” students on the basis of marks in part A only.
ii) Merit certificates to statewise Top 1% students on the basis of (A+B) marks.
iii) Merit certificate and a prize in the form of a book to Nationwise Top 1% students
based on (A+B) marks.

5. Result sheets and the “centre top 10%” certificates of NSEP are dispatched to the
Professor in charge of the centre. Thus you will get your marks from the Professor in
charge of your centre by January 2014 end.

6. TOP 300 (or so) students are called for the next examination -Indian National Physics
Olympiads (INPhO). Individual letters are sent to these students ONLY.

7. Gold medals will be awarded to TOP 35 students in this entire process.

8. No queries will be entertained in this regard.

www.examrace.com
NSEP-2013-14

INDIAN ASSOCIATION OF PHYSICS TEACHERS

NATIONAL STANDARD EXAMINATION IN PHYSICS 2013-2014


Total Time : 120 Minutes (A-1, A-2 & B) Code - 137

PART A MARKS = 180

N.B. : Physical constants are given at the end

SUB-PART A-1

(ONLY ONE OUT OF FOUR OPTIONS IS CORRECT )

 7
1. A certain quantity of oxygen     is compressed isothermally until is pressure is doubled (P2). The gas
 5
is then allowed to expand adiabatically until is original volume is restored. Then the final pressure (P3) in
terms of initial pressure (P1) is :
(a) P3 = 0.55 P1 (b) P3 = 0.76 P1 (c) P3 = 0.68 P1 (d) P3 = 2.55 P1
Sol. (b)
P0V0 = (2P0) V’ (isothermal process)

V
 2P0   = P ( V ) 
V0
V’ =
2 2 3 0

2 P0 V0
= P31
2
7
1
2 5

P3 = 2
P0
25

P0 P0 P0
P3 = = 2 5 =
2 0.4 (2 ) ( 4)1/ 5

P0
P3 = = 0.757 P0 .
( 4)1/ 5

2. A car fitted with a device which transmits sound 60 times per minute. There is no wind and speed of sound
in still air is 345 m/s. If you hear the sound 68 times per minute when you are moving towards the car with
a speed of 12 m/s, the speed of the car must be nearly.
(a) 20.0 m/s towards you (b) 30.0 m/s towards you
(c) 10.0 m/s away from you (d) 10.0 m/s towards you
Sol. (b)
C = 345 m/s fob = 68 time/1min

www.examrace.com
NSEP-2013-14
C  12
f’ = C  V f0
2

68 345  12
= 345  V
60 2

60
345 – V2 = 357 ×
68
357  60
V2 = 345 – = 345 – 315 = 30 m/s
68

3. A 43 m long rope of mass 5.0 kg joins two rock climbers. One climber strikes the rope and the second one
feels the effect 1.4 s later. What is the tension in the rope ?
(a) 110 N (b) 301 N (c) 215 N (d) 154 N
Sol. (a)
43
Vwave = × m/s
7

T
Vwave =

 T  ( Vwave )2 
2
 43  5  5
=   × ~ 109.69 N
 7  43

4. Two cats in a house mew at each other with sound intensities 5 × 10–9 Wm–2 and 9 × 10–6 Wm–2. By how
many decides is the louder sound above the other ?
(a) ~13 dB (b) ~23 dB (c) ~33 dB (d) ~43 dB
Sol. (c)
  
d = 10 log   
 0

 
 2   2  10 log  2 
 1 

 9  10 6 
 
= 10 log  9 
 5  10 

9
= 30 log   = 30 log (1.8)
5

5. An electron orbiting around a nucleus has angular momentum L. The magnetic field produced by the
electron at the centre of the orbit can be expressed as :
(a) B  ( 0 e / 8mr 3 ) L (b) B  ( 0 e / 4mr 3 ) L

(c) B  ( 0 e / mr 3 ) L (d) B  (e / 40mr 3 ) L


Sol. (b)
eV
B 0 r 2
= L=mVr
L 4 mVr

B  0 eV 1 0 L  0eL
=   B= =
L 4 r 2 mVr 4 r 4mr 3 www.examrace.com
NSEP-2013-14
6. Suppose a radioactive nucleus A disintegrates at origin into a nuclesus B with the emission of a positron
(e+) and a neutrino (v) such that the positron and the neutrino move at right angles to each other and carry
momentum 2 × 10–22 kg ms–1 (+Y-axis) and 5 × 10–23 kg ms–1 (+X-axis) respectively. Then the nucleus.
(a) A recoils with the momentum 2.86 × 10–22 kg ms–1 at angle 14.03º w.r.t. +X-axis
(b) A recoils with the momentum 2.06 × 10–22 kg ms–1 at angle 14.03º w.r.t. –X-axis
(c) B recoils with the momentum 2.86 × 10–22 kg ms–1 at angle 14.03º w.r.t. +X-axis
(d) B recoils with the momentum 2.06 × 10–22 kg ms–1 at angle 14.03º w.r.t. –X-axis
Sol. (d)

5
PP = 5 × 10–23 kg m/s = × 10–22
10
Pn = 2 × 10–22 kg m/s
2
 1
PB = ( 2) 2     10  22
2

17
=  10 22 = 2.06 × 10–22
2
PP
tan  = P = 4
n

  tan 1 4

7. Two identical strings with fixed ends separated by height h have their other ends tied to a body P of mass m
as shown in figure. When the body rotates with uniform angular speed 2 2g / h in a horizontal plane about
the vertical axis the ratio of tensions (T1/ T2) in the string is :

3 5 2 5
(a) (b) (c) (d)
5 3 5 2

www.examrace.com
NSEP-2013-14
Sol. (b)
T1 cos   mg  T2 cos 
(T1 – T2) cos   mg
T1 sin   T2 sin  = m 2 r

(T1  T2 ) sin  m2r



(T1  T2 ) cos  mg

T1  T2  2r
tan  
T1  T2 g

T1  T2 2r h
tan    tan 
T1  T2 g 2

T1  T2 h 4  2g
 
T1  T2 2g h
T1 + T2 = 4T1 – 4T2
T1 5
5T2 = 3T1  
T2 3

8. Two identical bodies M2 and M3 each of 4.0 kg are tied to a massless inextensible string which is made to
pass around pulleys P1 and P2 as shown in figure. Angle ABC = 37º. The coefficient of kinetic friction
between the bodies and the surface on which they slide is 0.25. If the body M1 moves done with uniform
speed, neglecting the masses and friction of pulleys. M1 =

(a) 36.8 kg (b) 9.8 kg (c) 4.2 kg (d) 2.1 kg


Sol. (c)

Acceleration = 0
m1g = m2g + sin 37 + m 2 g cos 37  m3 g

3 1 4 1
= 4 × 10     
5 4 5 4 

 12 4 5 
= 40    
 20 20 20 
m = 4.2 kg www.examrace.com
NSEP-2013-14

9. A particle of mass 0.2 kg moves along a path given by the relation : r  2 cos t î  3 sin t ĵ . Then the
torque on the particle about the origin is :
2 3
(a) 13 k̂ Nm (b) k̂ Nm (c) k̂ Nm (d) 0 k̂
3 2
Sol. (d)
 
 
L = m r V

V =  2 sin t î  3 cos t ĵ

î ĵ k̂
2 cos t 3 sin t 
0  R̂ 6 cos 2 t  6 sin 2 t 
 2 sin t 3 cos t 0  6 k̂ cons tan t
L=0

10. A bead of mass 5.0 g can move without friction on a piece of wire bent in the form of a semicircular ring of
radius 0.10 m, as shown in the adjacent figure. This ring can freely rotate about the vertical axis OY. At what
height will the bead stay above the ground level OX, if this semicircular are revolves with angular velocity
10.63 rad/s ?

(a) 0.013 m (b) 0.087 m (c) 0.027 m (d) 0.073 m


Sol. (a)
N sin   mr2
N cos   mg

r2
tan  
g

0.1sin (10.63 )2
tan  
10

1 (10.63 )2
=
cos  100
2
 10 
cos     = 0.885
 10.63 
h = 0.1 [1 – 0.885] = 0.01

www.examrace.com
NSEP-2013-14
11. One of the flat surfaces of a cylinder (radius r and length l) and the flat surface of hemisphere are
cemented together. The cylinder and the hemisphere are made of the same material. The combined mass
of the system is M. The moment of inertia of this system about an axis coinciding with the axis of cylinder is:
(a) (1/10) M {5r2 + 4l2 } (b) (1/10) Mr2 {(15l + 8r) / (3l + 2r)}
2
(c) (1/10) Mr {(3l + 4r) / (3l + 2r)} (d) (1/10) M {(5r3 l + 4l2r2) / (5r + 4l)}
Sol. (b)

M1r 2 2
  M2r 2 ............(i)
2 5
M1 3 
Now,  and M1 + M2 = M
M2 2 r

2rM1
M1  M
3

 3  2r 
M1  M
 3 

 3  M   2r 
M1    , M2 =  3  2r M .
 3   2r   
From equation (i)
M1r 2 2
  M2r 2
2 5

r 2 3 M 2 2r
=  r 2. M
2 3  2r 5 (3  2r )

2 3 4r 
 = Mr   
 2( 3   2r ) 5(3   2r ) 

2  15  4r 
 = Mr  
 10(3  2r ) 

12. The radius of cross section of a long pipe varies gradually as r  r0 e  x , where x is the distance from the
pipe inlet and a = 0.4 m–1 is a constant. Then the ratio of Reynolds number for the cross sections separated
by a distance 8.0 m is :
(a) 24.5 (b) 28.5 (c) 2.45 (d) 2.85
Sol. (a)
Vr  Vr 2 AV
R=  
2 2 r 2r
here AV = volume flow rate.
 AV  1
R1 =  
 2  

 AV   1 
R2 =   2
 2   r 

R 2 r1 r0 e – ( 0 )
 
R1 r2 r0 e – ( 8 )

R2
8 32
www.examrace.com
R1 = e = e = 24.5
NSEP-2013-14
13. The Pitot tube shown in the figure is used to measure fluid flow velocity in a pipe of cross sectional area S.
It was invented by a French engineer Henri Pitot in the early 18th century. The volume of the gas flowing
across the section of the pipe per unit time is (The difference in the liquid columns is h, 0 and  are the
densities of liquid and the gas respectively)

h 0 g 2h 0 g hg0 2hg


(a) Q  2S (b) Q  S (c) Q  S (d) Q  S 0
  
Sol. (b)
1 2
v  p  0 gh
2

2 0gh
v

2h 0 g
QS

14. A thin ring has a radius R, density  and Young’s modulus Y. The ring is rotated in its own plane about an
axis passing through its centre with angular velocity  . Then the small increase in its radius is :
2R 3 32R 3 62R 3 2R 3
(a) dR  (b) dR  (c) dR  (d) dR 
Y Y Y 2Y
Sol. (a)

d
2T. sin  dm2R
2
 T.d = Ard 2R
 T = AR22
dR T AR 3 2 2R 3
 =  dR = = .
R AY AY Y

www.examrace.com
NSEP-2013-14
15. A uniform metal wire is clamped by chuck nuts at the two ends as shown in the adjacent figure. The wire
has cross section area A, length  and density  . A weight W is suspended from the midpoint of the wire.
Then the vertical displacement  , through which the midpoint moves down is given by :

1/ 2 1/ 3 1/ 2 1/ 3
 W   W   W   W 
(a)     (b)     (c)     (d)    
2  AY  2  AY  3  AY  4  AY 
Sol. (b)

L
T  YA
(L / 2)

YA L
T . (cos ec  1)
L/2 2
and 2T cos = 
 2YA (cosec – 1).cos = 

cos   8 8 
  cos  =   
sin  2 YA L/2 L 2 2YA
 82
4

 1/ 2 
8   482    8 28 2 
1 1 2 
 L / 2   L   =
2 YA

L/2 L2 2YA
 
1/ 3
 L  
 8 = 3
 8=   .
8 YA 2  YA 

16. Two bodies of mass M1 and M2 are kept separated by a distance d. The potential at the point where the
gravitational field produced by them is zero, is :

(a) 
G
d

M1  M2  2 M1M2  (b) 
G
d

M1M2  2 M1  M2 
(c) 
G
d

M1  M 2  2 M1M2  (d) 
G
d

M1M2  2 M1  M2 
Sol. (a)

GM1 GM2 d– x M2
  
x2 (d – x )2 x M1

d M2 M1 d
  1  x= www.examrace.com
x M1 M1  M2
NSEP-2013-14

 Ugrav =

– G M1  M2 M 1


– G M1  M2 M 2
d d

= –
G
d

M1  M1M2  M1  M2  M2 
= –
G
d

M1  M2  2 M1M2 
17. A ball dropped on ground from a height of 1.00 m rises to a height of 75 cm on rebounce. When thrown
down from the same height with a velocity of 2.0 m/s, it would rise to (assume g = 10 m/s2)
(a) 80 cm (b) 90 cm (c) 85 cm (d) 95 cm
Sol. (b)

3 
2g h 
4  = 3 1
e mgh = mV2
2gh 2 2

V= 2gh

V2 = eV1 V12 = (2)2 + 2g (1) = 4 + 20 = 24


3
= × 24
2
V2 = 18 = 2gh'

18
 = h’
2  10
h11 = 0.9 m

18. A metallic body of material with density of 8000 kg/m3 has a cavity inside. A spring balance shown its mass
to be 10.0 kg in air and 7.5 kg when immersed in water. The ratio of the volume of the cavity to the volume
of the material of the body must be
(a) 2/5 (b) 1/2 (c) 1 (d) 3/4
Sol. (c)
(V – VC) 8000 = 100
(V – VC) = (8000 – 1000)V = 7.5,
1000 V = 2.5
 V  VC 
  8= 4
 V  1

 V  VC 
  = 1 2V – 2VC = V
 V  2
V = 2VC
V
VC =
2

V
VC
2 1
V  VC =  V
V  
 2
www.examrace.com
NSEP-2013-14
19. In a steel factory it is found that to maintain M kg of iron in the molten state at its melting point an input power
P watt is required. When the power source is turned off, the sample completely solidifies in time t second.
The latent heat of fusion of iron is
2Pt Pt Pt PM
(a) (b) (c) (d)
M 2M M t
Sol. (c)
Pt
Pt = ML  L=
M

20. A LASER source of heat of power 1.2 W is placed very close to one end of a rod of cross section area
3 cm2 and thermal conductivity 400 W/mK. The length of the rod (L) required to maintain a temperature
difference of 10ºC across its ends is (assume that all power emitted by the source falls on the rod)
(a) 1.5 m (b) 2.2 m (c) 1.8 m (d) 1 m
Sol. (d)
P = 1.2w

d  d 
 kA  
dt  x 

10
1.2 = 400 × (3 × 10–4) ×
2

400  3  10 –3
L=  10
12
= 103 × 10–3 = 1 m

21. The temperature at which average de Broglie wavelength of helium atom becomes 0.5 nm is
(a) 6.6 K (b) 7.1 K (c) 279.6 K (d) 280.1 K
Sol. (a)
h 3
 and E0  kT
2mE0 2
Approximate : T = 6.6 K.

22. A dielectric slab is introduced between the plates of a capacitor. If the charge on the capacitor is q and the
magnitude of the induced charge on the dielectric surface is q’, then
(a) q’ < q (always) (b) q’ > q (always) (c) q’ = q (always) (d) q’ = 0
Sol. (a)

23. When two ends of a spring are pulled apart increasing its length it produces force equal to kx at its ends. At
a point 1/4 of its length from one end the force is
(a) 0.25 kx (b) 0.75 kx (c) 1.0 kx (d) 0.5 kx
Sol. (c)

www.examrace.com
NSEP-2013-14
24. Two semicircular wires of radius 20 cm and 10 cm have a common centre at the origin O as shown in the
figure. Assume that both the wires are uniformly charged and have an equal charge of 0.70 nC each. The
magnitude of electric field at the common centre of curvature O of the system is

(a) 100 V/m (b) 301 V/m (c) 401 V/m (d) 501 V/m
Sol. (b)

 Q Q 
ENet = 2K  2  
 r R 2 

2KQ  1 1 
=   2
 r 2
R 

2  9  10 9  0.7  10 9
= 3.14  75

= 300.96 V/m

25. An electron has its path unchanged when it is passing through a region of electric field (E = 3.4 × 104 V/m)
and a magnetic field (B = 2 × 10–3 Wb/m2) both perpendicular to each other. However, on switching off the
electric field, the electrons move along a circular path. What is the radius of circular path ?
(a) 4.82 m (b) 4.82 × 10–3 m (c) 4.82 × 10–2 m (d) None of the above
Sol. (c)
E = VB
3.4 × 104 = V × 2 × 10–3
V = 1.7 × 107
mV 9.1 10 31  1.7  10 7
R= R =
qB 1.6  10 19  2  10 3
= 4.83 × 10–2 m

26. A 1000 F capacitor fully charged to 250 V discharges through a resistance wire embedded in a thermally
insulated block of specific heat 2.5 × 102 J kg–1K–1 and mass of 0.01 kg. How much is the increases in the
temperature of the block?
(a) 12.5 K (b) 8.5 K (c) 7.0 K (d) 15.5 K
Sol. (a)
1
Ui = × 10–3 × (250)2
2
1
(25 )2  10 –1
 = 2 25
2 –2
  12.5k
2.5  10  10 2
www.examrace.com
NSEP-2013-14
27. Two bodies A and B hanging in air are tied to the ends of a string which passes over a frictionless pulley.
The masses of the string and the pulley are negligible and the masses of two bodies are 2 kg and 3 kg
respectively. (Assume g = 10 m/s2). Body A moves upwards under a force equal to
(a) 30 N (b) 24 N (c) 10 N (d) 4 N
Sol. (d)
3 – 2
a=  g  2cm – 2
 3  2 
Fnet A = T – 2g = 2 × a = 4N

28. An object is placed at a distance of 10 cm from a co-axial combination of two lenses A and B. The
combination forms a real image three times the size of the object. If lens B is concave with a focal length
30 cm, then focal length of lens A is
(a) 10 cm (b) 7.5 cm (c) 6 cm (d) –6 cm
Sol. (c)

29. Two concave refracting surfaces of equal radii of curvature face each other in air as shown in figure. A
point object O is placed midway between the centre and one of the poles. Then the separation between the
images of O formed by each refracting surface is

(a) 11.4 R (b) 1.14 R (c) 0.114 R (d) 0.0114 R


Sol. (c)

3
–1
3 1
–  2
event - 1 21  – 3R  –R
 
 2 

3 2 –1 3 –1 –2 –7 –9R
       V1 =
21 3R 2R 21 2R 3R 6R 7

3
–1
 1
–  2  2 –1  –1 2 –5
event - 2 2v 2  R  –R      – 
–  2v 2 R 2R 2v 2 2R R 2R
 2 

R
v2 = –
5

www.examrace.com
NSEP-2013-14
30. A ray of white light falls on an isosceles prism at such an angle that the red light leaves the prism
perpendicular to the other face of the prism. Find angle of deviation if the refractive index of the prism for
red light is 1.37 and refracting angle of prism is 45º
(a) 20º37’ (b) 28º37’ (c) 35º37’ (d) 30º37’
Sol. (d)

sin i =  sin r
 1 
sin i = (1.37)  
 2
sin i = 0.97
i = sin–1 (0.97)
 = sin–1 (0.97) – 45º
= 30.371

31. The voltage between the terminals of a battery is 6.00 V. When a wire is connected across its terminals it
falls to 5.6 V. If one more identical wire is connected between the terminals then it will fall to
(a) 4.80 V (b) 5.15 V (c) 5.25 V (d) 5.80 V
Sol. (c)
– V
r=  R
 V 
Let after connecting the identical wire between terminals potential dropis V
 6 – 5 .6  6 – V R
  R     V = 5.25 Volt
 6   V 2

32. Water is siphoned out from a tank at a higher level into another of identical size 2.0 m below. The length of
the siphon tube is 4.0 m and each of its ends is below the water surface by 10 cm. In the upper tank the
water level is at 1.00 m and in the lower one it is at 50 cm from the bases of the respective tanks. Water
through siphon rises in 40 cm of the length of the tube to a level which is 20 cm higher than the water level
in the upper tank before it begins to flow down. Assume the values of atmospheric pressure and accelera-
tion due to gravity to be 103.4 kPa and 10 m/s2. The pressure at the cross section of the tube at the highest
point is
(a) 101.9 kPa (b) 105.4 kPa (c) 107.4 kPa (d) 109.1 kPa
Sol. (a)

33. The impedance of the RL circuit given in the adjacent figure is expressed by the relation Z2 = A2 + B2. Then
the dimensions of AB are

(a) [M1L2I–2T–3] (b) [M2L4I–4T–6] (c) [M1L–1I–2T–3] (d) [M–1L–2I2T4]

www.examrace.com
NSEP-2013-14
Sol. (b)
2
V
[ AB ]   

2
 M1L2 T – 2  2 4 –6 –4
=  2 1   [M L T  ]
  T 

34. A micrometer screw gauge with pitch of 0.5 mm and 50 divisions on circular scale is used to measure the
diameter of a thin wire. Initially when the gap is closed the line of fourth division coincides with the
reference line. Three readings show 46th , 48th and 44th division coinciding with the reference line which is
beyond 0.5 mm of the main scale. The (best) measured value is
(a) 0.46 mm (b) 0.94 mm (c) 0.92 mm (d) 1.00 mm
Sol. (c)
Zero error = 4 × 
0.5
Reading = 0.5 mm + 48 ×
50
= 0.5 mm + 0.48 mm
= 098 mm
Actual value = 0.98 mm – 4 = 0.92 mm.

35. In a meter bridge experiment the resistance to be measured is connected in the right gap and a known
resistance in the left gap has value of 50  0.2  when the null point is judged to be at 60  0.2 cm. The
students notes that the ends of the bridge wire are not at 0.0 cm and 100.0 cm of the scale and makes a
guess that they may be somewhere within 0.2 cm beyond the scale ends. The value of the unknown
resistance should be expressed as
(a) 33.33 ± 1 (b) 75 ± 1  (c) 75.0 ± 0.9  (d) 33.4 ± 0.5 
Sol. (d)
40  50 100

60 3
|  1 |   2  R
=  1

1

R

 0.2 0.2 0.2 


x  x    
 60 40 50 

 100 
=   0.4   .
 3 

36. A body A revolves along a circular orbit close to the earth’s surface. Body B oscillates along an imaginary
straight tunnel drilled through the earth, whereas another body C through a similar longest tunnel. Let TA , TB
and TC be the corresponding periods of revolution or oscillation, then
(a) TA > TB = TC (b) TA > TC > TB (c) TA = TB = TC (d) TA < TB = TC
Sol. (c)
R3 / 2 R
TA =  2
GM g

R
TB = 2 g

R
TC = 2 g

www.examrace.com
NSEP-2013-14
37. The vibrations of a string of length 100 cm and fixed at both ends are represented by the equation :
y = 2 sin ( x / 10) cos (50 t). Then the equations of the component waves whose superposition gives the
above vibrations are
x x x x
(a) 2 sin ( + 50 t) + 2 sin ( – 50 t) (b) sin ( + 50 t) – sin ( – 50 t)
10 10 10 10
x x x x
(c) sin ( + 50 t) + sin ( – 50 t) (d) 2 sin ( + 50 t) + 2 sin ( + 50 t)
10 10 10 10
Sol. (c)

38. Hot coffee in a mug cools from 90ºC t o 70ºC in 4.8 minutes. The room temperature is 20ºC. Applying
Newton’s law of cooling the time needed to cool it further by 10ºC should be nearly
(a) 4.2 min (b) 3.8 min (c) 3.2 min (d) 2.4 min
Sol. (c)

 k( avg –  s )
t
20
90ºC  70ºC   k (80 – 20 )
4. 8
10
70ºC  60ºC   k ( 65 – 20 )
t
20 t 60
  
4.8 10 45
4
 t= × 2.4 = 3.2 min
3

39. A sinusoidal voltage of amplitude 15 V is connected between the input terminals of the circuit shown in the
figure. Assume that the diodes are ideal. In the output waveform

(a) positive peaks of the input will be clipped ot +12V and negative peaks will be clipped to –6 V
(b) positive peaks of the input will be clipped to +6V and negative peaks will be clipped to – 12 V
(c) positive peaks of the input will be clipped to +12V and negative peaks will be clipped to –12 V
(d) positive peaks of the input will be clipped to +6V and negative peaks will be clipped to –6 V
Sol. (a)
For positive cycle D1 – forward
D2 – reverse
So, N0 = +12
For negative cycle D1 – reverse
D2 – forward
So, v0 = –6 volt

www.examrace.com
NSEP-2013-14
40. Correlate a physicist among P, Q, R to an appropriate physicist among L, M, N. Let us pay a tribute to them.
P. Louis de Broglie L. Davison and Germer
Q. Max Plank M. James Clerk Maxwell
R. Christian Huygen N. Arthur H. Compton
(a) P & N, Q & M, R & L (b) P & L, Q & N, R & M
(c) P & L, Q & M, R & N (d) P & M, Q & L, R & N
Sol. (b)
h
 De Brogle
p
Justification was given by Davison and Germer. According to huygen light is wave Maxwell also used wave
concept of light. Max Plank and Compton used photon concept.

SUB- PART A-2


In question 41 to 50 any number of options (1 or 2 or 3 or all 4) may be correct. You are to identify
all of them correctly to get 6 marks. Even if one answer identified is incorrect or one correct
answer is missed, you get zero score.

41. In a circuit carrying an alternating current


(a) magnetic field around it oscillates with the frequency of the current
(b) heat is generated with double the frequency of the magnetic field
(c) voltage across the circuit is proportional to the rate of change of magnetic flux around it
(d) the current always lags in phase behind the applied voltage.
Ans. (a,b)

42. The stress exerted by vehicles on the central part of a bridge with convex curvature will :
(a) be more than that at the ends
(b) same as that at the ends
(c) be less than that at the ends
(d) decrease in proportion to the square of the speed of the vehicle
Ans. (a)

43. A cylindrical vessel filled with water is connected by a narrow pipe at its bottom to another identical empty
vessel. Then :
(a) potential energy of water is proportional to the square of the height of its level
(b) 3/4th of the potential energy is lost when the water flow stops
(c) half the potential energy is lost when the levels in both the vessels are same
(d) loss in potential energy is equal to the rise in the thermal energy
Ans. (a,c,d)

44. The deviation produced by a prism depends upon


(a) angle of incidence on face of prism (b) refracting angle of prism
(c) refractive index of prism (d) wavelength of light used
Ans. (a,b,c,d)

www.examrace.com
NSEP-2013-14
45. Consider nine identical resistances arranged as shown in the figure. In this arrangement electric current
enters at node A and leaves from node D. Let VAD = 5 volt and BE = 3 mA. Therefore :

(a) AB = 5 mA
(b) each resistance is (5000/11)
(c) effective resistance between A and D is 500
(d) power dissipation along the path BCD is (100/11)mW
Sol. (a,b,c,d)

(x + 3 + 3 + 3) r = 5
(x + 9) r = 5
x + 2x = 6 ...(1)
3x = 6  x = 2 mA ...(2)
5 5000
 11r = 5  r = = 
11 10 3 11
 x + 3 = 2 + 3 = 5mA
R (5 + 5) 10–3 = [x + 3 + 3 + 3] 10–3 r
R 10 = (x + 9) r
R (10) = (2 + 9) r
11  11   5000 
R= r  R =  10   11  = 500 
10   
2 2 2
(P)BCD = x r + 4x r = 5x r
2
 2   5000 
= 5   
 1000   11 

54 5000 100 100


= × = W = mW
1000  1000 11 1000  11 11

46. Mark the correct statement(s) of the following


(a) A convex mirror forms virtual images for all positions of object
(b) A concave mirror forms real images for all positions of the object
(c) A concave mirror can form a virtual magnified image
(d) The magnification produced by a convex mirror is always less than unity

Sol. (c)

www.examrace.com
NSEP-2013-14
u = +5
V = ?, f = +10
1 1 1
 =
V x f
1 1 1
 
V 5 10
1 1 1 1 2 1
    
V 10 5 10 10 10
V = –10
V 10
m=– =– =2
x 5

47. A solid cylindrical conductor of radius a and charge q is coaxial with a cylindrical shell of negligible
thickness, radius b(>a) and charge –q. The capacitance of this cylindrical capacitor per unit length is
proportional to :
(a) log (b/a) (b) log (a/b) (c) [log (b/a)]–1 (d) 1/ [logb – loga]
Sol. (c,d)

2k b
E= , V = 2 2k n 
r a


C = 2k n b 
a

4 0
= 2n b 
  a 

48. Mark the correct statement(s) of the following :


(a) In case of liquids the boiling point increases with pressure for all liquids
(b) In case of solids the melting point decreases with pressure for all solids
(c) In case of ice the melting point decreases with pressure
(d) In case of ice the melting point increases with pressure
Sol. (a,c)

www.examrace.com
NSEP-2013-14
49. The electric potential (in volt) in a region along the x-axis varies with x according to the relation
V(x) = 5 + 4x2,where x is in m. Therefore :
(a) the potential difference between the points x = 1 and x = –3 is 32 V
(b) force experienced by a charge of 1C placed at x = –1 m is 8N
(c) force experienced by the above mentioned charge is along the positive x-axis
(d) a uniform electric field exists in this region along the x-axis
Sol. (a,b,c)
V(x) = 5 + 4x2
V(1) = 5 + 4 = 9, V (–3) = 5 + 4(9) = 41
V(–3) – V(1) = 41 – 9 = 32 V
dV
E=– = – (0 + 8x) = – 8x = 8
dx
 F = 8 × 1 = 8 N ( )

50. A homogeneous bar of length L and mass M is situated at a distance h from a particle of mass m as shown.
The gravitational force exerted by the bar on the particle varies inversely as :

(a) (L – h)2 (b) (h + L/2)2 (c) h(h + L) (d) h2 if L << h


Sol. (c,d)

m
M 
h L G dx  h L
 L  GMm  1
F=

x h
x 2
=
L 

x 

h
GMm  1  GMm  1  1 
=   =  
L  x  h L L h hL

GMm  h  L  h  1
=   = GMmL
L  h (h  L )  L h (h  L )

GMm
= h (h  L )

1 1
F F  2 if L << h
h (h  L ) , h

www.examrace.com
NSEP-2013-14
PART B MARKS : 60

All questions are compulsory. All questions carry equal marks.

1. A particle is moving in positive x-direction with its velocity varying as v   x . Assume that at t = 0 the
particle was located at x = 0. Determine (i) the time dependence of velocity (ii) acceleration and (iii) the
mean velocity of the particle averaged over the time that the particle takes to cover the first s meters of the
path.
Sol. V=  x
squaring both side
 2 
V =x
2 2

 
V =2  2  x
2

 

2
 Acceleration is constant =
2
u=0
(i)  V = u + at
2
= t
2

2
(ii) acceleration (a) =
2
(iii) V=  s

uv  s
 average velocity = 
2 2

2. Two identical metal spheres of density p having equal and similar charges are supported from a common
point by means of a silk thread of length  and negligible mass. The two threads make an angle 21 with
each other when in equilibrium in air. When the same system is immersed in a dielectric liquid of density ,
then the angular separation changes to 22.
Then find (i) the relative permittivity ( r) of the liquid in terms of ,,1,2 . (ii) In case of the angular
separation remains unchanged even on immersing the system in the dielectric liquid find the expression
for  t.
Sol. Let charge on object is q.

kq2 1 q2
T sin 1 = Fe =  ......(i)
r2 4 0 r 2
T cos1 = mg .......(ii)
r = 2  sin 1 .......(iii)
Now if immered in liquid

Fe’ is new eelctrostatic force B is up thrust T1 is new tension.


www.examrace.com
NSEP-2013-14

1 q2
T1 sin 2 = Fe = .......(iv)
4 0  r r 2
T1 cos 2 = mg – B .....(v)
r’ = 2  sin 2 .......(vi)
Dividing (i) & (ii)

1 q2
tan 1 = 4 2 .....(vii)
0 ( 2 sin 1) mg

1 q2
similarity tan2 = 4  2 ....(viii)
0 r ( 2 sin  2 ) (mg  B)

Dividing (vii) & (viii)


tan 1  r sin 2  2 (mg – B )

tan  2 sin2 1 mg

tan 1 sin 2  2  Vg 


 r 1 – 
tan  2 2
sin 1  Vg 

sin 2  2  
r 1 – 
sin 2 1  

sin2 1 tan 1   


 r  1    r  
2
sin 2 tan 2     

sin 2 1 tan 1   
r   
sin 2 1 tan  2 r   –  
if 1 = 2

r =  r 
–

www.examrace.com
NSEP-2013-14
3. (a) The plane side of a thin planoconvex lens is silvered so that the lens acts as a concave mirror of focal
length 40 cm. The material of lens has refractive index 1.5. Determine the radius of curvature of the curved
surface of the lens.
(b) Light falls on one end of a cylindrical glass rod at an angle . Determine the smallest refractive index
that the glass may have so that the light after entering the rod does not leave it through its curved surface
irrespective of the value of .

Sol. (a)

f = 40 cm
1 1 2 1 2
   
f fm f  40 f

1  1 1 
 f = 80 cm, now f  (nrel – 1)  R – R 
 1 2 

1 1  1  1
 (1.5  1)  0    0.5 
f R  80 R
 R = 0.5 × 80 = 40 cm

(b)

sin( 90 – r ) cos 
n  n
sin 90 º sin r

sin(90 – r ) 1 1
  cos r =
sin 90 º n n

cos 
n
cos 
now n  n2 – 1
sin r
n

 n 2 – 1 = cos
for minimum n   0
 n2 – 1 = 1  n2 – 1 = 1
 n2 = 2
 n= 2

www.examrace.com
NSEP-2013-14
4. A cyclic process in indicated in the following PV-diagram. In the initial state (A) temperature, pressure and
volume of the system are 300 K, 1atm and 1000 cc. In the first process (AB), the adiabatic expansion
increases the volume to 2000 cc. This is followed by an isobaric compression to reduce the volume of gas
to 1000 c. The gas is brought to initial state by isobaric process. The system is 1 mole of a monoatomic
gas.
(a) Find the pressure and temperature at state B and C.
(b) Calculate the work done in adiabatic, isobaric, isobaric processes.
(c) Calculate the total work done in cyclic process and amount of energy supplied to the system
(d) Calculate the efficiency of cyclic process

Sol. (a)

3R 5R
Cv = , Cp =
2 2
5

3
PA VA = PB VB  1 (1) = PB 25/3
1
 PB = 5/3 atm
2
TA VA – 1 = TB VB –1

5
–1 2/3
 VA 3 TB  1
    TB = (300)  
 VB  TA 2

300
TB = k
22 / 3

10 5  1 
10 5 )10 – 3 ) – (2  10 – 3 ) 100 – 100 2 / 3 
PA VA – PB VB
 2 5/3
2 
(b) W =  –1 5 = 2
–1
3 3

 1 
J = 150  – 2 / 3  J
 2 

 10 
  (10 – 3 )
W BC = –  5 / 3 
2 

100
J=– J
25 / 3
W CA =0

www.examrace.com
NSEP-2013-14
(c) Total heat energy givven to system
Q = n CV (TA – TC) + n CP (TC – TB)
PB VB PC VC

TB TC

VB VC VB TB
 
TB TC , VC TC

2  300  1
  
1  22 / 3  TC

1  300 
TC =  K
2  22 / 3 

3R  300 
 Q =   300 – 
2  2  22 / 3 

5R  300 300 
+  – 
2  2  22 / 3 22 / 3 

3R  1  5R300  1 
=  300  1 – 5 / 2   J
2  2  2( 2 2 / 3 )  2 
Q = W

3R300  1  5R300
 1 – 5 / 2  – 
2  2  2(2 2 / 3 )(2)
  0.233
(d) 3R  1 
 3001 – 5 / 2 
2  2 
(We don’t consider n = 1)

5. A 750 Hz, 20 V source is connected to a resistance of 100 , a capacitance of 1.0 F and an inductance
of 0.18 H in series. Calculate the following quantities :
(a) Impedence of the circuit
(b) Draw an impedence diagram with suitable scale
(c) Power factor
(d) The time in which the resistance will get heated by 10ºC, provided that the thermal capacity of resistance
= 2 J/ºC
Sol.

z= ( XL  X C ) 2  R 2
  2n  2(750 )  1500 
XL = L  (1500 ) (0.18 )  848 .5 

1 10 6
XC = = = 212.12 
C 1500 

z= (636 .4)2  (100 )2


www.examrace.com
NSEP-2013-14

= 100 (6.364 )2  12
= 100 × 6.44  644 

XL  X C 848 .5  212 .12


tan   =
R 100
= 6.36
R 100
(C) cos   = = 0.155
z 644
(B) impedence is constant as n is constant
 rms 20
(D) irms = = A
z 644
2

H= irms 2 Rt   20 
 (100 )t
 644 
2
 20 
(ms) ( ) =   (100) t
 644 
2
 20 
(2) (10) =   (100) t
 644 
2
 644  1
t=    × 20
 20  100
t = 207.36 sec.

Physical constants you may need –


1. Charge on electron e = 1.6 × 10–19 C
2. Mass of an electron me = 9.1 × 10–31kg
3. Universal gravitational constant G = 6.67 × 10–11 N m2/kg2
4. Permittivity of free space 0 = 8.85 × 10–12 C2 / N m2
5. Gas constant R = 8.31 J/K mol
6. Planck constant h = 6.62 × 10–34 Js
7. Stefan constant  = 5.67 × 10–8 W/m2 k4
8. Boltzmann constant k = 1.38 × 10–23 J/K

www.examrace.com
INDIAN ASSOCIATION OF PHYSICS TEACHERS
NATIONAL STANDARD EXAMINATION IN PHYSICS 2014-15
Date of Examination: 23rd November, 2014
Time: 0930 to 1130 Hrs

Q. Paper Code: P 160

Write the question paper code mentioned above on YOUR answer sheet (in
the space provided), otherwise your answer sheet will NOT be assessed.
Note that the same Q. P. Code appears on each page of the question paper.
Instructions to Candidates –
1. Use of mobile phones, smartphones, ipads during examination is STRICTLY
PROHIBITED.
2. In addition to this question paper, you are given answer sheet along with Candidate’s
copy.
3. On the answer sheet, make all the entries carefully in the space provided ONLY in
BLOCK CAPITALS as well as by properly darkening the appropriate bubbles.
Incomplete/ incorrect/carelessly filled information may disqualify your candidature.
4. On the answer sheet, use only BLUE or BLACK BALL POINT PEN for making
entries and filling the bubbles.
5. Question paper has two parts. In Part A1(Q. Nos 1 to 60) each question has four
alternatives, out of which only one is correct. Choose the correct alternative and fill the
appropriate bubble, as shown.

In Part A2 (Q. Nos. 61 to 70) each question has four alternatives out of which any
number of alternatives (1, 2, 3 or 4) may be correct. You have to choose ALL correct
alternatives and fill the appropriate bubbles, as shown.

6. For Part A1, each correct answer gets 3 marks. A wrong one gets a penalty of 1 mark.
Part A2 full marks are 6 for each question, you get them when ALL correct answers are
marked.
7. Any rough work should be done only in the space provided.
8. Use of non-programmable calculator is allowed.
9. No candidate should leave the examination hall before the completion of the
examination.
10. After submitting your answerpaper, take away the Candidate’s copy for your reference.

Please DO NOT make any mark other than filling the appropriate
bubbles properly in the space provided on the answer sheet.
Answer sheets are evaluated using machine, hence CHANGE OF
ENTRY IS NOT ALLOWED.
Scratching or overwriting may result in a wrong score.
DO NOT WRITE ON THE BACK SIDE OF THE ANSWER SHEET.
Instructions to Candidates (continued)–
Read the following instructions after submitting the answer sheet.
11. Comments regarding this question paper, if any, may be sent by email only to
iaptpune@gmail.com till 25th November, 2014.
12. The answers/solutions to this question paper will be available on our website –
www.iapt.org.in by 3rd December, 2014.
13. CERTIFICATES and AWARDS –
Following certificates are awarded by the IAPT to students successful in NSEs
(i)Certificates to “Centre Top 10%” students
(ii)Merit Certificates to “Statewise Top 1%” students
(iii)Merit Certificates and a book prize to “National Top 1%” students
14. Result sheets and the “Centre Top 10%” certificates will be dispatched to the Prof-in-
charge of the centre by January, 2015.
15. List of students (with centre number and roll number only) having score above MAS will
be displayed on our website (www.iapt.org.in) by 22nd December, 2014. See the
Eligibility Clause in the Student’s brochure on our website.
16. Students eligible for the INO Examination on the basis of selection criteria mentioned in
Student’s brochure will be informed accordingly.
17. Gold medals will be awarded to TOP 35 students in the entire process.
Q.P. Code -P 160

INDIAN ASSOCIATION OF PHYSICS TEACHERS


NATIONAL STANDARD EXAMINATION IN PHYSICS 2014-15
Total Time : 120 minutes ( A-1 and A-2)

A–1
ONLY ONE OUT OF FOUR OPTIONS IS CORRECT.
N. B. – Physical constants are given at the end.

1. If the threshold of hearing is assumed to be the reference (0 dB), then the threshold of
pain is taken to be 120 dB. Let the corresponding sound intensities be I0 and I

respectively. Then,  is
(a) 120 (b) 1012 (c) 10-12 (d) 101.2


2. If E denotes the intensity of electric field, the dimensions of a quantity  are those of

(a) current (b) current density (c) electric potential (d) electric flux

3. Two stars of masses m1 and m2 distance r apart revolve about their centre of mass. The
period of revolution is
  
(a) 2π
 (b) 2
 (c) 2π
 (d) 2π

           

4. Let a body be placed at a point on the earth’s surface at a latitude λ where the radius of
the earth is R. Then, the body experiences an effective acceleration
"# 
(a)  −    (b)  +   !  (c)  −    (d)  −
$%& '

5. A particle moves in a plane with a constant speed along a path y = 2x2 + 3x – 4. When the
particle is at (0, – 4) the direction along which it is moving is inclined to the X axis at an
angle
(a) 630 (b) 720 (c) 270 (d) 00

6. Two particles A and B are moving in XY plane. Particle A moves along a line with
equation y = x while B moves along X axis such that their X coordinates are always
equal. If B moves with a uniform speed 3 m/s, the speed of A is
( )
(a) 3 m/s (b) ) m/s (c) 3√2 m/s (d) m/s

1
Q.P. Code -P 160

7. A uniform thin rod of length (4a + 2πa) and of mass (4m + 2πm) is bent and fabricated to
form a square surrounded by semicircles as shown in the figure. The moment of inertia of
this frame about an axis passing through its centre and perpendicular to its plane is

, - ,- ,)- )-


(a) ma2 (b) ma2 (c) ma2 (d) ma2
) )

8. A ball is dropped onto a horizontal surface from a height of 36 cm. After bouncing
several times it comes to rest covering a total distance of 100 cm measured in a vertical
direction. The percentage loss in its kinetic energy after its first impact is
(a) 36 (b) 64 (c) 53 (d) 96

9. A simple pendulum has a small disc shaped magnet as the bob whose magnetic moment
is along vertical. Just beneath the bob a current carrying coil is placed on a horizontal
table. The coil produces a uniform magnetic field. The dependence of the change in time
period |∆T| on current i can be graphically shown as

(a) (b) (c) (d)

10. Two coupled simple pendulums have nearly the same period. One of them is excited
while the other is at rest. Now energy keeps on transferring from one pendulum to the
other alternately. This periodic transfer of energy continues almost indefinitely with a
time period of 10 s. Then the difference of frequencies between the two pendulums is
(a) zero Hz (b) 0.1 Hz (c) 0.01 Hz (d) infinite

2
Q.P. Code -P 160

11. A large cylindrical vessel contains water to a height of 10 m. It is found that the thrust
acting on the curved surface is equal to that at the bottom. If atmospheric pressure can
support a water column of 10 m, the radius of the vessel is
(a) 10 m (b) 15m (c) 5 m (d) 25 m

12. A thin annular metal disc of inner and outer radii R1 and R2respectively, is freely
suspended from a point on its outer circumference. The length of the corresponding
equivalent simple pendulum is
"  " " ) " )"  " " ) "
(a) (b) (c) (d)
" " " "

13. Two identical thin metal strips, one of aluminum and the other of iron are riveted together
to form a bimetallic strip. The temperature is raised by 50o C. If the central planes of the
two strips are separated by 2 mm and the coefficients of thermal expansion for aluminum
and iron are respectively 30 × 10-6 / 0C and 10 × 10-6 / 0C, the average radius of curvature
of the bimetallic strip is about
(a) 50 cm (b) 100 cm (c) 150 cm (d) 200 cm

14. Standing waves are generated on a string loaded with a cylindrical body. If the cylinder is
immersed in water, the length of the loops changes by a factor of 2.2. The specific gravity
of the material of the cylinder is
(a) 1.11 (b) 2.15 (c) 2.50 (d) 1.26

15. A curved road with radius of curvature 200 m is banked with an angle of banking equal
to tan-1 (0.2). Now, if the traffic is at double the speed for which the road is designed, the
minimum value of the frictional coefficient needed is (g = 10 m/s2)
(a) 0.52 (b) 0.35 (c) 0.94 (d) 0.80

16. A plastic pipe filled with iron wires forms a soft iron core. Two identical coils that can
just slide over the pipe are placed on this soft iron core. Initially the pipe is kept
horizontal and a current is passed through the coils connected in series. The fields are in
opposition and the coils remain stationary with a separation of 5 cm. The system is now
made vertical and the separation between the coils reduces to 4 cm. Then the coefficient
of friction between the coils and the pipe is
(a) 0.41 (b) 0.02 (c) 0.5 (d) 0.3

17. A long straight wire carries a charge with linear density λ. A particle of mass m and a
charge q is released at a distance r from the wire. The speed of the particle as it crosses a
point distance 2r is
.'ln .'ln .'ln .'ln
(a)
-1 (b)
-1 (c)
-1 (d)
-1
2 2 2 2

3
Q.P. Code -P 160

18. A uniform meter scale is supported from its 20 cm mark. A body suspended from 10 cm
mark keeps the scale horizontal. However, the scale gets unbalanced if the body is
completely immersed in water. To regain the balance the body is shifted to the 8 cm
mark. Therefore, the specific gravity of the material of the body is
(a) 5 (b) 6 (c) 7 (d) 4

19. Temperature of 100 g of water in a thermoflask remains fixed for a pretty long time at
50oC. An equal mass of sand at 20oC is poured in the flask and shaken for some time so
that the temperature of the mixture is 40oC. Now the experiment is repeated with 100 g of
a liquid at 50oC and an equal amount of sand at 20oC when the temperature of the
mixture is found to be 30oC. The specific heat of the liquid (in kJ kg-1K-1) is
(a) 1.05 (b) 2.01 (c) 1.55 (d) 1.95

20. Let vavg, vp and vrms be respectively the average, the most probable and the root mean
square speeds of gas molecules according to Maxwell’s distribution. Then,
(a) vavg<vp<vrms (b) vp<vrms<vavg (c) vrms<vp<vavg (d) vp<vavg<vrms

21. A coal-based thermal power plant producing electricity operates between the
temperatures 27oC and 227oC. The plant works at 80% of its maximum theoretical
efficiency. Complete burning of 1 kg of coal yields 36000 kJ of heat. A house needs 10
units of electricity each day. Coal used for supplying the amount of energy for the house
in one year is
(a) 1141 kg (b) 580 kg (c) 605 kg (d) 765 kg

22. A copper-constantan thermocouple has thermoelectric power 40 µV/oC. One junction is


at 0oC while the other is at 50oC. The thermocouple is connected to a 30-0-30
galvanometer to produce a full scale deflection. If a 100 ohm resistance is connected in
series with the galvanometer, the galvanometer gives a deflection of 10 divisions. The
figure of merit of the galvanometer is
(a) 1.3 µA/div (b) 2.0 µA/div (c) 2.3 µA/div (d) 4.0 µA/div

23. A fresh dry cell of 1.5 volt and two resistors of 10 kΩ each are connected in series. An
analog voltmeter measures a voltage of 0.5 volt across each of the resistors. A 1000 µF
capacitor is fully charged using the same source. the same voltmeter is now used to
measure the voltage across it. The initial value of the current and the time in which the
voltmeter reading falls to 0.5 volt are respectively
(a) 60 µA, 11 s (b) 120 µA, 15 s (c) 150 µA, 15 s (d) 150 µA, 11 s

24. A charge of + 2 µC is situated off-centre of a hollow spherical metallic shell. Then,


(a) – 2µC charge gets uniformly distributed on the inner surface of the shell.
(b) + 2µC charge gets non-uniformly distributed on the outer surface of the shell.
(c) – 2µC charge gets non-uniformly distributed on the inner surface of the shell.
(d) no charge appears on the outer surface of the shell.

4
Q.P. Code -P 160

25. Two simple pendulums with heavy bobs – one using iron wire and the other aluminium
wire are excited simultaneously. It is found that when the first pendulum completes 1000
oscillations the other completes 1001. When the temperature is raised by toC, it is found
that the two pendulums now oscillate together. If the coefficients of thermal expansion of
iron and aluminium are 10 × 10-6 /oC and 30 × 10-6 /oC, the value of tis
(a) 77.2oC (b) 123.2oC (c) 100.1oC (d) 105.2oC

26. Consider a body moving through air at a speed greater than that of sound. Out of the
following terms that one which is NOT connected with this event is
(a) sonic boom (b) ultrasonic (c) Mach number (d) conical wavefront

27. A short bar magnet is placed along N-S direction with N pole pointing north. The neutral
points are located 20 cm away from the bar magnet. If BHis the horizontal component of
earth’s magnetic field, then the magnetic field due to the bar magnet at a distance of 40
cm along its axis is
45 45 45 45
(a) (b) (c) (d)
, 6 (7

28. A plane mirror coincides with a plane having equation x = 3. A particle is moving along a
line with direction ratios 3,4,5. If speed of the particle is √2 , the velocity of its image is

(a) 8 9̂ + 8 ;̂ + 8 <= (b) − 8 9̂ − 8 ;̂ − <= (c) 8 9̂ + 8 ;̂ − 8 <= (d) − 8 9̂ + 8 ;̂ + <=


) , ( ) , ) , ( ) ,

29. An unpolarized light is travelling along Z axis through three polarizing sheets. The
polarizing directions of the first and the third sheet are respectively parallel to X axis and
Y axis whereas that of the second one is at 60oto the Y axis. Then, the fraction of the
initial light intensity that emerges from the system is about
(a) zero (b) 0.093 (c) 0.031 (d) 0.28

30. One face of a glass (µ = 1.50) lens is coated with a thin film of magnesium fluoride MgF2
(µ = 1.38) to reduce reflection from the lens surface. Assuming the incident light to be
perpendicular to the lens surface, the least coating thickness that eliminates the reflection
at the centre of the visible spectrum (λ = 550 nm) is about
(a) 0.05 µm (b) 0.10 µm (c) 1.38 µm (d) 2.80 µm

31. Consider the analogy between an oscillating spring-body system and an oscillating LCR
circuit. Then, the correspondence between the two systems that is NOT correct is
(a) chargeq corresponds to displacement x of the body.
(b) inductanceL corresponds to mass m of the body.
(c) capacitanceC corresponds to spring constant k .
(d) magnetic energy corresponds to kinetic energy of the body.

5
Q.P. Code -P 160

32. A 50 Hz ac source is connected to a capacitor C in series with a resistance 1 kΩ. The rms
voltages measured across them are 5 volt and 2 volt respectively. Assume the capacitor to
be ideal. The peak value of the source voltage and the capacitance are respectively
(a) 7 V, 1.27 µF (b) 5.3 V, 2.3 µF (c) 7.62 V, 1.27 µF (d) 3 V, 2.3 µF

33. Refer to the circuit given below. Initially the switch S is in position 1 for 1.5 s. Then the
switch is changed to position 2. After a time t (measured from the change-over of the
switch) the voltage across 5 kΩ resistance is found to be about 1.226 volt. Then, t is

(a) 330 ms (b) 500 ms (c) 33 ms (d) data insufficient

34. A long straight wire of radius R carries a uniformly distributed current i. The variation of
magnetic field B from the axis of the wire is correctly represented by the graph

(a) (b)

(c) (d)

6
Q.P. Code -P 160

35. Two copper coils A and B are wound over a plastic pipe. Coil A is connected to a
sinusoidal voltage source of frequency 50 Hz so that a current of 100 mA passes through
it. The voltage across coil B is 5 volt. Now if coil B is short circuited, there is a change of
current of 2 mA in coil A. Then, the mutual inductance between the two coils and the
percentage change in the impedance of coil A are respectively
(a) 160 mH, 2% (b) 16 mH, 0% (c) 1.6 mH, 2% (d) 0.16 mH, 0%

36. A coil is wound on an iron rod and connected to an ac source as shown in the figure. Two
more coils AB and CD are also wound on the same rod. If ends B and C are joined, a
filament bulb connected between ends A and D glows well. However, if B and D are
joined and the bulb is connected between A and C, it glows feebly. This shows that

(a) coils AB and CD are in series in the first case while they are in parallel in the second
case.
(b) in the second case the two coils are in phase addition and they have unequal number
of turns.
(c) in the second case the two coils are in phase opposition and they have equal number
of turns.
(d) in the second case the two coils are in phase opposition and they have unequal
number of turns.

37. The age of an organic material is usually determined by measuring its 14C content (carbon
dating). The ratio of the number of stable isotope of 14C atoms present to the number of
radioactive 14C atoms in a certain material is found to be 3:1. If the half life of 14C atoms
is 5730 years, the age of the material under investigation is
(a) 7944 years (b) 17190 years (c) 11460 years (d) 13972 years

38. The arrangement of NAND gates shown below effectively works as

(a) AND gate (b) OR gate (c) NAND gate (d) NOR gate

7
Q.P. Code -P 160

39. A beam of 28 keV electrons strikes a target generating X rays. The minimum wave length
λmin (called cutoff wavelength) of the X rays generated is
(a) 4.4 nm (b) 44 nm (c) 0.044 nm (d) 0.44 nm

Group of Q. Nos. 40 to 47 are based on the following paragraph.


A potentiometer is made using a resistance wire about 5 m long and having a resistance
of 8 Ω/m. The diagram shows the arrangement on a wooden board. The wire is turned
round brass screws (P) used as pegs giving 5 parallel segments 1.0 m each. The wire
remains taut under moderate tension. It is held in place by strips 3 mm thick marked as S
and S'. The experimental circuit shows the labeled electrical components. L is Leclanche
cell (emfe1 = 1.40 volt) and D is Daniel cell (emfe2 = 1.08 volt). Note that answers
obtained in any earlier question/s may be needed in further questions and such
answers should be used wherever needed.

40. It is required to decide the ratio (e1 / e2) by sum and difference method ALSO. Four cells
with different values of emfsE are available. One must use a cell with emfE equal to
(a) 1.40 volt (b) 2.0 volt (c) 4.50 volt (d) 1.08 volt

41. The best material for strips marked S and S' is


(a) plastic (b) aluminium (c) cast iron (d) plated brass

42. Assuming that due to stretching of wire while preparing the potentiometer, its resistance
has increased by 2% and a potential gradient of 0.6 mV/mm is needed, then R must be
(a) 13.5 ohm (b) 40.8 ohm (c) 20.4 ohm (d) 135 ohm

8
Q.P. Code -P 160

43. It is claimed that the strips S and S' serve two purposes – (I) to decide the end points of
the wire, and (II) to keep the wire in its place. Then,
(a) both (I) and (II) are important.
(b) (I) is more important than (II).
(c) (II) is more important than (I).
(d) both are unimportant.
44. The length of wire between the adjoining pegs carries current
(a) equal to that in the potentiometer wire.
(b) equal to half the current in the potentiometer wire.
(c) nearly zero.
(d) equal to zero.
45. Two new and different cells having emf’sv1 and v2 have their balance points P1 and P2
respectively. Then,
(a) v1 >v2 (b) v1 <v2 (c) v1 = v2 (d) information is not sufficient.

46. The ‘emf under test’ contains an arrangement as connected in the circuit.
It is observed that wherever the jockey is touched to wire the galvanometer shows full
scale deflection only on one side. The possible causes (considered one at a time) are (I) e
is D and e' is L, (II) e is L and e' is D, (III) key K is not inserted, (IV) value of R is much
larger than that set as per Q. No. 42, (V) value of R is very small. The possible causes are
(a) (I), (II) and (III) (b) (II), (III) and (IV)
(c) (I), (III) and (IV) (d) (II), (III) and (V)

47. If the jockey is touched at a point on the wire 1.0 cm away from the balance point, then
the galvanometer (G = 1 kΩ) will show a current equal to
(a) 2 µA (b) 4 µA (c) 6 µA (d) 8 µA

Group of Q. Nos. 48 to 55 are based on the following paragraph.


A large number of pendulums with identical bobs (mass m) but varying lengths are
suspended from a thick thread. Another pendulum of a heavier bob (mass M) is also
suspended from the same thread as shown.

9
Q.P. Code -P 160

This pendulum with the heavier bob is used as a ‘driver’ to drive the other pendulums
called as ‘driven’ pendulums. Assume that the amplitude of the driver is maintained
constant (by some suitable mechanism). Let the frequency of the driver be f0.

48. The time periods – hence the frequencies (f) and the amplitudes (A) of the driven
pendulums in steady state are measured. The variation of A with f is correctly shown by
the graph

(a) (b) (c) (d)

49. It is observed that


(a) all the pendulums except one are at rest.
(b) all the pendulums oscillate in phase with the ‘driver’.
(c) one of the pendulums oscillates with maximum amplitude.
(d) the pendulum with maximum amplitude oscillates in phase with the ‘driver’.

50. The frequency of the pendulum having maximum amplitude is


(a) f0 / 2 (b) f0 (c) 2 f0 (d) not related to f0

51. The pendulum in Q. No. 50 above is set into oscillation with an initial amplitude of 10.0
cm. Soon this pendulum comes to rest momentarily and the driver is seen to oscillate with
an amplitude of 8.16 cm. Then, mass M equals
(a) 1.5 m (b) 2 m (c) 2.5 m (d) 3 m

Now only one of the driven pendulums is oscillated. The driver and all other driven
pendulums are clamped.

52. A simple pendulum of length L has a period T. If length is changed by ∆L, the change in
period ∆T is proportional to
( (
(a) T (b) T2 (c) (d)
> √>

10
Q.P. Code -P 160

53. The variation of amplitude A with respect to time t is shown as

(a) (b) (c) (d)

54. Which of the following will give a straight line graph?


(a) log A against t (b) log A against 1/t (c) A against t2 (d) A2 against t

55. If v is the velocity of the bob the force that is responsible for decrease of amplitude is
proportional to
( (
(a) v 2 (b) v (c) ? (d) ? 

Q. Nos. 56 to 60are to be solved as group questions.


Note that answers obtained in any earlier question/s may be needed in further
questions and such answers should be used wherever needed.

56. An object is placed 30 cm away from a symmetric convex lens and an image two thirds
of the size of the object is produced. The object is moved by a distance of 20 cm so as to
get a magnified image. Now we get
(@
(a) a real image of magnification .
7
(b) a virtual image of magnification 5.
(c) a real image at a distance of 40 cm.
(d) a virtual image at a distance of 60 cm.

57. A symmetric concave lens of focal length 24 cm is now placed in contact with the convex
lens and the object is brought back to its original position. The image formed will be

(a) a real one with a magnification 4.


(b) a real one at a distance of 40 cm.
(c) a virtual one at a distance of 120 cm.
(d) a virtual one with a magnification 2.5.

11
Q.P. Code -P 160

58. The concave lens is moved away from the object through a distance of 10 cm. We get an
image that is
(a) virtual and at a distance of about 17 cm from the concave lens.
(b) real and at a distance of about 47 cm from the object.
(c) virtual, diminished and at a distance of 10 cm from the concave lens.
(d) real and at a distance of 57 cm from the object.

59. Now consider again the lenses to be in contact with each other but made of material of
refractive index 1.2. The system is immersed in a medium of refractive index µ and it is
found that the focal length of the system remains numerically the same as when in air.
Therefore, µ is
(a) less than 1.2 (b) between 1.2 and 1.5
(c) greater than 1.5 (d) equal to 1.5

)
60. The given convex lens (refractive index ) is made to rest on the surface of a lake such
,
that its upper surface is in air while the lower one is in water (refractive index ). Rays
)
from the sun overhead converge at a distance ‘a’ inside the water, while rays from a
luminescent anglerfish beneath at the bottom of the lake converge at a distance ‘b’ in air.
Therefore,
(a) a = 12 cm, b = 12 cm (b) a = 24 cm, b = 12 cm
(c) a = 18 cm, b = 12 cm (d) a = 24 cm, b = 18 cm

12
Q.P. Code -P 160

A2
In Q. Nos. 61 to 70 any number of options (1 or 2 or 3 or all 4) may be correct. You are to
identify all of them correctly to get 6 marks. Even if one answer identified is incorrect or
one correct answer is missed, you get zero marks.

61. Four point masses are placed in a plane so that their centre of mass is at (1,1). Three of
them are of mass m each and are placed at (0,0), (2,0) and (0,2) respectively. The fourth
point of mass 2m is displaced from its initial position such that centre of mass of the
system moves to (2,1). Then, the displacement of the fourth point mass is
(a) parallel to X axis. (b) inclined at an angle 45o with X axis.
8
(c) of magnitude units. (d) of magnitude 5 units.

62. A block A of mass 2 kg rests on a horizontal surface. Another block B of mass 1 kg


moving at a speed of 1 m/s when at a distance of 16 cm from A, collides elastically with
A. The coefficient of friction between the horizontal surface and each of the blocks is 0.2.
Then, ( g = 10 m/s2 ),
(a) after collision block B rebounds.
(b) after collision block B comes to rest.
(c) final separation between the blocks is 3 cm.
(d) final separation between the blocks is 5 cm.

63. A uniform disc of mass M and radius R is lifted using a string as shown in the figure.
Then,

(a) its linear acceleration is g upward. (b) its linear acceleration is g downward.
B
(c) its angular acceleration is . (d) its rate of change of angular momentum is MgR.
"

13
Q.P. Code -P 160

64. Four thin straight long wires are all parallel to Z axis. They pass through the points
A(3,0,0), B(0,3,0), C(-3,0,0) and D(0,-3,0). They all carry currents in <= direction of
magnitudes 0.3 A, 0.6 A, 0.3 A and 0.3 A respectively. The magnitude of magnetic field
at the origin O due to
(a) wires at A and C is zero. (b) wires at A and B is 2√2 × 10-8 T

(c) wires at A and D is 2√2 × 10-8 T (d) all wires is 2 × 10-8 T

65. In a drip irrigation system water flows at 0.4 m/s through a 25 mm diameter pipe. At each
of the plants in the field water is expected to be delivered at 0.02 m/s through a 2 mm
opening. The drip works for 2 hours a day. Then,

(a) the system feeds 2250 plants.


(b) a plant gets about 3.2 litres of water a day.
(c) the system feeds 3125 plants.
(d) a plant gets about 1.8 litres of water a day.

66. Refer to the circuit given below. Output voltage V0 is measured between points a andb.
Then,

(a) the peak value of V0 is 2.5 volt above the minimum if the diode is assumed to be
ideal.
(b) the positive half cycle of the input is clipped.
(c) the circuit acts as a rectifier.
(d) the peak value of V0 is about 3.2 volt above the minimum if D is silicon diode (non-
ideal).

67. Two constant volume gas thermometers – one containing helium and the other containing
oxygen are used to measure the boiling point of liquid nitrogen. For calibrating the He
thermometer first it is dipped in boiling water and afterwards in boiling liquid nitrogen
and the pressure was found to change by a factor of 5. The process is repeated with
oxygen thermometer. Then, which of the following statement/s is/are true?

(a) According to He thermometer liquid nitrogen boils at 74.6 K.


(b) Oxygen gas thermometer also gives the same result.
(c) Oxygen gas thermometer cannot be used in this situation.
(d) Helium gas thermometer cannot give the linear variation of pressure with
temperature.
14
Q.P. Code -P 160

68. A hollow prism filled with hot water is used with usual arrangement to obtain a spectrum.
The water prism is set in minimum deviation position. It is observed that the spectrum
shifts so that deviation increases. Indicate the correct statement/s.
(a) Refractive index of water increases with decrease of temperature.
(b) Refractive index of water increases with increase of temperature.
(c) Speed of light decreases with decrease of temperature.
(d) Speed of light increases with decrease of temperature.

69. A vertical narrow wire is illuminated with laser. Alternate dark and bright bands are
formed on a graph paper pasted on a distant wall. Indicate the correct statement/s.
(a) Making appropriate measurements it is possible to determine the diameter of the wire.
(b) This phenomenon exhibits that light does not follow rectilinear paths.
(c) This is a case of Fraunhoffer diffraction.
(d) This is a case of interference of an infinitely large number of Huygens’ secondary
waves leading to a diffraction pattern.

70. Consider an element of a stretched string along which a wave travels. During its
transverse oscillatory motion, the element passes through a point at y = 0 and reaches its
maximum at y = ym. Then, the string element has its maximum
(a) kinetic energy at y = ym.
(b) elastic potential energy at y = ym.
(c) kinetic energy at y = 0.
(d) elastic potential energy at y = 0.

-x-x-x-x-x-x-x-x-x-
Physical constants you may need…
Charge on electron e = 1.6 × 10-19 C Mass of electron me = 9.1 × 10-31 kg
Universal gravitational constant G = 6.67 × 10-11 Nm2/ kg2
Permittivity of free space ε0 = 8.85 × 10-12 C2/Nm2
Universal gas constant R = 8.31 J/mol K Planck constant h = 6.62 × 10-34Js
Stefan constant σ = 5.67 × 10-8 W/m2K4 Boltzmann constant k = 1.38 × 10-23 J/K
Mass of proton mp = 1.67 × 10-27 kg
Boiling point of nitrogen = 77.4 K Boiling point of oxygen = 90.19 K
Boiling point of hydrogen =20.3 KBoiling point of helium = 4.2 K

15
INDIAN ASSOCIATION OF PHYSICS TEACHERS
NATIONAL STANDARD EXAMINATION IN PHYSICS 2014-15
ANSWERS / SOLUTIONS
Q. P. Code 160

A1

1. (c) Use the definition of dB scale: 120 = 10 log10 , which gives = 10 -12.
2. (b) Use Gauss law where so that E= and .
3. (d) Let the distances of the stars with masses m1 and m2 from their centre of mass
be r1 and r2 respectively so that (r1 + r2) = r, say. The necessary centripetal force
is provided by the gravitational force between them, so that
. This gives . From this one
can write the expression for T.
4. (c) Effective acceleration is the vector sum of and acceleration due to
rotation of the earth r where r is the distance from the axis of rotation. Also
one can neglect r compared to g while determining the magnitude g'. Therefore,
. Also r can be written as where λ is the latitude.
5. (b) The direction along which the particle moves is the direction of velocity
which in turn, is the slope of the trajectory. This is and
hence the answer.
6. (c) If the particle A is at (x, y), particle B will be at (x, 0). The speed of B is given
to be m/s which is also the x component of the velocity of A. Since
for particle B, the equation of trajectory is y = x, = 3 m/s.
Therefore, speed of A is .
7. (c) Moment of inertia =
8. (c) One can write the series 100 = 36 + 2 giving
where e is the coefficient of restitution. Now, the percentage loss of
kinetic energy = . Note that the speed after impact is
e times the initial speed. This gives the percentage loss as 52.94 which is almost
53%.
9. (a) The magnetic force exerted and hence the corresponding acceleration is
proportional to the current. Consider the expression for the period of a simple
pendulum in which the effective acceleration due to gravity can be written as

where a is a constant. Then,

1
where T0 is the original period. Therefore, the
change in period |ΔT| is linearly proportional to the current i.
10. (b) The beat frequency is the difference in frequencies which is the reciprocal of
the beat period T which is given to be 10 s.
11. (b) Thrust on the curved surface whereas the thrust
on the bottom = . The atmospheric pressure in terms of
column of water is 10 m. equating the two thrusts gives R = 15 m.
12. (b) Moment of inertia of annular disc about the axis under consideration can be
written as . Comparing the expression for the periodic time
of this oscillating disc as a compound pendulum with that of a simple pendulum,
we get the equivalent length.
13. (d) For aluminum one can write where R2 is the radius
of the arc of aluminum strip and t is the temperature. Note that θ is the angular
width of the strips. A similar expression can be written for iron strip. The
difference between the radii, say d, for aluminum and iron is the distance between
the central planes of the two strips. The expression for d can be obtained from the
above two expressions as . Also (R1 + R2) can be approximated as
2i and the value of R can be determined.
14. (d) In this case is constant. If wavelength is λ (when the cylinder is in air), that
when the cylinder is immersed in water will be .
15. (a) As per road design, the rated speed is m/s. Now, the speed is
to be 40 m/s which will be helped by friction. With the help of free body diagram,
one can write and .
Simplifying this one gets . Using tanθ = 0.2, one gets
approximately.
16. (a) Each of the coils acts as a magnetic dipole. The force of interaction between
them varies inversely as the fourth power of the distance of separation. When the
coils are placed vertically only the weight mg of the coil balances this force.
However, when they are placed horizontally a force equal to µmg balances this
force. Thus, which is almost 0.41.
17. (b) The electric field at a distance x from the wire is so that the force
on a charge q would be . Writing the force as , the speed can be
determined by integrating the expression.
18. (b) Let w1 be the weight of the body in air and w2 be its weight in water. Using
law of moments, one gets w1 = 3mg and w2 = 2.5 mg where mg is the weight of
the meter scale. The specific gravity is then = 6.
19. (a) Let s1 be the specific heat of sand and s be that of the liquid. Using the
principle of calorimetry, one gets 4200 = 2 s1 and s1 = 2 s and hence the answer.

2
20. (d) Consider the expressions for the different speeds, ,

and , where symbols carry their usual meanings. Now determine their
inter-relation.
21. (a) Ideal efficiency is 40% whereas the actual efficiency is 40 × 0.8 = 32%. In a
year the number of units required is 10 × 365 = 3650 units. One unit corresponds
to 3600 kJ of energy, so that in a year the actual output is required to be 36000 ×
365 kJ, which is (equivalent of 1 kg of coal) × 365. With 32% efficiency this
gives the amount of coal required throughout a year is 1140.625 kg of coal.
22. (a) The thermoemf e = 40 (50 – 0) = 2000 µV. If G is the resistance of the
galvanometer, the current through it is µA which produces full scale
deflection of 30 divisions. Therefore, where is the figure of merit of
the galvanometer. After connecting 100 ohm resistance in series with the
galvanometer, the equation becomes . Solving these equations one
gets G = 50 ohm and µA/div.
23. (d) Let the resistance of the voltmeter be R (in kΩ, for convenience). Considering
the voltage drops, one gets R = 10 kΩ. Initial current is then obviously 150 µA.
The time required for the voltage across the capacitor to fall from V0 to V is given
by . Using this one gets t = 11 s.
24. (c) A charge – 2 µC is non-uniformly distributed on the inner surface of the
spherical shell whereas an equal positive charge is uniformly distributed on the
outer surface.
25. (c) If T1 and T2 are the periodic times of the two pendulums, and

hence the ratio of lengths is . This gives L1 = 1.002 L2. Now,


one can write the expressions for the increased lengths after the temperature is
raised by t0 C and equate the two. Solving this one gets the temperature t.
26. (b) Ultrasonic has to do with the frequency and not the speed.
27. (b) Clearly the neutral point is on the equator of the magnet and the magnetic
field at that point is . Writing a similar expression for the field at a
point on the axis, the ratio can be taken to give the answer.
28. (d) Knowing the direction ratios, one can write the unit vector in the direction in
which the object is moving. Therefore, the velocity of the object can be written as
. Since, the mirror is along the plane x = 3, the
velocity will have its x component only inverted.
29. (b) If I0 is the initial intensity of light entering the first polarizing sheet, the
intensity after this sheet is . The intensity of light after the second sheet is
and similarly that after the third sheet is .
Finally expressing I3 in terms of I0 one gets the result.
30. (b) Note that if the rays reflected from the two interfaces (air-film and film-glass)
interfere destructively, the reflection will be eliminated. For this the total path

3
difference must be an odd multiple of half the wavelength. If L is the film
thickness, where the wavelength in MgF2 is . Now
for L to be minimum n should be zero. Substituting the values one gets the result.
31. (c) The capacitance C corresponds to and NOT k.
32. (c) Current through the circuit is obviously 2 mA. Therefore, the capacitive
reactance is kΩ. From this the capacitance turns out to be 1.27 µF.
The peak value of the source voltage is times the net RMS voltage which is
volt.
33. (a) The charging time constant is 220 ms. Since the contact is more than five time
constants the capacitor gets fully charged, that is the voltage across the capacitor
is 10 volt. Now, the discharging time constant is 330 ms. Therefore, after 330 ms,
the voltage across the capacitor and hence across the two resistors together is
3.678 volt. Out of this the voltage across the 5 kΩ resistor is 1.226 volt.
34. (c) The variation of B with the distance is linear inside the wire.
35. (a) The voltage across coil B is where is the current in
coil A. This gives . Taking into account the rms values of
current and voltage, M comes out to be 159 mH. Initial current in coil A is, say I1
= 0.1 A. Therefore, if V is the voltage across it, the impedance of coil A is 10V.
After coil B is shorted, the current in coil A increases to 0.102 A, hence its
impedance is . Then the percentage change in impedance is
1.96% or about 2%.
36. (d) Due to the emf’s induced being in phase opposition the net current is small.
Also since the number of turns is not the same the current does not drop to zero
and hence the bulb glows feebly.
37. (c) Let N2 be the number of atoms of stable isotopes and N1 be that of radioactive
isotopes. If N0 is the initial number then, N2 = N0 – N1. Now, N1 = N0 e –λt = (N1 +

N2) e –λt. This gives . Writing one gets the answer.


38. (c) With the help of truth table for NAND gate write the output at every stage for
all possible cases of the input.
39. (c) The minimum wavelength λmin corresponds to νmax where all the energy carried
by the electron is totally transferred to X ray photon. Writing λmin = where E is
the energy of the electron, one gets the answer.
40. (c) Particularly when the cells assist each other (sum), the driving cell must have
an emf E greater than the sum of the emf’s [ > (1.40 + 1.08)] of the cells under
test.
41. (d) Electrical conductivity of brass is the greatest among the materials given. Due
to plating the strips are free from corrosion.
42. (c) Due to stretching the resistance of the potentiometer wire is 40 + 2% = 40.8
ohm. Potential gradient v can be written as where r is the resistance of
the wire and R is the resistance connected in the circuit. Solving for R gives the
answer.

4
43. (b) Determining the end point is more important than keeping the wire in place.
44. (c) The metal strip S has a very small resistance (close to zero, but NOT zero).
45. (b) Note that point P1 is closer to the common point (where the positive terminal
of E and that of the cell under test are connected) than the point P2 and the emf is
proportional to the balancing length.
46. (c) If e < e', the emf under test will be negative and the current will always pass
through the galvanometer. If key is open, current is only due to the emf under test
which never becomes zero. If R is too large, the potential difference across the
total length of wire may be smaller than the emf under test.
47. (c) The potential difference across 1 cm wire is 6mV and hence the current
through the galvanometer is 6 µA.
48. (b) Obviously the amplitude is a maximum for one particular frequency at which
a pendulum resonates with the driver pendulum.
49. (c) The pendulum which has the same natural frequency of oscillation as that of
the driver pendulum has maximum amplitude.
50. (b) The frequency of the resonating pendulum is obviously the same as f0 the
frequency of the driver pendulum.
51. (a) When one of the coupled pendulums stops, its energy is completely
transferred to the driver pendulum. If A and A' are respectively the amplitudes of
the coupled pendulum and the driver pendulum, . This
gives M = 1.5 m.
52. (c) Considering the expression for the periodic time T of a simple pendulum, ΔT
is proportional to
53. (b) The amplitude of a simple pendulum exponentially decreases with time.
54. (a) The graph of log A against t is a straight line as A varies exponentially with t.
55. (b) The decrease of amplitude is due to a damping force that is proportional to
velocity v.
56. (d) Considering the object distance u and the magnification m, the image distance
v comes out to be 20 cm and the focal length f to be 12 cm. Now, the object has to
be moved closer to the lens so as to get a magnified image. Taking u' = 10 cm,
image distance comes out to be 60 cm.
57. (a) Focal length of the combination is + 24 cm. With u = 30 cm, image distance is
120 cm and the magnification is 4, that is the image is real and magnified.
58. (d) Convex lens forms a real inverted image at 20 cm from the lens. This acts as
the virtual object for the concave lens forming a final image at or
approximately 17 cm from the concave lens. Note that this is a real image of the
virtual object formed by the concave lens. Therefore, from the object the final
image is at a distance of 57 cm.
59. (d) At first consider the convex lens made of material of refractive index 1.2 and
placed in air (µ = 1) and then immersed in a medium of refractive index µ = 1.5.
The focal length happens to be numerically 5k where k is a factor decided by the
radii of curvature. It is seen that the focal length numerically remains the same.
The same argument can be made for the concave lens. Therefore, when the two
lenses are in contact, the effective focal length remains numerically the same.

5
60. (d) Use the formula for refraction at a spherical surface. Since the media on the
two sides of the lens are different, consider the formation of image by one surface
at a time. Consider parallel rays from air and n1 = 1, n2 = , image distance by the
first surface is 36 cm. For the second surface n1 = and n2 = , final image is
formed at 24 cm. Thus, a = 24 cm. Using a similar procedure and considering
parallel rays incident from water, the final image is formed at b = 18 cm.

A2
61. (a), (c) After displacement of mass 2m, there is no change in y coordinate of
centre of mass. This indicates that the displacement is along X axis. Using the
expression for the X coordinate of centre of mass, initially the X coordinate of
mass 2m comes out to be 1.5 whereas it is 4 when the mass is displaced.
Therefore, the displacement is of magnitude 2.5 or units.
62. (a), (d) Deceleration due to the frictional force is 2 m/s2, so that block B collides
with a velocity of 0.6 m/s. Using conservation of momentum, 2v1 + v2 = 0.6 where
v1 and v2 are the velocities of blocks A and B after collision. Also since the
collision is elastic (coefficient of restitution = 1), v1 v2 = 0.6. This gives v1 = 0.4
m/s and v2 = 0.2 m/s. The negative sign for v2 indicates that block B rebounds.
Displacement of block A after collision is 4 cm to the right whereas that of block
B is 1 cm to the left, so that the final separation is 5 cm.
63. (a), (c), (d) The net force upward is Mg so that the acceleration is g upwards. The
net torque is anticlockwise and is of magnitude MgR which is also the same as the
rate of change of angular momentum. Since the moment of inertia is , the
angular acceleration is .
64. (a), (c), (d) Using the expression for the magnetic induction due to a long straight
wire and the right hand rule, the magnetic inductions at the origin due to the wires
at A, B, C and D are 2 × 10-8 T along OY', 4 × 10-8 T along OX, 2 × 10-8 T along
OY and 2 × 10-8 T along OX' respectively. Therefore, the inductions due to wires
at A and C cancel out. Using the vector addition, the induction due to wires at A
and D is T and that due to all wires is 2 × 10-8 T.
65. (c), (d) Using the equation of continuity, a relation can be written as
where n is the number of plants. This gives n = 3125.
Further the flow of water is 0.25 ml per second so that in 2 hours each plant gets
1.8 litre of water.
66. (a), (c), (d) During the positive half cycle, the diode remains open and the output
is the same as the input and hence 2.5 volt. Since the negative half cycle is
clipped, the circuit acts as a rectifier. If the diode happens to be non-ideal the
output is 2.5 + 0.7 = 3.2 volt above minimum.
67. (a), (c) At constant volume, pressure is directly proportional to temperature.
Using this the temperature at which liquid nitrogen boils comes out to be 74.6 K.
Also since at this temperature oxygen is not in gaseous state (boiling point of O2
is 90 K), it cannot be used in gas thermometer.

6
68. (a), (c) On cooling of water, deviation increases indicating that the refractive
index of water increases. Considering the expression for refractive index in terms
of speed of light, the speed of light decreases with decrease of temperature.
69. (a), (b), (c), (d) Knowing the wavelength diameter of wire can always be
determined. Obviously in experiments on diffraction light does not follow
rectilinear paths. This is a case of Fraunhoffer diffraction. Diffraction is
essentially a case of interference of a large number of wavelets.
70. (c), (d) The kinetic energy is obviously maximum when the element passes
through the mean position. In the mean position the string element happens to be
in its maximum stretched and hence its elastic potential energy is also maximum.

-x-x-x-x-x-x-x-x-

7
INDIAN ASSOCIATION OF PHYSICS TEACHERS
NATIONAL STANDARD EXAMINATION IN PHYSICS 2015-16
Date of Examination: 22nd November, 2015
Time: 0930 to 1130 hrs

Q. Paper Code: P 136

Write the question paper code mentioned above on YOUR answer sheet (in the space
provided), otherwise your answer sheet will NOT be assessed.
Note that the same Q.P. Code appears on each page of the question paper.

Instructions to Candidates –
1. Use of mobile phones, smartphones, ipads during examination is STRICTLY PROHIBITED.
2. In addition to this question paper, you are given answer sheet along with Candidate’s copy.
3. On the answer sheet, make all the entries carefully in the space provided ONLY in BLOCK
CAPITALS as well as by properly darkening the appropriate bubbles.
Incomplete/incorrect/carelessly filled information may disqualify your candidature.
4. On the answer sheet, use only BLUE or BLACK BALL POINT PEN for making entries and
filling the bubbles.
5. Question paper has two parts. In part A1(Q. Nos. 1 to 60) each question has four
alternatives, out of which only one is correct. Choose the correct alternative and fill the
appropriate bubble, as shown.
Q. No. 22 a c d

In part A2 (Q. Nos. 61 to 70) each question has four alternatives out of which any number
of alternatives (1, 2, 3 or 4) may be correct. You have to choose ALL correct alternatives
and fill the appropriate bubbles, as shown.
Q. No. 64 a c

6. For Part A1, each correct answer gets 3 marks. A wrong one gets a penalty of 1 mark. Part
A2 full marks are 6 for each question, you get them when ALL correct answer are marked.
7. Any rough work should be done only in the space provided.
8. Use of non – programmable calculator is allowed.
9. No candidate should leave the examination hall before the completion of the examination.
10. After submitting your answerpaper, take away the Candidate’s copy for your reference.

Please DO NOT make any mark other than filling the appropriate bubbles
properly in the space provided on the answer sheet.
Answer sheets are evaluated using machine, hence CHANGE OF ENTRY IS NOT
ALLOWED.
Scratching or overwriting may result in a wrong score.
DO NOT WRITE ON THE BACK SIDE OF THE ANSWER SHEET.

FIITJEE Ltd., North West Delhi Centre, 31-32-33, Central Market, West Avenue Road, Punjabi Bagh (West), New Delhi - 110026, Ph: 011-45634000
INDIAN ASSOCIATION OF PHYSICS TEACHERS
NATIONAL STANDARD EXAMINATION IN PHYSICS 2015-16
Q. Paper Code: P 136

1. An expression containing certain physics quantities is 1273.43  51.7052  745   21 . After


evaluation the correct answer is:
(A) 41301.2208 (B) 4.1 10 4
(C) 41307 (D) 41000

2. A body of mass m and radius R rolling horizontally without slipping at a speed v climbs a
3v 2
ramp to a height . The rolling body can be
4g
(A) a sphere (B) a circular ring
(C) a spherical shell (D) a circular disc

3. A particle of mass 10 g starts from rest at t = 0 s from a point (0 m, 4 m) and gets


accelerated at 0.5 m/s2 along x  3y  4 3  0 in XY plane. The angular momentum of the
particle about the origin (in SI units) at t = 2 s is
(A) 0.01 3k (B) 0.02 3k
(C) zero (D) 20 3k

4. A body released from a height H hits elastically an inclined plane at a point P. After the
impact the body starts moving horizontally and hits the ground. The height at which point P
should be situated so as to have the total time of travel maximum is
(A) H (B) 2 H
H H
(C) (D)
4 2

5. A thin rod of length l in the shape of a semi circle is pivoted at one of its ends such that it is
free to oscillate in its own plane. The frequency f of small oscillations of the semicircular rod
is

(A)
1 g
(B)
1 g 2  4
(C)
1 g    2
(D)
2
1 g  1 
2 21 2 2l 2 l 2 2l

6. Two air bubbles with radii r1 and r2 r2  r1  formed of the same liquid stick to each other to
form a common interface. Therefore, the radius of curvature of the common surface is
(A) r1 r2 (B) infinity
r2 2 2 r1 r2
(C) r2  r1 (D)
r1 r2  r1

7. A particle executes a periodic motion according to the relation x  4 cos2  50t  sin  500t  .
Therefore, the motion can be considered to be the superposition of n independent simple
harmonic motions, where n is
(A) 2 (B) 3
(C) 4 (D) 5

8. A car moving along a straight road at a speed of u m/s applies brakes at t = 0 second. The
ratio of distances travelled by the car during 3rd and 8th second is 15 : 13. The car covers a
distance of 0.25 m in the last second of its travel. Therefore, the acceleration a (in m/s2) and
the speed u (in m/s) of the car are respectively
(A) –0.1, 16 (B) –0.2, 12
(C) –0.5, 20 (D) –0.1, 16

FIITJEE Ltd., N-W Centre, 31, 32, 33, Central Market, West Avenue Road, Punjabi Bagh, New Delhi - 26, Ph: 011-45634000
9. Masses m1 and m2 are connected to a string passing over a
pulley as shown. Mass m1 starts from rest and falls through
a distance d in time t. Now, by interchanging the masses
the time required for m2 to fall through the same distance is
2 t. Therefore, the ratio of masses m1 : m2 is
2 3
(A) (B)
3 2
5 4
(C) (D)
2 3

10. The graph of specific heat of water (on Y axis) against temperature (on X axis) between 0o C
and 100o C
(A) is a straight line parallel to the temperature axis
 
(B) is a straight line passing through a point 15o C, 1 cal / g  o C and having a small positive
slope.
(C) has a minimum between 14.5o C and 15.5o C
(D) has a minimum at about 30o C

11. A particle diode (p – n junction) when forward biased in equivalent to


(A) a closed switch (B) a cell (potential difference)
(C) a small resistance (D) all the above in series

12. The circuit shown below is equivalent to


(A) OR gate
(B) NOR gate
(C) AND gate
(D) NAND gate

13. Which one of the following statements in connection with a semi conducting material is
NOT CORRECT?
(A) They have negative temperature coefficient of resistance
(B) They have a moderate forbidden energy gap.
(C) Current is carried by electrons and holes both.
(D) Every semi conducting material is a tetravalent element.

Group of Q. No. 14 to 21 is based on the following paragraph

Generally light emitted from a source contains several wavelengths. Similarly the electrical
voltage at the output of a sensor contains a mixture of dc and several ac components of
different amplitudes and different frequencies. Filter circuits are used to pass desired
frequencies and / or to eliminate undesired frequencies. The frequencies transmitted by the
filter form the pass band while the frequencies eliminated by the filter form the stop band or
rejection band.

We can think of four basic types of electrical filters – a low pass filter where frequencies
below a certain cutoff frequency  c are passed. Similarly one can think of a high pass filter,
band pass filter, band stop (or band rejection) filter. The cutoff frequency  c is the frequency
1
at which the output voltage falls to times its maximum value.
2
An inductor and/or a capacitor is an essential component of a filter. Generally a resistance is
included in a filter circuit to determine the time constant and hence the cutoff frequency.

FIITJEE Ltd., N-W Centre, 31, 32, 33, Central Market, West Avenue Road, Punjabi Bagh, New Delhi - 26, Ph: 011-45634000
14. Refer to the RC networks (1) and (2) shown below. Which of the following statements is true?

(A) Each of the two networks represents a low pass filter.


(B) Each of the two networks represents a high pass filter.
(C) Network (1) represents a low pass filter while network (2) a high pass filter.
(D) Network (1) represents a high pass filter while network (2) a low pass filter.

15. The input – output voltage relation for a certain high pass filter is given by
V0 CR

Vi 1  2C2R2
The cut off frequency   c  for this filter will be
1 2  1
(A) (B) (C) (D)
2RC RC 2RC RC

16. The input – output voltage relation for a certain filter circuit is given by
V0 A

Vi 2
 
12  2  22
where  is the angular frequency of the input while 1 , A and  are constants. This relation
is meant for
(A) low pass filter (B) high pass filter
(C) band pass filter (D) band stop filter

17. Refer to the following schematic diagrams of different combinations of a low pass filter (LPF)
and a high pass filter (HPF). Assume f1  f2 . The combination that works as a band pass
filter is

(A) 
i (B) ii
(C) iii (D) iv

18. Refer to the schematic diagram in Q. No. (47). The combination that works as a band
elimination filter is
(A) 
i (B) ii
(C) iii (D) iv

FIITJEE Ltd., N-W Centre, 31, 32, 33, Central Market, West Avenue Road, Punjabi Bagh, New Delhi - 26, Ph: 011-45634000
19. An astrophysicist desires to study radiation at wavelengths higher than those for visible light
coming from a certain celestial body. He must use an optical filter that is
(A) high pass (B) low pass
(C) band pass (D) band rejection

20. Figure (A) below shows an acoustical filter that consists of a set of identical cavities
connected by narrow tubes and figure (B) shows its electrical analog. The acoustical filter
represented by figure (A) is

(A) low pass (B) high pass


(C) band pass (D) band rejection

21. Graphs I, II and III and IV shown below represent the frequency response of different types
of filter circuits. The correct order of these graphs corresponding to low pass, high pass band
pass and band top filter is

(A) I, II, III, IV (B) II, IV, III, I


(C) IV, II, III, I (D) IV, III, II, I

Group of Q. Nos. 22 to 30 is based on the following paragraph.

Equal volumes of two liquids (L1 and L2) are taken in two identical calorimeters. Both L1 and
L2 are initially at about 80oCC. Calorimeters are corked fitted with thermometers to record
the temperatures of the liquids. The temperatures are recorded every 30 s alternating
between the two liquids, that is the temperatures are recorded at an interval of 1 min for any
one liquid. The graphs of temperature  (oC) versus time t (min) for two liquids L1 and L2 are
as shown.

22. From the graphs it can be said that


(A) Newton’s law of cooling is not valid
(B) the specific heat of L2 is greater than that of L1
(C) the observation recorded are not consistent
(D) none of the above statements are correct

FIITJEE Ltd., N-W Centre, 31, 32, 33, Central Market, West Avenue Road, Punjabi Bagh, New Delhi - 26, Ph: 011-45634000
23. Equal volumes of the two liquids are necessary so that
(A) heat contents of the two liquids are equal
(B) the exposed surfaces are equal
(C) the calculations are simplified
(D) none of the above

24. The nature of the outer surfaces of the calorimeters


(A) should be blackened and rough
(B) should be silvered and rough
(C) should be silvered and polished / shining
(D) could be arbitrary

25. Which of the following arrangements would be the ideal environment for the
two calorimeters?
(A) A double walled box, both inner and outer space filled with water
(B) A double walled box with water in the inner box and empty outer box
(C) A double walled box with water in the outer box and empty inner box
(D)In air without any box.

26. The two curves will


(A) intersect at some later time
(B) merge after a long time
(C) remain separate at all times
(D) be parallel to the X axis but distinct after a long time

27. Given: mass of L1 = 50 g and mass of L2 = 62.5 g. If water equivalent of calorimeters is


s
assumed to be negligible, then 1 equals
s2
(A) 1.04 (B) 0.60
(C) 0.95 (D) 1.64

28. If 1 and 2 are the densities of L1 and L2 respectively then, identify the correct statement
(A) s1  s2 , 1   2 (B) s1  s2 , 1   2
(C) s1  s2 , 1   2 (D) s1  s2 , 1   2

29. If the experiment is carried out with equal masses of the two liquids, then
(A) L1 will cool faster
(B) L2 will cool faster
(C) both the liquids will cool at the same rate
(D) nothing can be said about the rates as data are insufficient

30. The entire experiment is repeated with other two liquids having nearly the same specific
heats. Then,
(A) the two curves will be coincident.
(B) the two curves will be parallel.
(C) the two curves will intersect at a point.
(D) nothing can be said about the two curves as data are insufficient.

31. When a light wave is incident at the interface between two media, the reflection coefficient is
2
 n  1
given by 2
where n is the refractive index of the denser medium with respect to the
 n  1
rarer medium. Two stretched strings whose linear densities are 25 g/m and 9 g/m are
joined together. Assuming the law of optics holds goods here also, the reflection coefficient
for the pulse along the strings is
(A) 9/16 (B) 3/4 (C) 1/16 (D) 1/9

FIITJEE Ltd., N-W Centre, 31, 32, 33, Central Market, West Avenue Road, Punjabi Bagh, New Delhi - 26, Ph: 011-45634000
32. A certain perfect gas occupying 1 litre at 80 cm of Hg suddenly expands to 1190 cc while the
pressure falls to 60 cm of Hg. Therefore, the gas is
(A) polyatomic (B) diatomic
(C) monatomic (D) data inadequate

33. Two thin rods of lengths I1 and I2 at a certain temperature are joined to each other end to
end. The composite rod is then heated through a temperature . The coefficients of linear
expansion of the two rods are 1 and 2 respectively. Then, the effective coefficient of linear
expansion of the composite rod is
  2
(A) 1 (B) 1  2
2
I I  I I 
(C) 1 2 2 1 (D) 1 1 2 2
I1  I2 I1  I2

34. A yo-yo has a spool of mass m and radius R. A massless string is would around an axle of
radius b and of negligible mass. If the yo-yo released from rest has a downward acceleration
of g/9, the ratio R/b is
(A) 2 (B) 3
(C) 4 (D) 5

35. A pulley of negligible mass is suspended from a spring balance. Blocks weighing 5.0 kg and
3.0 kg are attached to the two end ends of a string passing over the pulley. The reading on
the spring balance will be
(A) 8.0 kg (B) 7.5 kg
(C) 2.0 kg (D) 4.0 kg

36. A uniform rod (ABCDAC) is bent in the shape of a kite as


shown. If a point X along AC is the centre of mass of the
structure, distance AX is
(A) 1.50m
(B) 1.08m
(C) 1.00m
(D) 1.10m

37. Two particles, each of mass m and charge q are attached at the ends of a light rod of length
2r. The rod is rotated at a constant angular speed  about an axis perpendicular to the rod
passing through its centre. The ratio of magnetic moment of the system to it angular
momentum
(A) q/m (B) q/2m
(C) 2q/m (D) q/4m

38. A jet of water of cross – sectional area A hits a plate normally with velocity v. The plate is
moving in the direction of the jet with velocity V. Therefore, the force exerted on the plate is
proportional to
(A) v (B) v2
(C) (v – V) (D) (v – V)2

39. A heavy cylindrical shaft (pile) of mall M is driven vertically through a distance s into the
ground by the blow of a pile – driver of mass m. The pile driver drops vertically through a
distance h into the head of the pile. The average resistance of the ground id
 m2 h   m2 h 
(A) g   2m (B) g   m  M 
M s    m  M s 
 M2 h   m2 h 
(C) g    m  M  (D) g   2  m  M 
m s    m  M s 

FIITJEE Ltd., N-W Centre, 31, 32, 33, Central Market, West Avenue Road, Punjabi Bagh, New Delhi - 26, Ph: 011-45634000
40. An optical fibre consists of a core (refractive index n1) surrounded by a cladding (refractive
index n2). A ray of light enters the fibre from air at and angle  with the fibre axis. The
maximum value of  for which the ray can propagate down the fibre is
n n
(A) sin1 1 (B) sin1 2
n2 n1
(C) sin1 n12  n22 (D) sin1 n12  n22

41. A wire ab of length 10 cm is fixed in the shape of


a sinusoidal curve as shown. The wire carries a

current of 1.2 A. In a uniform magnetic field B of
0.1 T, the wire experience a force whose magnitude
is
(A) 1.2 × 102N
(B) 4.8 × 103N
(C) zero
(D) insufficient data

42. A charge (2Q) is distributed uniformly on a spherical balloon of radius R. Another point
charge (+Q) is situated at the centre of the balloon. The balloon is now inflated to twice the
radius. Neglecting the elastic energy involved in the process, the change in total electric
energy of the system is
 Q2 Q2
(A) (B)
2 0 R 4  0 R
 Q2
(C) (D) zero
4  0 R

43. A rainbow is formed when a ray of sunlight passes through a spherical raindrop. Then the
total angle through which the ray deviates is (i and r denote the angles of incidence and of
refraction respectively)
(A) 2i – 4r (B)  + 2i – 4r
(C) 2(i – r) (D) 2( + i – 2r)

44. A series LCR circuit is connected to an ac source of frequency f and a voltage V.


At this frequency, reactance of the capacitor is 350 ohm while the resistance of the circuit is
180 ohm. Current in the circuit leads the voltage by 54° and power dissipated in the circuit is
140 watt. Therefore, voltage V is
(A) 250 volt (B) 260 volt
(C) 270 volt (D) 280 volt

45. A car has a rear view mirror of focal length 20 cm. A truck 2 m broad and 1.6 m in height is
overtaking the car with a relative speed of 15 km/hr. At the moment when the truck is 6 m
behind the car, the driver will see the image of the truck to be moving at a speed of
(A) 0.0043 m/s (B) 0.13 m/s
(C) 0.21 m/s (D) 4.17 m/s

46. In the circuit shown below the switch is closed at t = 0. For


0 < t < R (C1 + C2), the current I1 in the capacitor C1 in terms
of total current I is
C  C 
(A)  1  I (B)  2  I
 C2   C1 
 C1   C2 
(C)  I (D)  I
 C1  C2   C1  C2 

FIITJEE Ltd., N-W Centre, 31, 32, 33, Central Market, West Avenue Road, Punjabi Bagh, New Delhi - 26, Ph: 011-45634000
47. The earth is getting energy from the sum whose surface temperature is Ts and radius is R.
Let the radius of the earth be r and the distance from the sun be d. Assume the earth and
the sun both to behave as perfect black bodies and the earth is in thermal equilibrium at a
constant temperature Te. Therefore, the temperature Ts of the sun is xTe where x is
2d 2R 4d d
(A) (B) (C) (D)
R r r R

48. Imagine an atom made up of a proton and a hypothetical particle of double the mass as that
of an electron but the same charge. Apply Bohr theory to consider transitions of the
hypothetical particle to the ground state. Then, p the longest wavelength (in terms of
Rydberg constant for hydrogen atom) is
1 5 1 2
(A) (B) (C) (D)
2R 3R 3R 3R

49. The half life period of a radioactive element X is the same as the mean lifetime of another
radioactive element Y. Initially both of them have the same number of atoms. Then,
(A) X and Y have the same initial decay rate
(B) X and Y decay at the same rate always
(C) Y will decay at larger rate than X
(D) X will decay at larger rate than Y

50. A sodium atom emits a photon of wavelength 590 nm and recoils with velocity v equal to
(A) 0.029 m/s (B) 0.048 m/s
(C) 0.0023 m/s (D) data inadequate

51. Two coils wound ion the same magnetic core have inductances L1 and L2. When the two
coils are connected in series, the effective inductance is
(A) L1 + L2 (B) certainly greater than L1 + L2
(C) certainly less than L1 + L2 (D) none of the above

52. A particle of mass m and charge q moves along a diameter of a uniform spherical charge
distribution of radius R with total charge +Q. The angular frequency of the periodic motion
performed by the particle is
2 qQ 1 qQ 1 qQ 1 qQ
(A) 3
(B) 3
(C) 3
(D)
0 mR 2 0 mR 0 mR 4 0 mR3

53. A spherical body of mass m1 moving with a speed u1 collides elastically with a lighter
spherical body of mass m2 initially at rest. The maximum angle through which the heavier
body gets deflected after collision depends upon
(A) m1 and u1 only (B) m2 and u1 only
(C) m1 and m2 only (D) m1, m2 and u1 all

54. A non-conducting spherical shell of radius R surrounds a point charge q. The electric flux
through a cap of the shell of half angle  is
2q q q  2  q
(A) (B) 1  cos   (C) (D)
0 2 0 4  0 2 0

55. In a Young’s double slit experiment the intensity at a point is I where the corresponding path
difference is one sixth of the wavelength of light used. If I0 denotes the maximum intensity,
I
the ratio is equal to
I0
1 1 3 3
(A) (B) (C) (D)
4 2 2 4

FIITJEE Ltd., N-W Centre, 31, 32, 33, Central Market, West Avenue Road, Punjabi Bagh, New Delhi - 26, Ph: 011-45634000
56. A charge +q is placed at a distance of d from a point O. A conducting body surrounds point
O such that q remains outside. The electric field at O due to the induced charge is
(A) zero
1 q
(B) directed towards the charge q
4  0 d2
1 q
(C) directed away from the charge q
4  0 d2
(D) data insufficient

57. A coaxial cable consists of two thin cylindrical conducting shells of radii a and b (a < b). The
inductance per unit length of the cable is
 a  b  a  b  b
(A) 0 (B) 0 ln   (C) 0 ln   (D) 0 ln  
2 a 4  b  4  a  2  a 

58. Two coherent sources of light S1 and S2, equidistant from the
origin, are separated by a distance 2 as shown. They emit
light of wavelength . Interference is observed on a screen
placed along the circle of large radius R. Point P is seen to be
a point of constructive interference. Then, angle  (other than
0° and 90°) is
(A) 45°
(B) 30°
(C) 60°
(D) not possible in the first quadrant

59. If a current of 2 A passing through a certain electrolyte for t minutes librates 1 gram of
oxygen, then t is about
(A) 6000 (B) 100 (C) 50 (D) 25

60. A polarized light is incident on a Polaroid. Let I0 be the intensity of light transmitted by this
Polaroid. Now, a very large number (say N) of polaroids is placed in a row with their axes
displaced through a small angle  successively. If the last polarioid is crossed to the first one,
the intensity of light transmitted by the last Polaroid is about
l I
(A) zero (B) 0 (C) I0 (D) 0
2 N

61. Which of the following statement/s is/are correct in case of a resistance in a resistance box
used in a laboratory?
(A) The resistance is prepared using tungsten or nichrome wire
(B) The resistance is prepared using manganin wire
(C) Half of the length of the resistance wire is would clockwise and the remaining half
anticlockwise just to accommodate the whole length in a small space
(D) Half of the length of the resistance wire is wound clockwise and the remaining half
anticlockwise to make the inductive effect zero.

62. In a certain experiment density of the material of a small metallic cylindrical tube of a given
mass is to be determined. Its length is about 3 cm, outer diameter more than about 1 cm and
wall thickness about 2mm; the flat base being a little thicker than 2 mm. Which of the
following set/s of apparatus can be used to determine the volume of the tube accurately?
(A) Water and a measuring cylinder
(B) Water, a measuring cylinder and a micrometer screw gauge
(C) An overflow vessel, a measuring cylinder and water
(D) Only vernier callipers

FIITJEE Ltd., N-W Centre, 31, 32, 33, Central Market, West Avenue Road, Punjabi Bagh, New Delhi - 26, Ph: 011-45634000
63. An object and a screen are separated by a distance D. A convex lens of focal length f such
that 4f < D, is moved between the object and the screen to get two sharp images. If the two
positions of the lens are separated by a distance L, then
(A) L is equal to D D  4f 
(B) object distance in one position is numerically equal to image distance in the other
position

(C) the ratio of sizes of the two images is


D  L 
D  L 
2
D  L 
(D) the ratio of sizes of the two images is 2
D  L 
64. A transistor (pnp or npn) can be used as
(A) an amplifier (B) an oscillator
(C) a switch (D) a current source

65. Which of the following is/are the unit/s of magnetic field?


(A) tesla (B) Newton/ampere-meter
(C) weber/meter2 (D) volt-second/meter2

66. The inductance of a solenoid varies


(A) directly as the area of cross section
(B) directly as the square of the number of turns
(C) inversely as the length of the solenoid
(D) directly as the volume enclosed by the solenoid

67. Which of the following statement/s in case of a thermodynamic process is/are correct? (The
symbols carry their usual meaning
(A)  Eint =W indicates an adiabatic process (B)  Eint = Q suggests an isochoric process
(C)  Eint = 0 is true for a cyclic process (D)  Eint = – W indicates an adiabatic process

68. With a rise of temperature


(A) surface tension of water decreases (B) viscosity of water decreases
(C) viscosity of air decreases (D) viscosity of air increases

69. Which of the following statement/s are correct in case of a source of emf (such a primary
cell)?
(A) Inside the cell there always exist an electrostatic field and a non – electrostatic field of
equal magnitude directed opposite to it
(B) Potential difference is the work of an electrostatic field whereas electromotive force is the
work of a non –electrostatic field
(C) Under certain condition current can flow from positive terminal to negative terminal within
the cell
(D) When an external resistance is connected to the cell, the electrostatic field inside the cell
decreases in magnitude compared to the non – electrostatic field.
70. When photons each with energy 4.25 eV strike the surface of a metal A, the photoelectrons
given out have maximum kinetic energy TA and the corresponding de Broglie wave length is
A . When another metal surface B is irradiated with photons each with energy 4.70 eV, the
corresponding maximum kinetic energy TB is 1.50 eV less than TA. If the de Broglie wave
length B of these photoelectrons is twice that of A , then
(A) work function of metal A is 2.25 eV
(B) work function of metal A is 4.20 eV
(C) TA = 2.0 eV
(D) the radiation incident on metal A has a wavelength 292 nm.

FIITJEE Ltd., N-W Centre, 31, 32, 33, Central Market, West Avenue Road, Punjabi Bagh, New Delhi - 26, Ph: 011-45634000
INDIAN ASSOCIATION OF PHYSICS TEACHERS

NATIONAL STANDARD EXAMINATION IN PHYSICS 2016-17

Date of Examination: 27TH November, 2016

Time: 0830 to 1030 Hrs

Q. Paper Code: P162

Write the question paper code mentioned above on YOUR answer sheet (in the space
provided), otherwise you answer sheet will NOT be assessed. Note that the same Q.P.
Code appears on each page of the question paper.

Instruction to Candidates –

1. Use of mobile phones, smartphones, ipads during examination is STRICTLY PROHIBITED.


2. In addition to this question paper, you are given answer sheet along with Candidate’s copy.
3. On the answer sheet, make all the entries carefully in the space provided ONLY in BLOCK
CAPITALS as well as by properly darkening the appropriate bubbles.
Incomplete/incorrect/carelessly filled information may disqualify your candidature.
4. On the answer sheet, use only BLUE or BLACK BALL POINT PEN for making entries and
filling the bubbles.
5. The email Id and date birth entered in the answer sheet will be you login credentials for
accessing performance report. Please take care while entering.
6. Question paper has two parts. In part A1(Q. Nos. 1 to 60) each question has four
alternatives, out of which only one is correct. Choose the correct alternative and fill the
appropriate bubble, as shown.
Q. No. 22 a c d

In part A2 (Q. Nos. 61 to 70) each question has four alternatives out of which any number
of alternatives (1, 2, 3 or 4) may be correct. You have to choose ALL correct alternatives
and fill the appropriate bubbles, as shown.
Q. No. 64 a c

7. For Part A1, each correct answer gets 3 marks whereas 1 mark will be deducted for each
wrong answer. In Part A2, you get 6 marks if all the correct alternatives are marked.
No negative marks in this part.
8. Any rough work should be done only in the space provided.
9. Use of non – programmable calculator is allowed.
10. No candidate should leave the examination hall before the completion of the examination.
11. After submitting your answer paper, take away the Candidate’s copy for your reference.

Please DO NOT make any mark other than filling the appropriate bubbles properly in the space
provided on the answer sheet.
Answer sheets are evaluated using machine, hence CHANGE OF ENTRY IS NOT ALLOWED.
Scratching or overwriting may result in a wrong score.

DO NOT WRITE ON THE BACK SIDE OF THE ANSWER SHEET.

FIITJEE Ltd., Punjabi Bagh Delhi Centre, 31-32-33, Central Market, West Avenue Road, Punjabi Bagh (West), New Delhi - 110026, Ph: 011-45634000
Instructions to Candidates (continued) –

Read the following instructions after submitting the answer sheet.

12. Comments regarding this question paper, if any, may be sent by email only to
iapt.nse@gmail.com till 29th November, 2016.

13. The answers/solutons to this question paper will be available on out website –
www.iapt.org.in by 2nd December, 2016.

14. CERTIFICATES AND AWARDS –


Following certificates are awarded by the IAPT to students successful in NSEs
(i) Certificates to “centre Top 10%” students
(ii) Merit Certificates to “Statewise Top 1%” students
(iii) Merit Certificates and a book prize to “National Top 1%” students.

15. Result sheets can be downloaded from our website in the month of February. The “Centre
Top 10%” certificates will be dispatched to the prof-in-charge of the centre by February,
2017.

16. List of students (with centre number and roll number only) having score above MAS will be
displayed on our website (www.iapt.org.in) by 22nd December, 2016. See the Eligibility
Clause in the Student’s brochure on our website.

17. Students eligible for the INO Examination on the basis of selection criteria mentioned in
Student’s brochure will be informed accordingly.

Physical constants you may need …..

Magnitude of charge on electron e = 1.60 × 10–19 C Mass of electron me = 9.10 × 10–31 kg


–34
Universal gas constant R = 8.31 J/mol K Planck constant h = 6.62 × 10 Js
–23
Stefan constant  = 5.67 × 10–8 W/m2 K4 Boltzmann constant k = 1.38 × 10 J/K
–27
Mass of proton mp = 1.67 × 10 kg Faraday constant = 96,500 C/mol

Boiling point of nitrogen = 77.4 K Boiling point of oxygen = 90.19 K

Boiling point of hydrogen = 20.3 K Boiling point of helium = 4.2 K

Universal gravitational constant G = 6.67 × 10–11 Permittivity of free space 0 = 8.85 × 10–12 C2/Nm2
Nm2/Kg2

FIITJEE Ltd., Punjabi Bagh Delhi Centre, 31-32-33, Central Market, West Avenue Road, Punjabi Bagh (West), New Delhi - 110026, Ph: 011-45634000
INDIAN ASSOCIATION OF PHYSICS TEACHERS
NATIONAL STANDARD EXAMINATION IN PHYSICS 2016-17
Total Time: 120 minutes (A-1 and A-2)

A-1

ONLY ONE OUT OF FOUR OPTIONS IS CORRECT

1. The breakdown field for air is about 2 × 106 volt/m. Therefore, the maximum charge that can
be placed on a sphere of diameter 10 cm is
(A) 2.0 × 10–4 C (B) 5.6 × 10–7 C
–2
(C) 5.6 × 10 C (D) 2.0 × 102 C

2. A wire in the shape of square frame carries a current I and produces a magnetic field Bs at
its centre. Now the wire is bent in the shape of a circle and carries the same current. If Bc is
the magnetic field produced at the centre of the circular coil, then Bs/Bc is
(A) 82 (B) 82/ 2
(C) 8 2 / 2 (D) 8  2

3. A solid wooden block with a uniform cross section is floating in water (density w) with a
height h1 below water. Now a flat slab of stone is placed over the wooden block but the block
still flats with a height h2 below water. Afterwards the stone is removed from the top and
pasted at the bottom of the wooden block. The wooden block now floats with a height h3
under water. Therefore, the density of the stone is
h  h1 h  h3
(A) 2 w (B) 2 w
h3  h1 h2  h1
h  h1 h3
(C) 2 w (D) w
h2  h3 h2  h1

4. Two wires made of the same material, one thick and the other thin, are connected to form a
composite wire. The composite wire is subjected to some tension. A wave travelling along
the wire crosses the junction point. The characteristic that undergoes a change at the
junction point is
(A) Frequency only
(B) Speed of propagation only.
(C) Wavelength only.
(D) The speed of propagation as well as the wavelength.

5. Ultraviolet light of wavelength 300 nm and intensity 1 W/m2 falls on the surface of a
photosensitive material. If one percent of the incident photons produce photoelectrons then
the number of photoelectrons emitted per second from an area of 1 cm2 of the surface is
nearly
(A) 1.51 × 1013 (B) 1.51 × 1012
13
(C) 4.12 × 10 (D) 2.13 × 1011

6. At a certain height h above the surface of the earth the change in the value of acceleration
due to gravity (g) is the same as that at a depth x below the surface. Assuming h and x to be
enough small compared to the radius of the earth, x : h is
(A) 1 : 1 (B) 2 : 1
(C) 1 : 2 (D) 1 : 4

FIITJEE Ltd., Punjabi Bagh Delhi Centre, 31-32-33, Central Market, West Avenue Road, Punjabi Bagh (West), New Delhi - 110026, Ph: 011-45634000
7. Two point masses m1 and m2 are connected at the ends of a light rigid rod of length . The
moment of inertia of the system about n axis through their centre of mass and perpendicular
to the rod is
1  m1m2  2  m1m2  2
(A)   (B)  
2  m1  m2   m1  m2 
 m  m2  2
(C)  m1  m2   2 (D) [m12  m22 ]  1 
 m1m2 

8. Two particles of masses m and M are initially at rest and infinitely separated. At a later
instant when they are at a finite distance d from each other, their relative velocity of
approach is
1 1
 Gm  2  2G(m  M)  2
(A)   (B)  
 2d   d 
1 1
 G(m  M)  2  2GM  2
(C)   (D)  
 2d   d 

9. Two blocks of masses m and 2m are placed on a smooth


horizontal surface as shown. In the first case only a force
f1 is applied from left. Later on only a force f2 is applied
form right. If the force acting at the interface of the two
blocks in the two cases is the same, then f1 : f2 is
(A) 1 : 1 (B) 1 : 2
(C) 2 : 1 (D) 1 : 3

10. A ball A of mass 1 kg moving at a speed of 5 m/s strikes tangentially another ball B initially at
rest. The ball A then moves at right angles to its initial direction at a speed of 4 m/s. If the
collision is elastic, the mass (in kg) of ball B and its momentum after collision (in kg-m/s)
respectively are (approximately)
(A) 1.2 and 1.8 (B) 2.2 and 3.3
(C) 4.6 and 6.4 (D) 6.2 and 9.1

Group of Q. Nos. 11 to 14 is based on the following paragraph.

A nichrome wire AB, 100 cm long and of uniform cross section is mounted on a meter scale,
the points a and B coinciding with 0 cm and 100 cm marks respectively. The wire has a
resistance S = 50 ohm. Any point C along this wire, between A and B is called a variable
point to which one end of an electrical element is connected. In the following questions this
arrangement will be referred to as ‘wire AB’.

11. The emf of a battery is determined using the


following circuit with ‘wire AB’. The galvanometer
shows zero deflection when one of its terminals is
connected to point C. If the internal resistance of the
battery is 4 ohm, its emf is

(A) 3.75 volt (B) 4.05 volt


(C) 2.50 volt (D) 9.0 volt

FIITJEE Ltd., Punjabi Bagh Delhi Centre, 31-32-33, Central Market, West Avenue Road, Punjabi Bagh (West), New Delhi - 110026, Ph: 011-45634000
12. In the adjacent circuit arrangement it is found that
deflection in the galvanometer is 10 divisions. Also
the voltage across the ‘wire AB’ is equal to that
across the galvanometer. Therefore, the current
sensitivity of the galvanometer is about
(A) 0.050 div/A (B) 0.066 div/A
(C) 0.010 div/A (D) data insufficient

13. The ‘wire AB’ is now a part of the adjacent circuit.


With the resistors P = 50  and Q = 100 , the
null point is obtained at C where AC = 33 cm.
When the resistors are interchanged, the null
point is found at C with AC = 67 cm. The
systematic error in this experiment seems to be
due to non-coincidence of A and B with 0 cm
mark and 100 cm mark respectively. If these end
errors are equivalent to ‘a’ cm and ‘b’ cm
respectively, then they are
(A) 0 and 1 (B) 1 and 0
(C) 0.33 and 0.33 (D) 1 and 1

14. In the adjacent circuit a resistance R is used. Initially


with ‘wire AB’ not in the circuit, the galvanometer
shows a deflection of d divisions. Now, the ‘wire AB’
is connected parallel to the galvanometer and the
galvanometer shows a deflection nearly d/2 divisions.
Therefore
(A) R = G (B) R << G
SG
(C) R >> G (D) R 
SG

15. Consider a relation connecting three physical quantities A, B and C given by A = BnCm. The
dimensions of A, B and C are ‘LT], [L2T–1] and [LT2] respectively. Therefore, the exponents n
and m have values
(A) 2/3 and 1/3 (B) 2 and 3
(C) 4/5 and –1/5 (D) 1/5 and 3/5

16. Two identical rooms in a house are connected by an open doorway. The temperature in the
two rooms are maintained at two different values. Therefore,
(A) The room with higher temperature contains more amount of air.
(B) The room with lower temperature contains more amount of air.
(C) Both the rooms contain the same amount of air.
(D) The room with higher pressure contains more amount of air.

17. A vibrator of frequency f is placed near one of a long cylindrical tube. The tube is fitted with a
movable piston at the other end. An observer listens to the sound through a side opening.
As the piston is moved through 8.75 cm, the intensity of sound recorded by the observer
changes form a maximum to a minimum. If the speed of sound in air is 350 m/s, the
frequency f is
(A) 500 Hz (B) 1000 Hz
(C) 2000 Hz (D) 4000 Hz

FIITJEE Ltd., Punjabi Bagh Delhi Centre, 31-32-33, Central Market, West Avenue Road, Punjabi Bagh (West), New Delhi - 110026, Ph: 011-45634000
18. A heavy metal block is dragged along a round horizontal surface at a constant speed of 20
km/hr. The coefficient of friction between the block and the surface is 0.6. The block is made
of a material whose specific heat is 0.1 cal/g-oC can absorbs 25% of heat generated due to
friction. If the block is dragged for 10 min, the rise in temperature of the block is about(g = 10
m/s2)
(A) 12oC (B) 50oC
o
(C) 210 C (D) data insufficient

19. A gas is made to undergo a change of state from an initial state to a final state along
different paths by adiabatic process only. Therefore
(A) the work done is different for different paths
(B) the work done is the same for all paths
(C) there is no work done as there is no transfer of energy
(D) the total internal energy of the system will not change

20. Vectors A, B, C lie in XY plane and their resultant is R. The magnitudes of A, B and R
are100, 200 and 200 respectively. The angles made by these vectors with the positive
direction of X axis are 60o, 150o and 90o respectively. Therefore, the magnitude and the
angle made by C with positive direction of X axis respectively are
(A) 75, 315o (B) 110, 45o
(C) 156, 240o (D) 124, 62o

21. Two particles A and B are situated 10 m apart along X axis, B being farther of A, at t = 0.
Particle A is moving at 0.75 m/s parallel to +Y axis while B at 1 m/s along –X axis. After a
time t they come closes to each other. Therefore, t is
(A) 4.8 s (B) 6.4 s
(C) 6.0 s (D) 3.2 s

22. Out of the following differential equations, on that correctly represents the motion of a
second’s pendulum is
d2 x x d2 x x
(A) 2   0 (B) 2  2  0
dt  dt 
2 2
d x d x
(C) 2  x  0 (D) 2  2 x  0
dt dt

23. A block of mass 2 kg drops vertically from a height of 0.4 m onto a spring whose force
constant K is 1960 N/m. Therefore, the maximum compression of the spring is
(A) 0.40 m (B) 0.25 m
(C) 0.80 m (D) 0.1 m

24. Two blocks of masses m1 = 8 kg and m2 = 7 kg are connected by a light string passing over
a light frictionless pulley. The mass m1 is at rest on the inclined plane and mass m2 hangs
vertically. The angle of inclination is 30o. Therefore, the force of friction acting on m1 is
(A) 30 N up the plane (B) 30 N down the plane
(C) 40 N up the plane (D) 40 N down the plane

25. Two factories are sounding their sirens at 400 Hz each. A man walks from one factor
towards the other at a speed of 2 m/s, the speed of sound is 320 m/s. The number of beats
heard per second by the man is
(A) 6 (B) 5
(C) 2.5 (D) 7.5

FIITJEE Ltd., Punjabi Bagh Delhi Centre, 31-32-33, Central Market, West Avenue Road, Punjabi Bagh (West), New Delhi - 110026, Ph: 011-45634000
26. The adjacent figure shows I – V characteristics of a silicon
diode. In this connection three statements are made – (I) the
region OC corresponds to reverse bias of the diode, (II) the
voltage at point A is about 0.2 volt and (III) different scales
have been used along +ve and –ve directions of Y axis.
Therefore,

(A) only statement (I) is correct (B) only statements (I) and (II) are correct
(C) only statements (I) and (III) are correct (D) all statements (I), (II) and (III) are correct

27. Two identical lenses made of the same material of refractive index 1.5 have the focal length
12 cm. These lenses are kept in contact and immersed in a liquid of refractive index 1.35.
The combination behaves as
(A) convex lens of focal length 27 cm (B) concave lens of focal length 6 cm
(C) convex lens of focal length 9 cm (D) convex lens of focal length 6 cm

28. A cup of water is placed in a car moving at a constant acceleration a to the left. Inside the
water is a small air bubble. This figure that correctly shows the shape of the water surface
and the direction of motion of the air bubble is:

(A) A (B) B
(C) C (D) D

29. A sphere of radius R made up of Styrofoam(light polystyrene material) has a cavity of radius
R/2. The centre of the cavity is situated at a distance of R/2 from the centre of the Styrofoam
sphere. The cavity is filled with a solid material of density five times that of Styrofoam. Now,
the centre of mass is seen to be located at a distance x from the centre of Styrofoam sphere,
therefore x is
(A) R/2 (B) R/3
(C) R/4 (D) R/6

30. A resistor R is connected to a parallel combination of two identical batteries each with emf E
and an internal resistance r. The potential drop across the resistance R is
2ER ER
(A) (B)
2R  r R  2r
ER 2ER
(C) (D)
2R  r R  2r

31. The critical angle between a certain transparent medium and air is . A ray of light traveling
through air enters the medium at an angle of incidence equal to its polarizing angle .
Therefore, the angle of refraction is
(A) tan–1 (sin) (B) tan–1 (sin)
(C) sin–1 (tan) (D) sin–1 (tan)

FIITJEE Ltd., Punjabi Bagh Delhi Centre, 31-32-33, Central Market, West Avenue Road, Punjabi Bagh (West), New Delhi - 110026, Ph: 011-45634000
32. If a copper wire is stretched to make its radius decrease by 0.1%, the percentage change in
its resistance is approximately
(A) -0.4% (B) +0.8%
(C) +0.4% (D) +0.2%

33. Consider a manual camera with a lens having a focal length of 5 cm. It is focused at infinity.
For catching the picture of an object at a distance of 30 cm, one would
(A) move the lens out by about 1 cm (B) move the lens out by about 5 cm
(C) move the lens in by about 1 cm (D) find it impossible to catch the picture

34. Initially interference is observed with the entire experimental set up inside a chamber filled
with air. Now the chamber is evacuated. With the same source of light used, a careful
observer will find that
(A) the interference pattern is almost absent as it is very much diffused
(B) there is no change in the interference pattern
(C) the fringe width is slightly decreased
(D) the fringe width is slightly increased

35. Two identical loudspeakers, placed close to each other inside a room, are supplied with the
same sinusoidal voltage. One can imagine a pattern around the loudspeakers with areas of
increased and decreased sound intensity alternately located. Which of the following actions
will NOT change the locations of these areas?
(A) moving one of the speakers
(B) changing the amplitude of the signal voltage
(C) changing the frequency of the signal voltage
(D) replacing the air in the room with a different gas

36. A particle at rest explodes into two fragments of masses m1 and m2(m1 > m2) which move
apart with non zero velocities. If 1 and 2 are their de Broglie wavelengths respectively, then
(A) 1 > 2 (B) 1 < 2
(C) 1 = 2 (D) data insufficient

37. Two particles of masses m1 and m2 carry identical charges. Starting from rest they are
accelerated through the same potential difference. Then they enter into a region of uniform
magnetic field and move along circular paths of radii R1 and R2 respectively. Therefore, the
ratio of their masses m1 : m2 is
(A) R1 : R2 (B) R12 : R 22
(C) R22 : R12 (D) R1 : R2

38. A fixed horizontal wire M carries 200 A current. Another wire N running parallel to M carries
a current I and remains suspended in a vertical plane below M at a distance of 20 mm. Both
the wires have a linear mass density 10–2 kg/m. Therefore, the current I is
(A) 20 A (B) 4.9 A
(C) 49A (D) 200 A

39. An unpolarized light of intensity 32 W/m2 passes through three polarizers, such that the
transmission axis of last polarizer is crossed with that of the first. If the intensity of emergent
light is 3 W/m2, then the angle between the transmission axes of the first two polarizers is
(A) 30o (B) 19o
(C) 45o (D) 90o+
40. An electron is injected directly towards the centre of a large metal plate having a uniform
surface charge density of –2.0 x 10–6 C/m2. The initial kinetic energy of the election is
1.6 x 10–17 J. The electron is observed to stop as it just reaches the plate. Therefore, the
distance between the plate and the point from where the electron was injected is
(A) 4.4 x 10–4 m (B) 4.4 m
(C) 4.4 x 10–2 (D) data insufficient

FIITJEE Ltd., Punjabi Bagh Delhi Centre, 31-32-33, Central Market, West Avenue Road, Punjabi Bagh (West), New Delhi - 110026, Ph: 011-45634000
41. Graphs (drawn with the same scale) showing the variation of pressure with volume for a
certain gas undergoing four different cyclic processes A, B, C and D are given below.
The cyclic process in which the gas performs the greatest amount of work is

(A) A (B) B
(C) C (D) D

42. A rectangular metal tank filled with a certain liquid is as


shown in the figure. The observer, whose eye is in level
with the top of the tank, can just see the corner E of the
tank. Therefore, the refractive index of the liquid is

(A) 1.67 (B) 1.50


(C) 1.33 (D) 1.25

43. As shown in the figure, a block of mass m is suspended from a


support with the help of a system of identical springs. The force
constant of each spring k. Therefore, the frequency of
oscillations of the block is
1 3k 1 2k
(A) (B)
2 2m 2 3m
1 5k 1 6k
(C) (D)
2 6m 2 5m

44. The impedance (Z) of three electrical components e1, e2, and e3
has frequency (f) dependence as shown by the following three
curves.
The three components e1, e2, e3 are respectively
(A) R, L, C (B) R, C, L
(C) L, R, C (D) C, R, L

45. The half life period of a radioactive element E1 is equal to the main lifetime of another
radioactive element E2. Initially both the elements have the same number of atoms.
Therefore.
(A) E2 will decay faster (B) E1 will decay faster
(C) E1 and E2 will decay at the same rate (D) Data insufficient

46. A simple pendulum has a bon of mass m and a light string of length I. The string is replaced
by a uniform rod of mass m and of the same length I. The time period of this pendulum is
1/2 1/2
(A) 2 I / g  (B) 2  8I/ 9g 
1/2 1/2
(C) 2  9I / 8g (D) 2  2 / 3g

FIITJEE Ltd., Punjabi Bagh Delhi Centre, 31-32-33, Central Market, West Avenue Road, Punjabi Bagh (West), New Delhi - 110026, Ph: 011-45634000
47. A tennis ball is released from a height and allowed
to fall onto a hard surface. The adjacent graph
shows the variation of velocity of the ball with time
from the instant of release. The point of upward
maximum velocity of the ball is indicated by point
(A) A (B) B
(C) C (D) D

48. The diagram shows an oscillating block connected to


two identical springs. The frequency of oscillations can
be increased substantially by
(A) Increasing the amplitude of the oscillations
(B) Fixing an extra mass of the block
(C) Using softer pair of springs
(D) Using harder pair of springs

49. The variation of velocity with time of a toy car moving


along a straight line is an in adjacent figure. Which of the
following graphs correctly represents the variation of
acceleration with time for the toy car?

(A) (B)

(C) (D)

50. An ac source (sinusoidal source with frequency 50 Hz) is connected in series with a
rectifying diode, a 100 resistor, a 1000F capacitor and a milliammeter. After some time
the millimmater reads zero. The voltage measured across the capacitor with a dc voltmeter
is
(A) the peak voltage of the ac source
(B) rms voltage of the ac source.
(C) average voltage of the ac source over a half cycle.
(D) average voltage of the ac source over a full cycle.

FIITJEE Ltd., Punjabi Bagh Delhi Centre, 31-32-33, Central Market, West Avenue Road, Punjabi Bagh (West), New Delhi - 110026, Ph: 011-45634000
51. The frequency of the sound produced by a siren increases from 400 Hz to 1200 Hz while its
amplitude remains the same. Therefore, the ratio of the intensity of the 1200 Hz wave to that
of the 400 Hz wave is
(A) 1 : 1 (B) 3 : 1
(C) 1 : 9 (D) 9 : 1

52. The fundamental frequency of the output of a bridge rectifier driven by ac mains is
(A) 50 Hz (B) zero
(C) 100 Hz (D) 25 Hz

53. The force of attraction between the positively charged nucleus and the electron in hydrogen
e2
atom is given by f  k 2 . Assume that the nucleus is fixed. The electron, initially moving in
r
an orbit of radius R1 jumps into an orbit of smaller radius R 2 . The decrease in the total
energy of the atom is
ke2  1 1  ke2  R1 R2 
(A)    (B)   
2  R1 R 2  2  R22 R12 
ke2  1 1 ke2  R2 R1 
(C)    (D)  2  2
2  R2 R1  2  R1 R2 

54. It is observed that some of the spectral lines in hydrogen spectrum have wavelengths almost
equal to those of the spectral lines in He ion. Out of the following the transitions in He that
will make this possible is
(A) n  3 to n  1 (B) n  6 to n  4
(C) n  5 to n  3 (D) n  3 to n  2

Group of Q. No. 55 to 60 is based on the following paragraph.

A wheel of a car is made up of two parts (1)


the central metal rim, and (2) the rubber tyre.
The width of the tyre W = 16.5 cm and height
h = 10.7 cm. The rim overlaps the tyre. The
total weight of the car is 1500 kg distributed
evenly. The tyres are inflated with air to a
pressure 2.0 kg/cm2. The density of air at
pressure of 1.0 kg/cm2 The density of air at
pressure of 1.0 kg/cm2 and at room
temperature equals 1.29 g/litre. The outer
diameter of the tyre is 55.4 cm and that of the
rim is 40 cm.

Ignore the thickness of rubber and use the dimensions given here.
Note that the units mentioned above are conventional units used in everyday life.

55. Consider the following two statements about a tyre of a car.


Statement A : ‘The horizontal road surface is exactly tangential to the tyre’
Statement B : ‘The tyre is inflated with excess pressure’
Which of the following alternative is correct?
(A) Statement A is the result of statement B
(B) Statement B cannot be true
(C) Statement A cannot be true
(D) Neither of the statements A and B is true

FIITJEE Ltd., Punjabi Bagh Delhi Centre, 31-32-33, Central Market, West Avenue Road, Punjabi Bagh (West), New Delhi - 110026, Ph: 011-45634000
56. The left side front tyre was observed to be in contact with the road over a length L cm. The
value of L is
(A) 8.85 cm (B) 9.35 cm
(C) 11.36 cm (D) 10.35 cm

57. When five persons occupy the seats L increases by 2.5 cm. The average weight of a person
is
(A) 66 kg (B) 60 kg
(C) 62 kg (D) 64 kg

58. If five persons occupy the seats, the centre of the wheel is lowered by about
(A) 1 mm (B) 2 mm
(C) 3 mm (D) 4 mm

59. The mass of air in a tyre is about


(A) 24 g. (B) 49 g.
(C) 32 g. (D) 64 g.

60. The tyres of racing cars are very wide. Their width is nearly three times the above value.
This large width is for
(A) stability and acceleration (B) streamlining and acceleration
(C) streamlining and stability (D) streamlining, stability and acceleration.

A-2

In Q. No. 61 to 70 any number of options (1 or 2 or 3 or all 4) may be correct. You are to


identify all of them correctly to get 6 marks. Even if one answer identified is incorrect or one
correct answer is missed, you get zero marks.

61. Water is flowing through a vertical tube with varying


cross section as shown. The rate of flow is 52.5
ml/s. Given that speed of flow 1  0.35 m / s and
area of cross section A 2  0.5 cm2 . Which of the
following is/are true?
(A) A1  1.0 cm2 ,  2  0.70 m / s
(B) A1  1.5 cm2 ,  2  1.05 m / s
(C) h  5 cm
(D) h  10 cm

62. A simple laboratory power supply consists of a transformer, bridge rectifier and a filter
capacitor. It drives a suitable load. If due to some reason one of the diodes in the rectifier
circuit becomes open, then
(A) output voltage of power supply falls to zero.
(B) output voltage of power supply decreases to some nonzero value
(C) ac ripple in the output increases.
(D) ripple frequency decreases.

FIITJEE Ltd., Punjabi Bagh Delhi Centre, 31-32-33, Central Market, West Avenue Road, Punjabi Bagh (West), New Delhi - 110026, Ph: 011-45634000
63. Circuit A is a series LCR circuit with C A  C and L A  L . Another circuit B has CB  2C and
LB  L / 2 . Both the circuits have the same resistance and the capacitor and the inductance
are assumed to be ideal components. Each of the circuits is connected to the same
sinusoidal voltage source. Therefore,
(A) both the circuits have the same resonant frequency
(B) both the circuits carry the same peak current
(C) resonance curve for circuit A is more sharp than that for circuit B.
(D) resonance curve for circuit B is more sharp than that for circuit A.

64. The variation of acceleration with time for a


particle performing simple harmonic motion
along straight line is as in adjacent figure.
Therefore,
(A) the particle has non – zero displacement
initially.
(B) the displacement of the particle at point 1
is negative
(C) the velocity of the particle at point 2 is
positive.
(D) the potential energy at point 3 is maximum

65. Which of the following physical quantities have dimensions identical to each other?
(A) the Young’s modulus Y.
(B) 0 E2 where E is the electric field intensity and 0 is the perimittivity of free space.
B2
(C) where B is the magnetic field and  0 is the permeability of free space.
0
(D) kT where k is Bolzmann’s constant and T is the absolute temperature.

66. A small ball bearing is released at the top of a long vertical column of glycerin of height 2h.
The ball bearing falls through a height h in time t1 and then the remaining height with the
terminal velocity in time t 2 . Let W1 and W2 be the work done against viscous drag over
these heights. Therefore,
(A) t1  t 2 (B) t1  t 2
(C) W1  W2 (D) W1  W2

67. A particle moves in XY plane according to the relations x  kt and y  kt 1  pt  where k


and p are positive constants and t is time. Therefore,
(A) the trajectory of the particle is a parabola
(B) the particle has a constant velocity along X axis.
(C) the force acting on the particle remains in the same direction even if both k and p are
negative constants.
(D) the particle has a constant acceleration along – Y axis.

68. A charge q is situated at the origin. Let EA ,EB and EC be the electric fields at the points.
A  2,  3,  1 , B  1,  2, 4  and C  2,  4, 1 . Therefore.
(A) EA  EB
(B) no work is done in moving a test charge q0 from B to C.
(C) 2 E A  3 EB
(D) EB  EC

FIITJEE Ltd., Punjabi Bagh Delhi Centre, 31-32-33, Central Market, West Avenue Road, Punjabi Bagh (West), New Delhi - 110026, Ph: 011-45634000
69. A uniform spherical charge distribution of radius R produces electric fields E1 and E2 at two
points at distances r1 and r2 respectively from the centre of the distribution. Out of the
E
following the possible expression/s for 1 is/are
E2
2
r r 
(A) 2 (B)  1 
r1  r2 
R3 r r2
(C) (D) 1 23
r12 r2 R

70. A metallic wire of length l is held between two supports under some tension. The wire is
cooled through  ' . Let Y be the Young’s modulus,  the density and  the thermal
coefficient of linear expansion of the material of the wire. Therefore, the frequency of
oscillations of the wire varies as
(A) Y (B) z
1 
(C) (D)
l 

FIITJEE Ltd., Punjabi Bagh Delhi Centre, 31-32-33, Central Market, West Avenue Road, Punjabi Bagh (West), New Delhi - 110026, Ph: 011-45634000
INDIAN ASSOCIATION OF PHYSICS TEACHERS
NATIONAL STANDARD EXAMINATION IN PHYSICS 2018-19
Date of Examination: 25th November, 2018
Time: 0830 to 1030 hrs

Q. Paper Code: P160

Write the question paper code mentioned above on YOUR answer sheet (in the space
provided), otherwise your answer sheet will NOT be assessed.
Note that the same Q.P. Code appears on each page of the question paper.

Instructions to Candidates –
1. Use of mobile phones, smartphones, ipads during examination is STRICTLY PROHIBITED.
2. In addition to this question paper, you are given answer sheet along with Candidate’s copy.
3. On the answer sheet, make all the entries carefully in the space provided ONLY in BLOCK
CAPITALS as well as by properly darkening the appropriate bubbles.
Incomplete/incorrect/carelessly filled information may disqualify your candidature.
4. On the answer sheet, use only BLUE or BLACK BALL POINT PEN for making entries and
filling the bubbles.
5. The email ID and date birth entered in the answer sheet will be your login credentials for
accessing performance report. Please take care while entering.
6. Question paper has two parts. In part A1(Q. Nos. 1 to 60) each question has four
alternatives, out of which only one is correct. Choose the correct alternative and fill the
appropriate bubble, as shown.
Q. No. 22 a c d

In part A2 (Q. Nos. 61 to 70) each question has four alternatives out of which any number
of alternatives (1, 2, 3 or 4) may be correct. You have to choose ALL correct alternatives
and fill the appropriate bubbles, as shown.
Q. No. 64 a c

7. For Part A1, each correct answer carries 3 marks whereas 1 mark will be deducted for each
wrong answer. In Part A2, you get 6 marks if all the correct alternatives are marked. No
negative marks in this part.
8. Any rough work should be done only in the space provided.
9. Use of non – programmable calculator is allowed.
10. No candidate should leave the examination hall before the completion of the examination.
11. After submitting your answerpaper, take away the Candidate’s copy for your reference.

Please DO NOT make any mark other than filling the appropriate bubbles
properly in the space provided on the answer sheet.
Answer sheets are evaluated using machine, hence CHANGE OF ENTRY IS NOT
ALLOWED.
Scratching or overwriting may result in a wrong score.
FIITJEE Ltd., Punjabi Bagh Delhi Centre, 31-32-33, Central Market, West Avenue Road, Punjabi Bagh (West), New Delhi - 110026, Ph: 011-45634000
P160
INDIAN ASSOCIATION OF PHYSICS TEACHERS
NATIONAL STANDARD EXAMINATION IN PHYSICS 2018-19
Total Time: 120 minutes (A-1 and A-2)

A-1

ONLY ONE OUT OF FOUR OPTIONS IS CORRECT


1. The SI unit of permeability of free space is
weber henry tesla weber
(A) (B) (C) (D)
ampere ampere ampere  meter ampere  meter

2. A uniform solid drum of radius R and mass M rolls without slipping down a plane inclined at
an angle . Its acceleration along the p lane is
1 1 2 5
(A) g sin  (B) g sin  (C) g sin  (D) g sin 
3 2 3 7

3. A particle moves according to the law x = at, y = at(1 – t) where a and  are positive

constants and t is time. The time instant at which velocity makes an angle with
4
acceleration is
4 3 2 1
(A) (B) (C) (D)
   

4. The potential energy of a particle of mass m in a conservative force field can be expressed
as U = x – y where (x, y) denote the position coordinates of the body. The acceleration of
the body is
  2  2 2  2
(A) (B) (C) (D)
m m m m

5. A constant force F applied to the lower block of mass 15 kg makes it slide between the
upper block of mass 5 kg and the table below, as shown. the coefficients of static (s) and
kinetic (k) friction between the lower block and the table are 0.5 and 0.4 respectively and
those between the two blocks are 0.3 and 0.1. The accelerations of the upper and the lower
blocks are respectively.

(A) 1.96 m/s2 and 1.96 m/s2 (B) 1.96 m/s2 and 3.92 m/s2
(C) 0.98 m/s2 and 0.49 m/s2 (D) 0.98 m/s2 and 1.96 m/s2

6. Two bodies of equal masses moving with equal speeds make a perfectly inelastic collision.
If the speed after the collision is reduced to half, the angle between their velocities of
approach is
(A) 30° (B) 60°
(C) 90° (D) 120°

FIITJEE Ltd., Punjabi Bagh Delhi Centre, 31-32-33, Central Market, West Avenue Road, Punjabi Bagh (West), New Delhi - 110026, Ph: 011-45634000
7. A student performs an experiment with a simple pendulum and records the time for
20 oscillations. If he would have recorded time for 100 oscillations, the error in the
measurement of time period would have reduced by a factor of
(A) 80 (B) 20
(C) 10 (D) 5

8. A satellite is lunched from a point close to surface of the earth (radius R) with a velocity
  0 1.5, where 0 is the velocity in a circular orbit. If the initial velocity imparted to the
satellite is horizontal, the maximum distance from the surface of the earth during its
revolution is
(A) R (B) 2R
(C) 3R (D) 4R

9. The aperture diameter of a plane-convex lens is 6 cm and its thickness is 3 mm. If the speed
of light through its material is  = 2 × 108 m/s, the focal length of the lens is
(A) 40 cm (B) 35 cm
(C) 30 cm (D) 20 cm

10. Under standard conditions of temperature and pressure a piece of ice melts completely on
heating it. Obviously the increase in internal energy of the system (ice and water) is
(A) equal to the heat given. (B) more than the heat given
(C) less than the heat given (D) zero

11. Rocket fuel is capable of giving an exhaust velocity of rel = 2.4 km/s in the absence of any
external forces. The fuel required per kg of the payload to provide an exhaust velocity of
12 km/s to the rocket is
(A) 3670 kg (B) 8000 kg
(C) 147.4 kg (D) 478.4 kg

12. A vertical spring of length 0 and force constant K is stretched by  when a mass m is
suspended from its lower end. By pulling the mass down a little the system is left off to
oscillate. The time period of oscillation is
 
(A) 2 (B) 2 0
g g
1 m   0
(C) (D) 2
2 k g

13. Let R be the radius of the earth. In general, the loss of gravitational potential energy of a
body of mass m falling from a height h to the earth surface is
R
(A) mgh (B) mgh
Rh
Rh R
(C) mgh (D) mgh
R Rh

14. The velocity of a projectile at the highest point of its trajectory is 0.4 of its velocity at a
point at half its maximum height. The angle of projection is
(A) 30° (B) 45°
(C) 60° (D) tan–1 ( 0.4 )

FIITJEE Ltd., Punjabi Bagh Delhi Centre, 31-32-33, Central Market, West Avenue Road, Punjabi Bagh (West), New Delhi - 110026, Ph: 011-45634000
15. The combination of a steel wire (length 80 cm, area of cross section 1 mm2) and an
aluminium wire (length 60 cm, area of cross section 3 mm2) joined end to end is stretched by
a tension of 160 N. If the densities of steel and aluminium are 7.8 g/cc and 2.6 g/cc
respectively then, the minimum frequency of tuning fork which can produce standing waves
in the composite wire, with the joint as a node, is
(A) 179 Hz (B) 358 Hz
(C) 88 Hz (D) 118 Hz

16. In a stationary wave


(A) all the medium particles vibrate in the same phase.
(B) all the particles between two consecutive nodes vibrate in the same phase.
(C) any two consecutive nodes vibrate in the same phase.
(D) all the particles between two consecutive antinodes vibrate in the same phase.

17. An empty earthen pitcher is kept under a water tap and starts filling with water as the tap is
opened. The pitched of the sound produced
(A) goes on decreasing.
(B) goes on increasing.
(C) first increases and then decreases after the pitcher is half filled.
(D) does not change.


18. The molar specific heat of an ideal gas in a certain thermodynamic process is where  is
T
CP
a constant. If the adiabatic exponent is   , the work done in heating the gas form t0 t
CV
nT0 is
1 (n  1)
(A) ln n (B)  ln n  RT0
 (   1)
(n  1)
(C)  ln n – ( – 1)RT0 (D) RT0
(   1)

19. An aircraft flied at a speed  from city A to city B and back in time t0. City B is to the east of
city A at a distance d. The aircraft takes time t1 for the round trip if wind blows with speed w
along AB and time t2 if the wind blows with the same speed perpendicular to AB. Then,
(A) t1 = t2 = t0 (B) t1 > t2 > t0
(C) t1 < t2 < t3 (D) t1 > t0 > t2

20. The Hubble telescope in orbit above the earth has a 2.4 m circular aperture. The telescope
has equipment for detecting ultraviolet light. The minimum angular separation between two
objects that the telescope can resolve in ultraviolet light of wavelength 95 nm is
(A) 4.83  10 8 rad (B) 4.03  10 8 rad
(C) 2.41 10 8 rad (D) 2.00  108 rad

21. A projectile is fired from ground with velocity u at an angle  with the horizontal. It would be
moving perpendicular to its initial direction of projection after a time t equal to
u sin  2u sin 
(A) (B)
g g
u u
(C) (D)
g sin  2gsin 

22. The critical angle for light passing from glass to air is minimum for the light of wavelength
(A) 0.7 m (B) 0.6 m
(C) 0.5 m (D) 0.4 m
FIITJEE Ltd., Punjabi Bagh Delhi Centre, 31-32-33, Central Market, West Avenue Road, Punjabi Bagh (West), New Delhi - 110026, Ph: 011-45634000
23. A thin hollow equiconvex lens, silvered at the back, converges a beam of light parallel to the
 4
principal axis at a distance 0.2 m. When filled with water     , the same beam will be
 3
converged at a distance of
(A) 3.75 cm (B) 3.25 cm
(C) 0.12 m (D) None of the above

24. An air bubble is situated at a distance 2.0 cm from the centre of a spherical glass paper
weight of radius 5.0 CM AND REFRACTIVE INDEX 1.5. The bubble is seen through the
nearest surface. It appears at a distance  from the centre. Therefore,  is
(A) 3.75 cm (B) 3.25 cm
(C) 2.50 cm (D) 3.80 cm

25. A student while performing experiment with a sonometer with bridges separated by a
distance of 80 cm, missed out some of the observations. However, he claimed that the three
resonant frequencies for a given tuning fork were 84, 140 and 224 Hz. The speed of
transverse waves on the wire is
(A) 33.30 m/s (B) 330.0 m/s
(C) 44.80 m/s (D) 448.0 m/s

26. Which of the following curves represents spectral distribution of energy of black body
radiation?

(A) (B)

(C) (D)

27. A Sphere and a cube having equal surface area are made of the same material. The two are
heated to the same temperature and kept in identical surrounding. The ratio of their initial
rates of cooling is

(A) 1 : 1 (B) :1
2
 
(C) :1 (D) :1
3 6

FIITJEE Ltd., Punjabi Bagh Delhi Centre, 31-32-33, Central Market, West Avenue Road, Punjabi Bagh (West), New Delhi - 110026, Ph: 011-45634000
28. Consider the diffraction pattern due to a single slit. The first maximum for a certain
monochromatic light coincides with the first minimum for red light of wavelength 660 nm. The
wavelength of the monochromatic light is
(A) 660 nm. (B) 550 nm.
(C) 440 nm. (D) 330 nm.

29. A concave lens of focal length f produces an image (1/n) times the size of the object. The
distance of the object from the lens is

(A)  n  1 f (B)
 n  1 f
n

(C)
 n  1 f (D)  n  1 f
n

30. The Sun having radius R and surface temperature T, emits radiation as a perfect emitter.
The distance of the earth from the sum is r and the radius of the earth is R e . The total
radiant power incident on the earth is
R 2eR2 T 4 R 2eR2 T 4
(A) (B)
4 r 2 r2
4 R 2eR 2T 4 R2eR2 T 4
(C) (D)
r2 r2

31. A cylinder containing water (refractive index 4/3) is covered by an equiconvex glass
(refractive index 3/2) lens of focal length 25 cm. At the mid – day when the sun is just
overhead, the image of the sun will be seen at a distance of
(A) 100 cm (B) 50 cm
(C) 37.5 cm (D) 25 cm

32. A rectangular loop carrying a current is placed in a uniform magnetic field. The net force
acting on the loop.
(A) depends on the direction and magnitude of the current.
(B) depends on the direction and magnitude of the magnetic field.
(C) depends on the area of the loop.
(D) is zero.

33. The capacitor in the circuit shown below


carries a charge of 30 C at a certain time
instant. The rate at which energy is being
dissipated in the 3k resistor at that instant
is
(A) 4mW
(B) 9mW
(C) 27mW
(D) 48mW

34. A hollow conducting sphere of radius 15 cm has a uniform surface charge density
3.2 C / m2 . When a point charge q is placed at the centre of the sphere, the electric field at
25 cm from the centre just reverses its direction keeping the magnitude the same. Therefore,
q is
(A) 0.91C (B) 0.91C
(C) 1.81C (D) 1.81C

FIITJEE Ltd., Punjabi Bagh Delhi Centre, 31-32-33, Central Market, West Avenue Road, Punjabi Bagh (West), New Delhi - 110026, Ph: 011-45634000
35. An electron (e) and proton (p) are situated on the straight line as shown below. The
directions of the electric field at the point 1, 2 and 3 respectively, are shown as

(A)
(B)
(C)
(D)

36. In the circuit shown R1  R2 . The reading in


the galvanometer is the same with switch S
open or closed. Then.
(A) I1  IG
(B) I2  IG
(C) I3  IG
(D) I4  IG

37. A thin wire of length 1 m is placed perpendicular to the XY plane. If it is moved with velocity
 
  4iˆ  ˆj m / s in the region of magnetic induction B  ˆi  4ˆj Wb / m2 . The potential
difference developed between the ends of the wire is
(A) zero (B) 3V
(C) 15 V (D) 17V

38. A steel cooking pan has copper coating at its bottom. The thickness of copper coating is half
the thickness of steel bottom. The conductivity of copper is three times that of steel. If the
temperature of blue flame is 119o C and that of the interior of the cooking pan is 91o C , then
the temperature at the interface between the steel bottom and the copper coating in the
steady state is
(A) 980C (B) 103oC
o
(C) 115 C (D) 108oC

39. The total capacitance between points A and B in the


arrangement shown below is:
(A) 28 F
34
(B) F
7
(C) 23  F
34
(D) F
3

FIITJEE Ltd., Punjabi Bagh Delhi Centre, 31-32-33, Central Market, West Avenue Road, Punjabi Bagh (West), New Delhi - 110026, Ph: 011-45634000
40. A fiber sheet of thickness 1 mm and a mice sheet of thickness 2 mm are introduced between
two metallic parallel plates to form a capacitor. Given that the dielectric strength of fibre is
6400 kV/m and the dielectric constants of fiber and mica are 2.5 and 8 respectively, the
electric field inside the mica sheet just at the breakdown of fiber will be
(A) 200 kV/m (B) 2048 kV/m
(C) 3200 kV/m (D) 6400 kV/m

41. The position vector of a point mass is expressed as r  atiˆ  bt 2 ˆj . The trajectory of the
particle is
(A) a straight line (B) a parabola
(C) a hyperbola (D) none of the above

42. In a series LCR circuit fed with an alternating emf E = E0 sin t,
(A) the voltage across L is in phase with the applied emf E
(B) the voltage across C is the phase with the applied emf E
(C) the voltage across R is in phase with the applied emf E
(D) the voltage across L, C and R are all in phase with the applied emf E

43. A conducting wire is bent in the form of a n sided regular polygon enclosed by a circle of
radius R. The magnetic field produced at its center by a current I flowing through the wire is
 
oi sin n 0i cos n
(A) (B)
2R  2R 
n n
 
0i tan n 0i cot n
(C) (D)
2R  2R 
n n

44. The effective resistance between points A and B in the


circuit arrangement shown below is:
(A) 14
(B) 15 
(C) 30 
(D) none of the above

45. The magnetic dipole moment of an electron is the S state of hydrogen atom revolving in a
circular orbit of radius 0.0527 nm with a speed 2.2  106 is
(A) 4.64  1024 Am2 (B) 9.28  10 24 Am2
(C) 18.56  1024 AM2 (D) 2.32  1024 Am2

46. A steel cable hanging vertically can support a maximum load W. The cable is cut to exactly
half of its original length, the maximum load that it can support now is
W
(A) W (B)
2
W
(C) 2W (D) more than but less than W
2

FIITJEE Ltd., Punjabi Bagh Delhi Centre, 31-32-33, Central Market, West Avenue Road, Punjabi Bagh (West), New Delhi - 110026, Ph: 011-45634000
47. The string AB and AC each of length 40 cm, connect a ball of
mass 200 g to a vertical shaft as shown. When the shaft
rotates at a constant angular speed , the ball travels in a
horizontal circle with the strings inclined at y = 300 to the shaft
. If the tension in the string AC in 4 N, that in the string AB
and the angular speed  respectively, are
(A) 6.26 N and 11.32 rad/s
(B) 7.92 N and 14.32 rad/s
(C) 7.92 N and 11.32 rad/s
(D) 6.26 N and 14.32 rad/s

48. A tightly wound long solenoid carries a current 5 A. An electron shot perpendicular to the
solenoid axis inside it revolves at a frequency 108 rev/s. The number of turns per meter
length of the solenoid is
(A) 57 (B) 176
(C) 569 (D) 352

49. The same alternating voltage   V0 sin  t  is applied in both the LCR circuits shown
below. The current through the resistance R at resonance is

(A) maximum in Fig (1) and maximum in Fig (2)


(B) minimum in Fig (1) and maximum in Fig (2)
(C) maximum in Fig (1) and minimum in Fig (2)
(D) minimum in Fig (1) and minimum in Fig (2)

50. The switch S in the circuit shown is closed for a long time
and then opened at time t = 0. The current in the 100 k
resistance at t = 3s is
(A) zero
(B) 48 A
(C) 35.5 A
(D) 16 A

51. In the network shown below the voltage V0 is nearly


(A) 10 volt
(B) 11 volt
(C) 12 volt
(D) zero volt

52. The energy of the characteristic X – ray photon in a Coolidge tube comes from
(A) the kinetic energy of striking electron
(B) the kinetic energy of the free electrons of the target
(C) the kinetic energy of the ions of the target
(D) the electronic transition of the target atom

FIITJEE Ltd., Punjabi Bagh Delhi Centre, 31-32-33, Central Market, West Avenue Road, Punjabi Bagh (West), New Delhi - 110026, Ph: 011-45634000
53. The maximum wavelength that can ionize a hydrogen atom initially in the ground state is
(A) 660.0 nm (B) 364.5 nm
(C) 121.9 nm (D) 91.4 nm

54. The wavelength of the waves associated with a proton and a photon are the same.
Therefore, the two have equal
(A) mass (B) velocity
(C) momentum (D) kinetic energy

55. Which of the following sources emits light having highest degree of coherence?
(A) Light emitting diode (B) LASER diode
(C) Neon lamp (D) incandescent lamp

56. An alpha particle with kinetic energy K approaches a stationary nucleus having atomic
number Z. The distance of closest approach is b. Therefore the distance of closest approach
for a nucleus of atomic number 2Z is
(A) b/Z (B) 2 b
(C) 2 b (D) 4 b

57. In a photodiode the reverse current increase when exposed to light of wavelength 620 nm or
less. The band gap of the semiconductor used is
(A) 0.067 eV (B) 1.12 eV
(C) 2.00 eV (D) 2.42 eV

58. An electron in hydrogen atom jumps from a level n = 4 to n = 1. The momentum of the
recoiled atom is
(A) 6.8  10–27 kg -m/s (B) 12.75  10–19 kg -m/s
(C) 13.6  10–19 kg -m/s (D) zero

59. For the Boolean equation Y = AB + A(B + C) + B(B + C) + B , which of the following
statements is correct?
(A) Y does not depend on A but depends on B
(B) Y does not depend on B but depends on A
(C) Y does not depend on B
(D) Y depends only on C

60. Refer to the common emitter amplifier circuit shown below using a transistor with  = 80 and
VBE = 0.7 volt. The value of resistance RB is

(A) 330  (B) 330 


(C) 220  (D) 220 k

FIITJEE Ltd., Punjabi Bagh Delhi Centre, 31-32-33, Central Market, West Avenue Road, Punjabi Bagh (West), New Delhi - 110026, Ph: 011-45634000
A-2
QUESTIONS WITH MORE THAN ONE OPTION CORRECT

61. A horizontal insulated cylinder of volume V is divided into four identical compartments by
stationary semi-permeable thin partitions as shown. The four compartments from left are
initially filled with 28 g helium, 160 g oxygen, 28 g nitrogen and 20 g hydrogen respectively.
The left partition lets through hydrogen, nitrogen and helium while the right partition lets
thorough hydrogen only. The middle partition lets through hydrogen and nitrogen both. The
temperature T inside the entire cylinder is maintained constant. After the system is set in
equilibrium

He O2 N2 H2

14RT
(A) pressure of helium is
V
20RT
(B) pressure of oxygen is
V
4RT
(C) pressure of nitrogen is
3V
10RT
(D) pressure of hydrogen is
V

62. After charging a capacitor C to a potential V, it is connected across an ideal inductor L. The
capacitor starts discharging simple harmonically at time t= 0. The charge on the capacitor at
a later instant is q and the periodic time of simple harmonic oscillations is T. therefore.
(A) q = CV sin(t) (B) q = CV cos(t)
1
(C) T  2 (D) T  2 LC
LC

63. In the circuit arrangement shown two cells supply a current l to a load resistance R = 9 .
One cell has an emf E1 = 9V and internal resistance r1 = 1  and the other cell has an emf
E2 = 6 V and internal resistance r2 = 3 . The currents are as shown, then

(A) I1 = 0.9 A and l2 = 0.5 A


(B) I  0.85A
(C) if the cell of emf E1 is removed, current I will be smaller
(D) if the cell of emf E2 is removed, current I will be smaller

64. A transparent cylindrical rod of length l = 50 cm, radius R = 10 cm and refractive index  =
3 lies onto a horizontal plane surface. A ray of light moving perpendicular to its length is
incident on the rod horizontally at a height h above the plane surface such that this ray
emerges out of the rod at a height 10 cm above the plane surface. Therefore h is
(A) 1.34 cm (B) 1.73 cm
(C) 10.0 cm (D) 18.66 cm
FIITJEE Ltd., Punjabi Bagh Delhi Centre, 31-32-33, Central Market, West Avenue Road, Punjabi Bagh (West), New Delhi - 110026, Ph: 011-45634000
65. Two point charges +1 C and - 1C are place at points(0, - 0.1m) and (0, +0.1 m)
respectively in XY plane. Then choose the correct statement/s from the following
(A) the electric field at all points on the Y axis has the same direction
(B) the dipole moment is 0.2C-m along +X axis direction
(C) no work has to be done in bringing a test charge from infinity to the origin
(D) electric field at all points on the X axis is along +Y axis

66. An inductance L, a resistance R and a batter B are connected in series with a switch S. The
voltages across L and R are VL and VR respectively. Just after closing the switch S
(A) VL will be greater than VR
(B) VL will be less than VR
(C) VL will be the same as VR
(D) VL will decrease while VR will increase as time progresses
67. A string of length l, tied to the top of a pole, carries as ball at its
end as shown. On giving the ball a single hand blow
perpendicular to the string, it acquires an initial velocity 0 in the
horizontal plane and moves in a spiral of decreasing radius by
curling itself around the pole. Therefore,

(A) the instantaneous centre of revolution of the ball is the point of contact of the string with
the pole at that instant.
(B) the instantaneous centre of revolution of the ball will be fixed at the point where the string
was initially fixed.
(C) the angular momentum of the system will not be conserved.
(D) the angular momentum of the system will not conserved.

68. A circular loop of conducting wire or radius 1 cm is cut at a point A on its circumference. It is
then folded along a diameter through A such that the two semicircular  loops lie in two
mutually perpendicular planes. In this region a uniform magnetic field B of magnitude 100
mT is directed perpendicular to the diameter through A and makes angles of 30o and 60o
with the planes of the two semicircles. The magnetic field reduces at a uniform rate from
100 mT to zero in a time interval of 4.28 ms. Therefore
(A) instantaneous emf in the two loops are in the ratio 2 : 1
(B) instantaneous emf in the two loops are in the ratio 3 : 1
(C) the total emf between free ends at point A is 5 mV
(D) the total emf between free ends at point A is 1.4 mV

69. A converging lens of focal length 40 cm is fixed at 40 cm in front of a screen. An object


placed 120 cm from the fixed lens is required to be focused on the screen by introducing
another identical lens in between. The second lends should be placed at a distance x form
the object where x is
(A) 40 cm (B) 50 cm
(C) 140 cm (D) 150 cm

FIITJEE Ltd., Punjabi Bagh Delhi Centre, 31-32-33, Central Market, West Avenue Road, Punjabi Bagh (West), New Delhi - 110026, Ph: 011-45634000

70. Mysteriously a charged particle moving with velocity    0 î entered the tube of Thomson’s
apparatus where the parallel metallic plates of length 5 cm along X axis are separated by


2 cm. Under the influence of a magnetic field B  4.57  102 kˆ T, the particle is found to 
o
deflect by an angle of 5.7 . When a potential of 2000 volt is applied between the two plates,
the particle is found to move straight to the screen without any deflection. Then
(A) the velocity 0 = 2.19  108 m/s
(B) the charge to mass ratio of the particle is 9.58  107 C/kg
(C) radius of the circular path in the magnetic field is 50 cm
(D) the particle is identified as a proton

FIITJEE Ltd., Punjabi Bagh Delhi Centre, 31-32-33, Central Market, West Avenue Road, Punjabi Bagh (West), New Delhi - 110026, Ph: 011-45634000
INDIAN ASSOCIATION OF PHYSICS TEACHERS
NATIONAL STANDARD EXAMINATION IN PHYSICS 2019 - 20
(NSEP 2019 - 20)
Date of Examination: November 24, 2019
Q. Paper Code: 61

Total Time: 120 minutes (A-1 and A-2)

A-1
ONLY ONE OUT OF FOUR OPTIONS IS CORRECT

1. An observer stands on the platform at the front edge of the first bogie of a stationary train.
The train starts moving with uniform acceleration and the first bogie takes 5 seconds to cross
the observer. If all the bogies of the train are of equal length and the gap between them is
negligible, the time taken by the tenth bogie to cross the observer is:
(a) 1.07s (b) 0.98s
(c) 0.91s (d) 0.81s

2. The resistive force on an aeroplane flying in a horizontal plane is give by Ff = kv 2 , where k


is constant and v is the speed of the aeroplane. When the power output form the engine is
P0 , the plane flies at a speed v 0 . If the power output of the engine is doubled the aeroplane
will fly at a speed of
(a) 1.20v 0 (b) 1.26v 0
(c) 1.41v 0 (d) 2.82v0

3. A 3.0 cm thick layer of oil (density oil = 800 kg/m3) floats on water (density w =1000 kg/m3)
1
in a transparent glass beaker. A solid cylinder is observed floating vertically with of it in
3
1
water and in the oil. Oil is gently poured into the beaker until the cylinder floats in oil only.
3
The fraction of the solid cylinder in oil now is
3 2
(a) (b)
5 3
3 8
(c) (d)
4 9

4. A wedge of mass M rests on a horizontal frictionless surface. A block of mass m starts


sliding down the rough inclined surface of the wedge to its bottom. During the course of
motion, the centre of mass of the block and the wedge system
(a) does not move at all
(b) moves horizontally with constant speed
(c) moves horizontally with increasing speed
(d) moves vertically with increasing speed

5. A uniform circular disc rotating at a fixed angular velocity  about an axis normal to its plane
and passing through its centre has kinetic energy E. If the same disc rotates with an angular
velocity 2 about a parallel axis passing through the edge, its kinetic energy will be
(a) 2 E (b) 4 E
(c) 10 E (d) 12 E
FIITJEE Ltd., Punjabi Bagh Delhi Centre, 31-32-33, Central Market, West Avenue Road, Punjabi Bagh (West), New Delhi - 110026, Ph: 011-
45634000
6. In the following figure the velocity time graphs for three particles 1, 2 and 3 are shown.

The magnitude of average acceleration of the three particles, over 10 s, bear the relationship
(a) a1  a2  a3 (b) a2  a1  a3
(c) a3  a2  a1 (d) a1 = a2 = a3

7. The potential energy (U) of a particle moving in a potential field varies with its displacement
(x) as shown below.

The variation of force F (x) acting on the particle as a function of x can be represented by

(a) Fig (i) (b) Fig (ii)


(c) Fig (iii) (d) Fig (iv)
FIITJEE Ltd., Punjabi Bagh Delhi Centre, 31-32-33, Central Market, West Avenue Road, Punjabi Bagh (West), New Delhi - 110026, Ph: 011-
45634000
8. A pendulum is made by using a thread of length 300 cm and a small spherical bob of mass
100 g. It is suspended from a point S. The bob is pulled from its position of rest at O to the
point A so that the linear amplitude is 25 cm. The angular amplitude in radian and the
potential energy of the bob in joule at A are respectively
(a) 0.10 and 0.10 (b) 0.083 and 0.01
(c) 0.251 and 2.94 (d) 0.083 and 0.24

9. Consider the following physical expressions


(I) v 2 ( : density, v velocity)
YL
(II) (Y : Young’s modulus, L : length)
L
2
(III) (: surface density of charge)
0
(IV) hrg (h: rise of a liquid in a capillary tube of radius r)
The expressions having same dimensional formula are
(a) I and II only (b) II and III only
(c) II, III and IV only (d) I, II and III only

10. Two simple pendulums of lengths 1.44 m and 1.0 m start swinging together in the same
phase. The two will be in phase again after a time of
(a) 6 second (b) 9 second
(c) 12 second (d) 25 second

11. A home aquarium partly filled with water slides down an inclined plane of inclination angel 
with respect to the horizontal. The surface of water in aquarium
(a) remains horizontal
(b) remains parallel to the plane of the incline
(c) forms an angle  with the horizon where 0 <  < 
(d) forms an angle  with horizon, where  <  < 90

12. A sound source of a constant frequency travels with a constant velocity past an observer.
When it crosses the observer the sound frequency sensed by the observer changes from
449 Hz to 422 Hz. If the velocity of sound is 340 m/s, the velocity of the source of sound is
(a) 8.5 m/s (b) 10.5 m/s
(c) 12.5 m/s (d) 14.5 m/s

13. Light of wavelength 640 nm falls on a plane diffraction grating with 12000 lines per inch. In
the diffraction pattern on a screen kept at a distance of 12 cm from the grating, the distance
of the second order maximum from the central maximum is
(a) 1.81 cm (b) 2.41 cm
(c) 3.62 cm (d) 7.25 cm

14. If the force acting on a body is inversely proportional to its speed, the kinetic energy of the
body varies with time t as
0 1
(a) t (b) t
2 −1
(c) t (d) t

FIITJEE Ltd., Punjabi Bagh Delhi Centre, 31-32-33, Central Market, West Avenue Road, Punjabi Bagh (West), New Delhi - 110026, Ph: 011-
45634000
15. As shown in the figure, a block of mass m is hung from the
ceiling by the system of springs consisting of two layers. The
force constant of each of the springs is k. The frequency of the
vertical oscillations of the block is.
1 k 1 4k
(a) (b)
2 5m 2 5m
1 5k 1 6k
(c) (d)
2 6m 2 5m

16. Two simple harmonic motions are given by x1 = a sin t + acos t and
a
x 2 = a sin t + cos t the ratio of the amplitude of a the first to the second and the phase
3
difference between them respectively are
3  3 
(a) and (b) and
2 12 2 12
2  3 
(c) and (d) and
3 12 2 6

17. A particle is projected from the ground with a velocity v = 3iˆ + 10ˆj ms−1 . The maximum ( )
height attained and the range of the article are respectively given by (use g = 10 m/s2)
(a) 5m and 6m (b) 3m and 10m
(c) 6m and 5m (d) 3m and 5m

18. A 20 cm long capillary tube stands vertically with lower end just in water. Water rises upto
5 cm. If the entire system is now kept on a freely falling platform, the length of the water
column in the capillary tube will be
(a) 5cm (b) 10cm
(c) zero (d) 20 cm

19. Position – time graph of a particle moving in a potential field is shown beside. If the mass of
the particle is 1 kg its total energy is approximately

−4 −4
(a) 15.45  10 J (b) 30.78  10 J
−4 −4
(c) 7.71 10 J (d) 3.85  10 J

FIITJEE Ltd., Punjabi Bagh Delhi Centre, 31-32-33, Central Market, West Avenue Road, Punjabi Bagh (West), New Delhi - 110026, Ph: 011-
45634000
20. Plots of intensity (I) of radiation emitted by a black body
versus wavelength (  ) at three different temperatures
T1,T2 and T3 respectively are shown in figure. Choose
the correct statement.
(a) T1  T2  T3 necessarily
(b) T3  T2  T1 necessarily
(c) T2 = ( T1 + T3 ) / 2 necessarily
(d) T22 = T1T3 necessarily

21. Consider a composite slab consisting of two different materials having equal thickness and
equal area of cross – section. The thermal conductivities are K and 2K respectively. The
equivalent thermal conductivity of the composite slab is
2K
(a) (b) 2K
3
4K
(c) 3k (d)
3

22. A large horizontal uniform disc can rotate freely about a rigid vertical axis passing through its
centre O. A man stands at rest at the edge of the disc at a point A. The mass of the disc is
22 times the mass of the man. The man starts moving along the edge of the disc. When he
reaches A, after completing one rotation relative to the disc, the disc has turned through
(a) 30o (b) 90o
(c) 60o (d) 45o

23. Two factories are soilding their sirens at 400 Hz each. A man walks from one factory towards
the other at a speed of 2 m/s. The velocity of sound is 320 m/s. The number of beats heard
by the person in one second will be
(a) 6 (b) 5
(c) 4 (d) 2.5

24. The temperature of an isolated black body falls from T1 to T2 in time t. Then t = Cx where x
is
 1 1  1 1 
(a)  −  (b)  2 − 2 
 T2 T1   T2 T1 
 1 1   1 1 
(c)  3 − 3  (d)  4 − 4 
 T2 T1   T2 T1 

25. Two charges – q and – q are placed at point (0, d) and (0, – d). A charge + q, free to move
along X axis, will oscillate with a force proportional to
1 1
(a) 2 (b) 2
x +d 2
x
x 1
(c) 3
(d)
(d2 + x 2 2 ) x 2 + d2

26. The average translational kinetic energy of oxygen (M = 32) molecules at a certain
temperature is 0.048 eV. The translational kinetic energy of nitrogen (M = 28) molecules at
the same temperature is (consider the two gases to be ideal)
(a) 0.0015 eV (b) 0.042 eV
(c) 0.048 eV (d) 0.768 eV

FIITJEE Ltd., Punjabi Bagh Delhi Centre, 31-32-33, Central Market, West Avenue Road, Punjabi Bagh (West), New Delhi - 110026, Ph: 011-
45634000
27. The log – log graph for a non – linear oscillator is
shown below. Assuming the constants to have
appropriate dimensions the relationship between
time period (T) and the amplitude (A) can be
expressed as
(a) T = 1000A 2
(b) T = 4A1/2
(c) T = 4A2 + B
(d) T = 8A 3

28. In many situations the point source emitting a wave starts moving, through the medium, with
velocity V greater than the wave velocity in that medium. In such a case when source
velocity (V) > wave velocity (v), the wave front changes
(a) from spherical to plane (b) from spherical to conical
(c) from plane to spherical (d) from cylindrical to spherical

29. If the average mass of a smoke particle in an Indian kitchen is 3  10−17 kg, the rms speed of
the smoke particles at 270 C is approximately
(a) 2 cm/sec (b) 2 m/sec
(c) 2 km/sec (d) none of these

30. Two wires, made of same material, one thick and the other thin are joined to form one
composite wire. The composite wire is subjected to the same tension throughout. A wave
travels along the wire and passes the point where the two wires are joined. The quantity
which changes at the joint are
(a) frequency only (b) propagation speed only
(c) wavelength only (d) both propagation speed and wavelength

31. The frequency of the third overtone of a closed end organ pipe equals the frequency of the
fifth harmonic of an open end organ pipe . Ignoring end correction, the ratio of their lengths
lopen :Iclose is
(a) 10:7 (b) 10:9
(c) 2:1 (d) 7:10

32. A concave mirror has a radius of curvature R and forms the image of an object placed at a
distance 1.5R from the pole of the mirror. An opaque disc of diameter half the aperture of the
mirror is placed with the pole at the centre. As a result
(a) the position of the image will be the same but its central half will disappear
(b) the position of the image will be the same but its outer half will disappear
(c) the complete image will be seen at the same position and it will be exactly identical with
the initial image
(d) the complete image will be seen at the same position but it will hot be identical in all
respect with the initial image.

33. A ray of white light is made incident on the refracting surface of a prism such that after
refraction at this surface, the green component falls on the second surface at its critical
angle The colours present in the emergent beam will be
(a) violet, indigo and blue (b) violet, indigo, blue, yellow, orange and red
(c) yellow, orange and red (d) all colours

34. In a compound microscope, having tube – length 30 cm, the power of the objective and the
eye – piece are 100D and 10D respectively. Then the magnification produced by the
microscope when the final image is at the least distance of distinct vision (25 cm) will be
(a) 55 (b) 64
FIITJEE Ltd., Punjabi Bagh Delhi Centre, 31-32-33, Central Market, West Avenue Road, Punjabi Bagh (West), New Delhi - 110026, Ph: 011-
45634000
(c) 77 (d) 90

35. Parallel rays are incidents on a glass sphere of diameter 10 cm and having refractive index
1.5. The sphere converges these rays at a certain point. The distance of this point from the
centre of the sphere will be
(a) 2.5 cm (b) 5 cm
(c) 7.5 cm (d) 12.5 cm

36. A jet of water from 15 cm diameter nozzle of a fire hose can reach the maximum height of
25 m. The force exerted by the water jet on the hose is
(a) 4.24 kN (b) 17.32 kN
(c) 2.17 kN (d) 8.66 kN

37. In an electromagnetic wave the phase difference between electric vector and magnetic
vector is

(a) zero (b)
2
3
(c)  (d)
2

38. A rectangular slab of glass of refractive index 1.5 is immersed in water of refractive index
1.33 such that top surface of the slab remains parallel to water level. Light from a point
source in air is incident on the surface of water at an angle  such that the light reflected
from the glass slab is plane polarized, the angle  is
(a) 84.4o (b) 48.4o
(c) 56.3o (d) 53.1o

1
39. In a spectrometer the smallest main scale division is of a degree. The total number of
3
divisions on the vernier scale attached to the main scale is 60 which coincide with the 59
divisions of the main circular scale. The least count of the spectrometer is
(a) 20’ (b) 20”
(c) 30” (d) 30’

40. White light is used to illuminate two slits in Young’s double sit experiment. Separation
between the two slits is b and the screen is at a distance D (>>b) from the plane of slits. The
wavelength missing at a point on the screen directly in fron of one of the slits is
2b 2 2b 2
(a) (b)
3D D
2 2
b b
(c) (d)
3D 2D

41. In an ink – jet printer, an ink droplet of mass m is


given a negative charge q by a computer –
controlled charging unit. The charged droplet then
enters the region between two deflecting parallel
plates of length L separated by distance d (see
figure) with a speed v. All over this region there
exists a uniform downward electric field E (in the
plane of paper). Neglecting the gravitational force
on the droplet, the maximum charge that can be
given to this droplet, so that it does not hit by of the
plates, is

FIITJEE Ltd., Punjabi Bagh Delhi Centre, 31-32-33, Central Market, West Avenue Road, Punjabi Bagh (West), New Delhi - 110026, Ph: 011-
45634000
mv 2L mv 2 d md mv 2L2
(a) (b) (c) (d)
Ed2 EL2 EV 2L2 Ed

42. A converging beam of light is pointing to P. Two observations are made with (i) a convex
lens of focal length 20 cm and (ii) a concave lens of focal length 16 cm placed in the path of
the convergent beam at a distance 12 cm before the point P. It is observed that
(a) in both cases the images are real
(b) in both cases the images are virtual
(c) for (i) the image is real and for (ii) the image is virtual
(d) for (i) the image is virtual and for (ii) the image is real
43. A spherical capacitor is formed by two concentric metallic spherical shells. The capacitor is
then charged so that the outer shell carries a positive charge and the inner shell carries an
equal but negative charge. Even if the capacitor is not connected to any circuit, the charge
will eventually leak away due to a small electrical conductivity of the material between the
shells. What is the character of the magnetic field produced by this leakage current?
(a) Radially outwards for the inner shell to the outer shell.
(b) Radially inwards from the outer shell to the inner shell.
(c) Circular field lines between the shells and perpendicular to the radial direction.
(d) No magnetic field will be produced.
44. If a cell of constant emf produces the same amount of heat during the same time in two
independent resistors R1 and R2 when they are separately connected across the terminals of
the cell, one after the other. The internal resistance of the cell is
R1 + R2 R2 ~ R2 R12 + R 22
(a) (b) (c) (d) R1R2
2 2 2

45. In the circuit shown beside the charge on each capacitor is


C1C2
(a) (C1 + C2) (E1 – E2) (b) (E1 + E2 )
C1 + C2
C1C2
(c) (E1 − E2 ) (d) (C1 – C2) (E1 + E2)
C1 + C2

46. A stationary hydrogen atom emits photon corresponding to the first linear (highest wave
length) of Lyman series. If R is the Rydberg constant and M is the mass of the atom, the
recoil velocity of the atom is
Rh 3Rh 3Rh Rh
(a) (b) (c) (d)
4M M 4M M

47. Heat is absorbed or evolved when current flow sin a conductor having a temperature
gradient. This phenomenon is known as
(a) Joule effect (b) Peltier effect
(c) Seeback effect (d) Thomson effect

48. Identify the rank in order from the dimmest to the brightest when all
the identical bulbs are connected in the circuit as shown below.
(a) A = B > C = D (b) A = B = C = D
(c) A > C > B > D (d) A = B < C = D

49. The unit of magnetizing field is


(a) tesla (b) newton (c) ampere (d) ampere turn/meter
FIITJEE Ltd., Punjabi Bagh Delhi Centre, 31-32-33, Central Market, West Avenue Road, Punjabi Bagh (West), New Delhi - 110026, Ph: 011-
45634000
50. A star undergoes a supernova explosion. Just after the explosion, the material left behind
forms a uniform sphere of radius 8000 km with a rotation period of 15 hours. This remaining
material eventually collapses into a neutron star of radius 4 km with a period of rotation
(a) 14 s (b) 3.8 h (c) 0.021 s (d) 0.0135 s
51. A number of identical absorbing plates are arranged in between a source of light and a photo
cell. When there is not plate in between, the photo current in maximum. Under the
circumstances let us focus on the two statements:
(1) The photo current decreases with the increase in number of absorbing plates.
(2) The stopping potential increases with the increase in number of absorbing plates.
(a) Statements (1) and (2) are both true and (1) is the cause of (2).
(b) Statements (1) and (2) are both true but (1) and (2) are independent.
(c) Statement (1) is true while (2) is not true and (1) and (2) are independent.
(d) Statement (1) is true while (2) is not true and (2) is the effect of (1).
52. In a nuclear reaction, two photons each of energy 0.51 MeV are produced by electron-
positron annihilation. The wavelength associated with each photon is
(a) 2.44 × 10–12 m (b) 2.44 × 10–8 m (c) 1.46 × 10–12 m (d) 3.44 × 10–10 m

53. In the circuit shown if an ideal ammeter is connected between


A and B then the direction of current and the current reading
would be (assume I, remains unchanged)
(a) B to A and Is/ 2 (b) A to B and Is/ 4
(c) B to A and Is/ 9 (d) B to A and Is/ 3

54. Avalanche breakdown in a p-n junction primarily depends on the phenomenon of


(a) doping (b) collision (c) recombination (d) ionization

55. A source emits photons of energy 5 eV which are incident on a metallic sphere of work
function 3.0 eV. The radius of the sphere is r = 8 × 10–3 m. It is observed that after some
time emission of photoelectrons fro the metallic sphere is stopped. Charge on the sphere
when the photoemission stops is
(a) 1.77 × 10–16 C (b) 1.77 × 10–12 C
–12
(c) 1.11 × 10 C (d) 1.11 × 10–10 C

56. The de component of current in the output of a half-wave rectifier with peak value Io is
I I 2I
(a) zero (b) 0 (c) 0 (d) 0
 2 
57. In an experiment on photoelectric effect, the slope of straight line graph between the
stopping potential and the frequency of incident radiation gives
(a) Electron charge (e) (b) Planck constant (h)
h
(c) (d) Work function (W)
e
58. According to Bohr’s theory the ionization energy of H atom is 13.6 eV. The energy needed to
remove an electron from Helium ion (He+) is
(a) 13.6 eV (b) 16.8 eV (c) 27.2 eV (d) 54.4 eV

59. The phenomenon inverse to photo electric effect is


(a) Compton effect (b) Pair production
(c) Raman effect (d) Production of X-rays in Coolidge tube

60. A stationary hydrogen atom emits a photon of wavelength 1025 Å. Its angular momentum
changes by

FIITJEE Ltd., Punjabi Bagh Delhi Centre, 31-32-33, Central Market, West Avenue Road, Punjabi Bagh (West), New Delhi - 110026, Ph: 011-
45634000
h 2h h 3h
(a) (b) (c) (d)
  2 2
A-2
ANY NUMBER OF OPTIONS 4, 3, 2 or 1 MAY BE CORRECT

61. A block of mass m = 10 kg is hanging over a frictionless light fixed


pulley by an inextensible light rope. Initially the block is held at rest.
The other end of the tope is now pulled by a constant force F in the
vertically downward direction. The linear momentum of the block is
seen to increase by 2 kg m/s in 1 s (in the first second). Therefore,
(a) the tension in the rope is F
(b) the tension in the rope is 3 N
(c) the work done by the tension on the block, in first second, is
= 19.8 J
(d) the work done against the force of gravity, in first second, is = 9.8 J.

62. A ball of mass m1 travels horizontally along the x-axis in the positive direction with an initial
speed of v0. It collides with another ball of mass m2 that is originally at rest. After the
collision, the ball of mass m1 has velocity (v1x ˆi + v1y ˆj) and the ball of mass m2 has velocity
(v 2x ˆi + v 2y ˆj). Identify the correct relationship(s).
(a) 0 = m1v1x (b) m1v0 = m1v1y + m2v2y
(c) 0 = m1v1y + m2v2y (d) m1v0 = m1v1x + m2v2x

63. In a real gas


(a) the force of attraction between the molecules depends upon intermolecular distance.
(b) internal energy depends only upon temperature.
(c) internal energy is a function of both temperature and volume.
(d) internal energy is a function of both temperature and pressure.

64. A particle of mass m is thrown vertically up with velocity u. Air exerts an opposing force of a
constant magnitude F. The particle returns back to the point of projection with velocity v after
attaining maximum height h, then
u2 v2
(a) h = (b) h =
 F  F
2g +  2g − 
 m  m
 F  F
g − m  g + m 
(c) v = u   (d) v = u  
 F  F
g + m  g − m 
   

65. A pin of small length ‘a’ is placed along the axis of a concave mirror of focal length f, at the
distance u(>f) from its pole. The length of its image is ‘b’. If the same object its placed
perpendicular to its axis at the same distance u and the length of its image is now ‘c’, then
f2 u−f a2 f 3
(a) b = a (b) c = ab (c) c = b (d) bc =
(u − f )2
f (u − f )3

66. A thin rod of length 10 cm is placed along the axis of a concave mirror of focal length 30 cm
in such a way that one end of the image coincides with one end of the object. The length of
the image may be
(a) 7.5 cm (b) 12 cm (c) 15 cm (d) 10 cm

67. The mass of an electron can be expressed as


(a) 0.1512 MeV (b) 8.19 × 10–14 J / c2
FIITJEE Ltd., Punjabi Bagh Delhi Centre, 31-32-33, Central Market, West Avenue Road, Punjabi Bagh (West), New Delhi - 110026, Ph: 011-
45634000
(c) 9.1 × 10–31 kg (d) 0.00055 amu
Where is speed of light in vacuum
68. Select the correct statement(s), out of the following, about diffraction at N parallel slits.
(a) There are (N-1) minima between each pair of principal maxima.
(b) There are (N-2) secondary maxima between each pair of principal maxima.
(c) Width of principal maximum is proportional to 1/N.
(d) The intensity at the principal maxima varies as N2.

69. An electric dipole placed in a non-uniform electric field may experience


(a) non net force, no torque. (b) no net force, but no torque.
(c) no net force, but a torque. (d) a net force and a torque.

70. Two long parallel wires carry currents of equal magnitude (I)
but in opposite directions. These wires are suspended from
fixed rod PQ by four chords of equal length L as shown. The
mass per unit length of each wire is , the value of angle 
substended by two chords OA and OB, assuming it to be
small, is
   1
(a)  = I 0 (b)  = I 0
4 gL  gL
0 g  0 g
(c)  = I (d)  = I
4  L  L

FIITJEE Ltd., Punjabi Bagh Delhi Centre, 31-32-33, Central Market, West Avenue Road, Punjabi Bagh (West), New Delhi - 110026, Ph: 011-
45634000
*02161

Time: 9:00 AM to 10:00 AM


Question Paper Code: 61

Student's
Roll No:

Write the question paper code mentioned above on YOUR OMR Answer Sheet (in the space provided),
otherwise your Answer Sheet will NOT be evaluated. Note that the same Question Paper Code
appears on each page of the question paper.
Instructions to Candidates:
1. Use of mobile phone, smart watch, and iPad during examination is STRICTLY PROHIBITED.
2. In addition to this question paper, you are given OMR Answer Sheet along with candidate's copy.
3. On the OMR sheet, make all the entries carefully in the space provided ONLY in BLOCK CAPITALS as
well as by properly darkening the appropriate bubbles.
Incomplete/ incorrect/ carelessly filled information may disqualify your candidature.
4. On the OMR Answer Sheet, use only BLUE or BLACK BALL POINT PEN for making entries and filling
the bubbles.
5. Your fourteen-digit roll number and date of birth entered on the OMR Answer Sheet shall remain your
login credentials means login id and password respectively for accessing your performance / result in Indian
Olympiad Qualifier in Physics 2020-21 (Part I).
6. Question paper has two parts. In part A1 (Q. No.1 to 24) each question has four alternatives, out of which only
one is correct. Choose the correct alternative and fill the appropriate bubble, as shown.
Q.No. 12 a c d
In part A2 (Q. No. 25 to 32) each question has four alternatives out of which any number of alternative(s) (1, 2,
3, or 4) may be correct. You have to choose all correct alternative(s) and fill the appropriate bubble(s), as shown
Q.No. 30 a c d
7. For Part A1, each correct answer carries 3 marks whereas 1 mark will be deducted for each wrong answer.
In Part A2, you get 6 marks if all the correct alternatives are marked and no incorrect. No negative marks in
this part.
8. Rough work should be done only in the space provided. There are 08 printed pages in this paper.
9. Use of non- programmable scientific calculator is allowed.
10. No candidate should leave the examination hall before the completion of the examination.
11. After submitting answer paper, take away the question paper and candidate's copy of OMR for your reference
Please DO NOT make any mark other than filling the appropriate bubbles properly in the
space provided on the OMR answer sheet.
OMR answer sheets are evaluated using machine, hence CHANGE OF ENTRY IS NOT
ALLOWED. Scratching or overwriting may result in a wrong score.
DO NOT WRITE ON THE BACK SIDE OF THE OMR ANSWER SHEET.

1
61
Instructions to Candidates (Continued) :

You may read the following instructions after submitting the answer sheet.

12. Comments/Inquiries/Grievances regarding this question paper, if any, can be shared on the
Inquiry/Grievance column on www.iaptexam.in on the specified format till February 12,
2021.
13. The answers/solutions to this question paper will be available on the website: www.iapt.org.in
by February 13, 2021.
14. CERTIFICATES and AWARDS:
Following certificates are awarded by IAPT to students, successful in the Indian Olympiad Qualifier
in Physics 2020-21 (Part I)
(i) “CENTRE TOP 10 %”
(ii) “STATE TOP 1 %”
(iii) “NATIONAL TOP 1 %”
(iv) “GOLD MEDAL & MERIT CERTIFICATE” to all students who attend OCSC-2021 at HBCSE
Mumbai
15. All these certificates (except gold medal) will be downloadable from IAPT website :
www.iapt.org.in after March 15, 2020-21.
16. List of students (with centre number and roll number only) having score above MAS will be
displayed on the website: www.iapt.org.in by February 25, 2021. See the Minimum Admissible
score Clause on the Student's brochure on the web.
17. List of Students eligible for evaluation of IOQP 2020-21 (Part II) shall be displayed on
www.iapt.org.in by March 1, 2021.
Physical constants you may need….

-31 -19
Mass of electron me = 9.10´ 10 kg Magnitude of charge on electron e = 1.60´ 10 C
-27 -12 2 -1 -2
Mass of proton mp = 1.67´ 10 kg Permittivity of free space e 0 =8.85´ 10 C N m
-2 -7 -1
Acceleration due to gravity g = 9.8 ms Permeability of free space m 0 = 4p ´ 10 Hm
-11 2 -2 -34
Universal gravitational constant G = 6.67´ 10 Nm Kg Planck's constant h = 6.625 ´ 10 Js
-1 -1 -1
Universal gas constant R = 8.31 Jmol K Faraday constant = 96,500 Cmol
-23 -1
Boltzmann constant k = 1.38´ 10 JK Rydberg constant R = 1.097 ´ 107 m-1

Stefan's constant s = 5.67´ 10-8 Wm-2K-4 Speed of light in free space c = 3 ´ 108 ms-1
23 -1
Avogadro's constant NA = 6.023 ´ 10 mol

2
61

Question Paper Code: 61

Time: 60 Minute Max. Marks: 120


Attempt All Thirty Two Questions
A-1
ONLY ONE OUT OF FOUR OPTIONS IS CORRECT. BUBBLE THE CORRECT OPTION.

1. If speed of light c, Planck's constant h and gravitational constant G are chosen as fundamental
quantities, dimensions of time in this system of units is
(a) ch3/2 G–3/2 (b) c–2 G1/2 h (c) c2 G1/2 h5/2 (d) c–5/2 G1/2 h1/2

2. A solid hemisphere is cemented on the flat surface of a solid cylinder of same radius R and same
material. The composite body is rotating about the axis of the cylinder of length l with angular
speed w. The radius of gyration K is

2  15R  8l  1  15l  8R R


(a) R   (b) R  
5  3R  2l  10  3l  2R


3  15R  8l  1  3l  2R 
(c) R   (d) R   R
10  3R  2l  10  15l  8R

3. The shortest period of rotation of a planet (considered to be a sphere of uniform density r) about its
own axis, such that any mass m kept on its equator is just to fly off the surface, is

5  3 5
(a) T  (b) T  (c) T  (d) T 
G 3 G G 3 G

4. A body of mass 10 kg at rest explodes into two fragments of masses 3 kg and 7 kg. If the total
kinetic energy of two pieces after explosion is 1680 J, the magnitude of their relative velocity
in m/s after explosion is:
(a) 40 (b) 50 (c) 70 (d) 80

5. A shot is fired at an angle a to the horizontal up a hill (Considered to be a long straight incline
plane) of inclination b to the horizontal. It will strike the hill horizontally if
(a) tan a = 2 tan b (b) sin a = sin 2 b (c) sin a = 2 sin b (d) tan a = 4 tan b

3
61
2
6. A particle is executing Simple Harmonic Motion of time period T  4 in a straight line. Starting
from rest, it travels a distance 'a' in the first second and distance 'b' in the next second travelling in
the same direction. The amplitude of SHM is
2a2 3a2 2a2
(a) (b) (c) (d) none of these
3ab 3a2b 2ab

7. The kinetic energy of a particle moving along a circle of radius R depends upon the distance
covered 's' as KE = as2 where a is a constant. The magnitude of the force acting on the particle as a
function of 's' is
2
2as
2
2as
2
s
(a) (b) (c) 2as (d) 2as 1  
R m R
8. The flow of water in a horizontal pipe is stream line flow. Along the pipe, at a point, where cross -
sectional area is 10 cm2, the velocity of water flow is 1.00 ms-1 and the pressure is 2000 Pa. The
pressure of water at another point where cross - sectional area is 5 cm2 is
(a) 2000 Pa (b) 1500 Pa (c) 3500 Pa (d) 500 Pa

9. Three containers A, B and C are filled with water at different temperature. When 1 litre of water
from A is mixed with 2 litre of water from B, the resulting temperature of mixture is 520 C.
When 1 litre of water from B is mixed with 2 litre of water from C, the resulting temperature of
mixture is 400 C. Similarly when 1 litre of water from C is mixed with 2 litre of water from A, the
resulting temperature of mixture is 340 C. Temperature of mixture when one litre of water from
each container is mixed (neglect the water equivalent of container) is
(a) 400 C (b) 420 C (c) 380 C (d) 450 C

10. Point charge q is kept at each corner of a cube of edge length l . The resultant force of repulsion on
any one of the charges due to all others is expressed as q q

 1 1  2  1 1  2 q
(a)  1   q (b)  1 q
q
 2 2 3 3 2 2 3 3
 0 l 2  0 l 2
q
q

(c) 10.1775 q 2 (d) none of these q q


2
 0 l
11. In an experiment with potentiometer, the balancing length is 250 cm for a cell. When the cell is
shunted by a resistance of 7.5 W, balancing point is shifted by 25 cm. If the cell is shunted by a
resistance of 20 W, the balancing length will be nearly
(a) 240 cm (b) 236 cm (c) 232 cm (d) 230 cm

4
61
5 7
12. One mole of a gas with   is mixed with two moles of another non-interacting gas with  
3 5
CP
The ratio of specific heats   of mixture is approximately
CV
(a) 1.50 (b) 1.46 (c) 1.49 (d) 1.53

13. An ideal gas is expanding such that PT3 = constant. The coefficient of volume expansion of the
gas is
1 2 3 4
(a) T (b) T (c) T (d) T

14. What is the magnetic induction B at the centre O of the semicircular arc if a current carrying wire
has shape of an hair pin as shown in figure? The radius of the curved part of the wire is R, the linear
parts are assumed to be very long.
0 I 0 I
(a) B  2    (b) B  2    I
4 R 4R R
O
3 I  2I
(c) B  0 2    (d) B  0
4R 4 R

15. A thin semi-circular metal ring of radius R has a positive charge q distributed uniformly over its
curved length. The resultant electric field E at the center O is j

q q
(a)  j (b) + j
2  0 R2
2
2  0 R2
2

q q
(c) + j 2 (d)  j
4  0 R2 4  0 R2
2 i

16. An Alternating Current is expressed as i = i1 cos wt + i2 sin wt. The RMS value of current is

2 2
(a) i1  i2  (b) i1 i2 (c) i12  i22 (d) i1 i2 
2 2 2 2

17. A charge + q is placed at each of the points x = x0, x = 3x0, x = 5x0, x = 7x0 ..............¥ on the x-axis
and a charge -q is placed at each of the points x = 2x0, x = 4x0, x = 6x0, x = 8x0 .......¥ here x0 is a
positive constant. Take the electric potential at a point due to a charge q at a distance r from it to be
1 q
V . The electric potential at the origin due to the above system of charges is
40 r
1 q 1 q
(a) zero (b) ln 2 (c) (d) infinite
40 x0 40 x0 2ln 2

5
61
23
18. The Nucleus 10 Ne decays by b - emission through the reaction
23 23
10 Ne  Na  10    energy The atomic masses are 10
11
23
Ne  22.994466 u and
0
23
11 Na  22.989770 u, 1
 = 0.000549 u. The maximum kinetic energy that the emitted electron
can ever have is
(a) 4.374 MeV (b) 3.862 MeV (c) 2.187 MeV (d) 1.931 MeV

19. The distance between two slits in Young's double slits experiment is d = 2.5 mm and the distance
of the screen from the plane of slits is D = 120 cm. The slits are illuminated with coherent beam of
light of wavelength  = 600 nm. The minimum distance (from the central maximum) of a point
where the intensity reduces to 25% of maximum intensity is
(a) 24 µm (b) 48 µm (c) 96 µm (d) 120 µm

20. What amount of heat will be generated in a coil of resistance R (ohm) due to a total charge Q
(coulomb) passing through it if the current in the coil decreases down to zero halving its value
every Dt second?
2 2 2
(a) 1 Q R (d) 1 Q R
2
(b) Q R ln2 (c) 1 Q R ln2
2 t t 2 t 4 t

21. In the L R circuit shown in figure, switch S is closed at time t = 0, the charge that passes through the
battery of emf E in one time constant is (e being the base of natural logarithm).

(a) EL (b) EL
eR2 eR
2
(c) eER (d) EL E s
L R
22. Natural Uranium is a mixture of 238
92 U
and 235
92 U
with a relative mass abundance of 140 : 1. The ratio
of radioactivity contributed by the two isotopes of natural uranium, if their half-lives are 4.5 ´ 109
8
years and 7.0 ´ 10 years respectively is
(a) 99.3 : 0.7 (b) 50.3 : 49.7 (c) 95.6 : 04.4 (d) cannot be estimated

23. A cylinder of length l > 1m filled with water ( = 43 ) up to the brim, kept on a horizontal table is
covered at its top by an equiconvex glass ( = 1.5) lens of focal length 25 cm when in air. At mid
day, 12.00 noon, Sun is just overhead and light rays comes parallel to the principal axis of the lens.
The sun rays will be focused
(a) 25 cm behind the lens in the water (b) 37.5 cm behind the lens in the water
(c) 50 cm behind the lens in the water (d) 100 cm behind the lens in the water

24. Even the radiation of highest wave length in the ultraviolet region of hydrogen spectrum is just
able to eject photoelectrons from a metal. The value of threshold frequency for the given metal is
(a) 3.83 ´ 1015 Hz (b) 4.33 ´ 1014 Hz (c) 2.46 ´ 1015 Hz (d) 7.83 ´ 1014 Hz

6
61
A-2
ANY NUMBER OF OPTIONS 4, 3, 2 or 1 MAY BE CORRECT
MARKS WILL BE AWARDED ONLY IF ALL CORRECT OPTIONS ARE BUBBLED AND NO WRONG OPTION

25. A parallel plate capacitor of plate area A and plate separation d is charged to potential V. Then the
battery is disconnected. A slab of dielectric constant k is then inserted between the plates of the
capacitor so as to fill the space between the plates completely. If Q, E and W denote respectively,
the magnitude of charge on each plate, the electric field between the plates (after the slab is
inserted) and work done on the system, in question, in the process of inserting the slab, then
 kAV V  AV 2 1
(a) Q  k 0 AE (b) Q  0 (c) E  (d) W  0 (1  )
d kd 2d k

26. The magnitudes of the gravitational field at distances r1 and r2 from the centre of a uniform solid
sphere of radius R and mass M are F (r1) and F (r2) respectively. Such that

F r1  r1
(a)  if r1  R and r2  R
F r2  r2
F r1  r22
(b)  if r1 R and r2 R
F r2  r12
F r1  r1
(c)  if r1 R and r2 R
F r2  r2
F r1  r12
(d)  if r1  R and r2  R
F r2  r22
27. The intensity of sound at a point P is I0, when the sounds reach this point directly and in same phase
from two identical sources S1 and S2. The power of S1 is now reduced by 64 % and the phase
difference (f) between S1 and S2 is varied continuously. The maximum and minimum intensities
recorded at P are now Imax and Imin such that
I 16
(a) Imax = 0.64 I0 (b) I min  0.36 I 0 (c) I max = 16 (d) max 
I min I min 9

28. An ideal monatomic gas is confined within a cylinder by a spring loaded piston of cross-sectional
area 4 ´10-3 m2. Initially the gas is at 400 K and occupies a volume 2 ´10-3 m3 and the spring is in
its relaxed position. The gas is heated by an electric heater for some time. During this time the gas
expands and the piston moves out by a distance 0.1 m. The spring connected to the rigid wall is
massless and frictionless. The force constant of the spring is 2000 Nm-1 and atmospheric pressure
is 105 Nm-2 then
T=400K
(a) The final temperature of the gas is 720 K.
(b) The work done by gas in expanding is 50 J
(c) The heat supplied by heater is 190 J
(d) The heat supplied by heater is 290 J

7
61
29. A particle of mass m is located in a one dimensional potential field U (x) = U0 (1 - cos ax);
U0 and a are constants. Which of the following statement/s is/are correct?
(a) The particle will execute Simple Harmonic Motion for small displacements.
(b) The stable equilibrium condition is x = 0
2 m
(c) The time period of small oscillations is
a U0
U0
(d) The angular frequency for small oscillations is   a
m

30. A ray of light is incident on an equilateral prism made of flint glass (refractive index 1.6) placed in
air.
(a) The ray suffers a minimum deviation if it is incident at angle 530.
(b) The minimum angle of deviation suffered by the ray is 460.
(c) If prism is immersed in water (  4
0
3 ) the minimum deviation produced by the prism is 14 .
(d) The minimum deviation produced by the prism is 23.60 if it is immersed in a liquid of refractive
index  = 1.2

31. In a p-n junction diode, the current (i) varies with applied biasing voltage (V) and can be expressed
as i  i0 e qV/kT  1 where i0 = 5 x 10 A is reverse saturation current, k is Boltzmann constant
-12

and q is the charge on the electron


At Absolute Temperature T = 300K
(a) The forward current is approximately 59.5 mA for a forward bias of 0.6 volt
(b) The current increases approximately by 2.75A if the biasing voltage changes from 0.6 V to 0.7 V
(c) The dynamic resistance of p-n junction is approximately 435 m  at the biasing voltage of 0.6 V
(d) The change in reverse bias current when biasing voltage change from -1 volt to -2 volt happens
to be practically zero.

32. A charged oil (density 880 kg m-3) drop is held stationary between two parallel horizontal metal
plates 6.0 mm apart when a potential difference of V = 103 volt is applied between the two plates.
When the electric field is switched off, the drop falls. At a certain time the drop is seen to fall a
distance of 2.0 mm in 35.7 s and next 1.2 mm in 21.4 s. (The upper plate in the experiment is at
higher potential).
Given that the viscosity of air = 1.80×10-5 Nsm-2 and density of air = 1.29 kg m-3
(a) The radius of the drop is a = 7.25 × 10-7m
-19
(b) The charge on the drop is q = 8.0 × 10 C
(c) The terminal velocity of the oil drop, under its free fall, is 5.6 × 10-5 ms-1
(d) The oil drop carries 5 excess electrons

8
61

Time: 9:00 AM to 10:15 AM


Question Paper Code: 61
Roll No. of
Student’s

Write the question paper code mentioned above on YOUR OMR Answer Sheet (in the space
provided), otherwise your Answer Sheet will NOT be evaluated. Note that the same Question
Paper Code appears on each page of the question paper.

Instructions to Candidates:

1. Use of mobile phone, smart watch, and iPad during examination is STRICTLY PROHIBITED.
2. In addition to this question paper, you are given OMR Answer Sheet along with candidate’s copy.
3. On the OMR sheet, make all the entries carefully in the space provided ONLY in BLOCK
CAPITALS as well as by properly darkening the appropriate bubbles.
Incomplete/ incorrect/ carelessly filled information may disqualify your candidature.
4. On the OMR Answer Sheet, use only BLUE or BLACK BALL POINT PEN for making entries
and filling the bubbles.
5. Your 14-digit roll number and date of birth entered on the OMR Answer Sheet shall remain
your login credentials means login id and password respectively for accessing your performance /
result in Indian Olympiad Qualifier in Physics 2021-22 (Part I).
6. Question paper has two parts. In part A1 (Q. No.1 to 24) each question has four alternatives, out of
which only one is correct. Choose the correct alternative and fill the appropriate bubble, as below.
Q.No.12 a c d
c c
d
In part A-2 (Q. No. 25 to 32) each question has four alternatives out of which any number of
alternative(s) (1, 2, 3, or 4) may be correct. You have to choose all correct alternative(s) and fill
the appropriate bubble(s), as shown
Q.No.30 a c
7. For Part A-1, each correct answer carries 3 marks whereas 1 mark will be deducted for each
wrong answer. In Part A-2, you get 6 marks if all the correct alternatives are marked and no
incorrect. No negative marks in this part.
8. Rough work should be done in the space provided. There are 11 printed pages in this paper
9. Use of non- programmable scientific calculator is allowed.
10. No candidate should leave the examination hall before the completion of the examination.
11. After submitting answer paper, take away the question paper & Candidate’s copy of OMR for your
reference.

Please DO NOT make any mark other than filling the appropriate bubbles
properly in the space provided on the OMR answer sheet.

OMR answer sheets are evaluated using machine, hence CHANGE OF ENTRY IS
NOT ALLOWED. Scratching or overwriting may result in a wrong score.

DO NOT WRITE ON THE BACK SIDE OF THE OMR ANSWER SHEET.

1
61

Instructions to Candidates (Continued) :


You may read the following instructions after submitting the answer sheet.

12. Comments/Inquiries/Grievances regarding this question paper, if any, can be shared on the
Inquiry/Grievance column on www.iapt.org.in on the specified format till January 22, 2022.
13. The answers/solutions to this question paper will be available on the website:
www.iapt.org.in by January 20, 2022.

14. CERTIFICATES and AWARDS:


Following certificates are awarded by IAPT to students, successful in the
Indian Olympiad Qualifier in Physics 2021-22 (Part I)
(i) “CENTRE TOP 10 %” To be downloaded from iapt.org.in after 15.03.22
(ii) “STATE TOP 1 %” Will be dispatched to the examinee
(iii) “NATIONAL TOP 1 %” Will be dispatched to the examinee
(iv) “GOLD MEDAL & MERIT CERTIFICATE” to all students who attend OCSC – 2022 at
HBCSE Mumbai
Certificate for centre toppers shall be uploaded on iapt.org.in
15. List of students (with centre number and roll number only) having score above MAS will
be displayed on the website: www.iapt.org.in by February 06, 2022 See the
Minimum Admissible Score Clause on the student’s brochure on the web.
16. List of students eligible for evaluation of IOQP 2021-22 (Part II) shall be displayed on
www.iapt.org.in by February 10, 2022.

Physical constants you may need….

Mass of electron me  9.10 10


31
kg Magnitude of charge on electron e  1.60 1019 C

27
Mass of proton m p  1.67 10 kg Permittivity of free space  0  8.85 1012 C 2 N 1m2

Acceleration due to gravity g = 9.81 ms-2 Permeability of free space 0  4 107 Hm1
11
Universal gravitational constant G  6.67 10 Nm2 Kg  2 Planck’s constant h  6.63 1034 Js

Universal gas constant R  8.31 Jmol 1 K 1 Faraday constant  96,500 Cmol 1

Boltzmann constant k  1.38 1023 J K1 Rydberg constant R  1.097 10 m


7 1

Stefan’s constant   5.67 108 Wm2 K 4 Density of water at 4 C,   1.0 103 kg m3

Avogadro’s constant A  6.023 1023 mol 1 Density of mercury   13.6 103 kg m3

Speed of light in free space c  3.0 108 m s1 Speed of sound in air = 330 m s1

2
61

PHYSICS 2021-22 (Part I) (NSEP 2021 – 22)


Time: 75 Minute Max. Marks: 120
Attempt All Thirty Two Questions
A–1
ONLY ONE OUT OF FOUR OPTIONS IS CORRECT. BUBBLE THE CORRECT OPTION.

1. Consider the process of the melting of a spherical ball of ice originally at 0 C . Assuming
that the heat is being absorbed uniformly through the surface and the rate of absorption is
proportional to the instantaneous surface area. Which of the following is true for the radius
(r) of the ice ball at any instant of time? Assume that the initial radius of the ice ball at
t  0 is r  R and that the shape of the ball always remains spherical during melting. Also
0
assume that L and  are respectively the latent heat and density of ice at 0 C
 kt
(a) radius decreases exponentially with time as r  R0 e  L . Here k is constant
 k t
(b) radius decreases exponentially with time as r  R0 e 2 L
k
(c) radius of the ice ball decreases with time linearly with a slope 
L
k
(d) radius of the ice ball decreases with time linearly with a slope 
2L
2. The work done by the three moles of an ideal gas in the cyclic process ABCD shown in the
diagram is approximately. Given that
T1 = 100 K, T2 = 200 K and
T3 = 600K, T4 = 300 K

(a) 7.5 kJ (b) 5.0 kJ


(c) 2.5 kJ (d) Zero

3. The molar specific heat capacity of a certain gas is expressed as C  C  


P.
V T
The equation of state for the process can be written as (α & A are constant)
V
(a) PV = RT (b) V   T 2 (c) V 2  lnT (d) T  Ae

4. A metal bar of length moves with a velocity v parallel to an infinitely long straight wire
carrying a current I as shown in the figure. If the nearest end of the perpendicular bar always
remains at a distance 2 from the current carrying wire, the potential difference (in volt)
between two ends of the moving bar is
v
0 Iv 0 Iv
(a) (b)
2 6
2l
 Iv  Iv
(c) 0 n 2 (d) 0 n 1.5
2 2 II

3
61

5. Two point charges +Q each are located at (0, 0) and (L, 0) at a distance L apart on the
X - axis. The electric field (E) in the region 0 < x < L is best represented by

(a) Fig. a (b) Fig. b (c) Fig. c (d) Fig. d

6. A long straight wire AB of length L (L >> a, L >> b) and resistance R is connected to a time
varying source of emf V(t). The variation of applied emf V(t) with time is shown in
Fig. B. A circular metallic loop of radius r = b is placed coplanar with the current carrying
wire with its centre at a distance 'a' from the axis of the wire as shown. The induced current
in the loop is

V(t)
b

A B 0 T/2 t T
L
Fig. B

+ –
Fig. A V(t)
(a) clockwise from 0 to T/2 and anticlockwise from T/2 to T
(b) anticlockwise from 0 to T/2 and clockwise from T/2 to T
(c) clock wise from 0 to T
(d) anticlockwise from 0 to T

7. A simple circuit consists of a known resistance RA  2 M  and an unknown resistance RB


both in series with a battery of 9 volt and negligible internal resistance. When the voltmeter is
connected across the resistance RA, it measures 3 volt but when the same voltmeter is
connected across RB it reads 4.5 volt. The voltmeter measures 9 V across the battery.
Considering that the voltmeter has a finite resistance r, the correct option is

(a) RB  3M  and r  6.0 M  (b) RB  2.5M  and r  6.0 M 

(c) RB  4M  and r  12 M  (d) RB  4.5M  and r  6.0 M 

8. The optical powers of the objective and the eyepiece of a compound microscope are 100 D
and 20 D respectively. The microscope magnification being equal to 50 when the final image
is formed at d = 25 cm i.e., the least distance of distinct vision. If the separation between the
objective and the eyepiece is increased by 2 cm, the magnification of the microscope will be
(a) 62 (b) 50 (c) 38 (d) 25

4
61

9. A hollow non-conducting cone of base radius R = 50 cm and semi vertical angle of 15 0 has
been uniformly charged on its curved surface up to three-fourth of its slant length from base
with a surface charge density   2.5 C / m2 . The electric field produced at the location of the
vertex of the cone is
 ln 2  ln 2  ln 2  ln 2
(a) (b) (c) (d)
2 0 4 0 8 0 16 0

10. A freely falling spherical rain drop gathers moisture (maintaining its spherical shape all the
dm
way) from the atmosphere at a rate  k t 2 where t is the time and m is the instantaneous
dt
mass of the drop, the constant k 12 gm / s3 . If the drop, of initial mass m0  2gm, starts
falling from rest, the instantaneous velocity of the drop exactly after 5 second shall be
(ignore air friction and air buoyancy)

(a) 12.4 ms-1 (b) 49.0 ms-1 (c) 122.5 ms-1 (d) data insufficient

11. Two planets, each of mass M and radius R are positioned (at rest) in space, with their centres
a distance 4R apart. You wish to fire a projectile from the surface of one planet to the other.
The minimum initial speed for which this may be possible is

2GM 2GM 4GM 3GM


(a) (b) (c) (d)
5R 3R 3R 2R

12. A thin uniform metallic rod of length L and radius R rotates with an angular velocity ω in a
horizontal plane about a vertical axis passing through one of its ends. The density and the
Young's modulus of the material of the rod are  and Y respectively.
The elongation in its length is

  2 L3   2 L3 ω
(a) (b)
6Y 3Y
L
  2 RL2   2 L3
(c) (d)
2Y 2Y

13. Consider a particle of mass m with a total energy E moving in a one dimensional potential
field. The potential V(x) is plotted against x in the figure beside.
The plot of momentum – position graph of this particle is qualitatively
best represented by

All plots are symmetrical about x - axis


(a) Fig. a (b) Fig. b (c) Fig. c (d) Fig. d

5
61

14. Knowing that the parallel currents attract, the inward pressure on the curved surface of a thin
walled, long hollow metallic cylinder of radius R = 50 cm carrying a current of i = 2 amp
parallel to its axis distributed uniformly over the entire circumference, is

(a) 2.0510 1 Nm  2 (b) 2.5510 3 Nm  2 (c) 2.0510 5 Nm  2 (d) 2.5510 7 Nm 2

15. Two masses move on a collision path as shown. Before the collision the object with mass
3
2M moves with a speed v making an angle   sin 1
to the x-axis while the object with
5
3 4
mass M moves with a speed v making an angle   sin 1 with the x-axis. After the
2 5
collision the object of mass 2M is observed to be moving to the right along the x-axis with a
4
speed of v . There are no external forces acting during the collision. The correct option is
5

Before the
2M collision After the collision

 2M M

(a) The velocity of mass M, after the collision, is zero.


(b) The centre of mass is moving along x-axis before the collision.
5
(c) The velocity of centre of mass after the collision is v
2
5
(d) The total linear momentum of the system before the collision along x - axis is Mv
6

16. A large hemispherical water tank of radius R is filled with water initially upto a height
R
h . The water starts dripping out through a small orifice of cross section area 'a' at its
2
spherical bottom. The time taken to get the tank completely empty (neglect viscosity) is
19  R 2 R 3 R 2 R
(a) t  (b) t 
60a g 10a g
17  R 2 R  R2 R
(c) t  (d) t 
60a g 4a g

17. If Pascal (Pa), the unit of pressure volt (V), the unit of potential and meter (L), the unit of
length are taken as fundamental units, the dimensional formula for the permittivity  0 of
free space is expressed as

(a) Pa– 1V 2 L– 2 (b) Pa1V – 2 L2 (c) Pa1V2L– 2 (d) Pa– 1V – 2 L2

6
61

18. A cycle wheel of mass M and radius R fitted with a siren at a point on its circumference, is
mounted with its plane vertical on a horizontal axle at about 3 feet above the ground. An
observer stands in the vertical plane of the wheel at 100 m away from the axle of the wheel
on a horizontal platform. The siren emits a sound of frequency 1000 Hz and the wheel rotates
clockwise with a uniform angular speed    rad / sec. Initially at t = 0 sec the siren is
nearest to the observer and moves downwards. The observer records the highest pitch of
sound for the first time after (speed of sound in air is 330 ms -1)

(a) 0.30 s (b) 1.8 s (c) 2.3 s (d) 9.8 s

19. On a right angled transparent triangular prism ABC, when a ray of light is incident on face
AB, parallel to the hypotenuse BC, it emerges out of the prism grazing along the surface AC.
If instead the ray is made incident on face AC, parallel to the hypotenuse CB it gets totally
reflected on face AB. The refractive index µ of the material of the prism is
3 3
(a)   2 (b) 2 (c) 3    2 (d)  
2 2

20. A circular disc of radius R = 10 cm is uniformly rolling on a horizontal A


surface with a velocity v = 4 ms-1 of centre of mass without slipping,
the time taken by the disc to have the speed of point A (which lies on B
the circumference) equal to the present speed of point B (point B lies
midway between centre and the point A) is

(a) t = 0.025 s (b) t = 0.036 s


(c) t = 0.046 s (d) t = 0.064 s

5
21. As shown in the figure, a particle of mass m  1010 kg, moving with velocity v0 = 10 m/s
approaches a stationary fixed target with impact parameter b from a large distance. If the
K
fixed rigid target has a core with repulsive central force F  r   3 where constant K > 0 and
r
the particle scatters elastically. The closest distance of approach (if numerically K = b2) is

b –3
Kr

(a) b (b) b 2 (c) b 3 (d) 2b

3
22. If the specific activity of C14 nuclide in a certain ancient wooden toy is known to be of that
5
in a recently fallen tree of the same class, the age of the ancient wooden toy is
(The half life of C14 is 5570 years )

(a) 5570 years (b) 4105 years (c) 3342 years (d) 2785 years

7
61

In questions 23 and 24 mark your answer as


(a) If statement I is true and statement II is true and also if the statement II is a correct
explanation of statement I
(b) If statement I is true and statement II is true but the statement II is a not a correct
explanation of statement I
(c) If statement I is true but the statement II is false
(d) If statement I is false but statement II is true

23. Statement I: Work done in bringing a charge q from infinity to the center of a uniformly
charged non – conducting solid sphere of radius R (with a total charge Q)
is zero.
Statement II: The potential difference between the Centre and the surface of the uniformly
charged non – conducting solid sphere of radius R (with a total charge Q)
1 Q
is  .
4 0 2R

24. Statement I: The current flowing through a p-n junction is more in forward bias than that in
the reverse bias.
Statement II: The diffusion current, dominant in forward bias, is more than the drift
current, dominant in the reverse bias.

8
61

A- 2
ANY NUMBER OF OPTIONS 4, 3, 2 or 1 MAY BE CORRECT
MARKS WILL BE AWARDED ONLY IF ALL THE CORRECT OPTIONS ARE BUBBLED

25. A simple pendulum consisting of a small bob of mass m attached to a massless inextensible
string of length  2m , hanging vertically from the ceiling, is oscillating in a vertical plane
with an angular amplitude  m such that the maximum tension in its string is three times the
minimum tension in the string i.e., Tmax = 3Tmin. The correct option(s) is/are

(a) The maximum tension in the string is Tmax = mg (3 – 2cos θm)


9
(b) The maximum tension in the string is Tmax  mg
5
(c) The maximum velocity of the bob on its way is vmax = 3.96 ms-1
 
(d) The angular amplitude θm lies in the range  m 
4 3

26. Two small masses m and M lie on a large horizontal frictionless circular track of radius R.
The two masses are free to slide on the track but constrained to move along a circle. Initially
the two masses are tied by a thread with a compressed spring between them (spring of
negligible length being attached with none of the two masses). The compressed spring stores
a potential energy U0. At a certain time t = 0 the thread is burnt and the two masses are
released to run opposite to each other leaving the spring behind. The total mechanical energy
remaining conserved. On the circular track the two masses make a head on perfectly elastic
collision. Take M = 2m for all calculations. Which of the following option(s) is / are correct?


(a) The angle turned by mass m before the collision is   4
3
4U 0
(b) The velocity of mass m on the track is u 
3m
m
(c) The time taken to collide for the first time is t1  2 R
3U
0
2m
(d) The time taken for the second collision is t2  2 R
3U
0

27. The electric field component of an electromagnetic wave is expressed as


E   3 j  b k  10 sin 10  x  2 y  3z   t  in SI units. Taking c  3 108 m s1 as the speed of
3 7

electromagnetic wave in vacuum, choose the correct option(s)

(a) The value of constant beta is   3 108  14


(b) The value of constant b is b = 2.
(c) The average energy density of the em wave is U  6.5 106  0 in SI units.
(d) The amplitude of magnetic field is B 1.20 10 11 Tesla

9
61

28. A parallel beam of light is made incident (as shown) on the flat diametric
plane of a transparent semi-circular thin sheet of thickness t (t << R)
of refractive index   2 at an angle of 450. As a result of
refraction, the light enters the semi-circular sheet and comes
out at its curved surface.

(a) Light rays come out at the curved surface for values of θ in
the range 750 ≤ θ ≤ 1650.
(b) The range of angle θ is independent of the angle of incidence.
(c) The range of angle θ depends on the refractive index of the material
(d) All the emergent rays of light shall cross the line OP which is a refracted ray at  1200
Here θ is the angle between the vertical diameter AB and the concerned radius of the
semicircular sheet of radius R.

29. A certain rod of uniform area of cross section A (A = 1.0 cm2) with its length = 2 m is
thermally insulated on its lateral surface. The thermal conductivity  K  of the material of the

rod varies with temperature T as K  where α is a constant. The two ends of the rod are
T
maintained at temperature of T1  90 C and T2  10 C. The correct option(s) is /are
(a) The temperature at 50 cm from the colder end is 17.32 C
(b) The temperature at 50 cm from the hotter end is 51.96 C
(c) The rate of heat flow per unit area of cross section of the rod is 1.1 in SI units.
(d) The temperature gradient is numerically higher near the hot end compare to that near the
cold end.

30. Positronium is a short-lived ( 10  9s) bound state of an electron and a positron (a positively
charged particle with mass and charge equal (in magnitude) to an electron) revolving round
their common centre of mass. If E0, v0 and a0 are respectively the ground state energy, the
orbital speed of electron in first orbit and the radius of the first (n = 1) Bohr orbit for
Hydrogen atom, the corresponding quantities E, v and a for the positronium are

E
(a) E  0 (b) a  a0 (c) a  2a0 (d) E  E0 , v  v0 , a  a0
2

10
61

31. A thin double convex lens of radii of curvature R1 = 20 cm and R2 = 60 cm is made-up of a


transparent material of refractive index μ = 1.5. Choose the correct option(s)

(a) The focal length of the lens is f = 30 cm when in air.


(b) The lens behaves as a concave mirror of focal length f M  10 cm when silvered on the
surface of radius R2 = 60 cm
(c) The lens behave as a concave lens (diverging lens) if the image space beyond R2 = 60 cm
5
radius surface is filled with a transparent liquid of refractive index   . The object
3
space prior to the surface of radius R1 = 20 cm is air.
(d) A beam of rays incident parallel to principal axis focuses at 48 cm behind the lens if
water     fills the entire space behind the surface of radius R2 = 60 cm. The object
4
 3
space prior to the surface of radius R1 = 20 cm is air.

32. A thick hollow cylinder of height h and inner and outer radii a and b (b > a) made up of a
poorly conducting material of resistivity ρ lies coaxially inside a long solenoid at its middle.
The radius of the solenoid is larger than b. Throughout the interior of the solenoid, a uniform
time varying magnetic field B   t is produced parallel to solenoid axis. Here β is a constant.
In this time varying magnetic field

(a) the emf induced at a certain radius r (a < r < b) in the hollow cylinder is  r 2 
(b) the induced current circulating in the thick hollow cylinder between radii a and b is
i
4

h 2 2
b a 
(c) the resistance offered to the circulation of current by the thick hollow cylinder is
2
R
b
h  ln
a
(d) no electric field is detectable outside the solenoid.

11
61

ROUGH WORK

12

You might also like